LMFT Clinical Exam - Practice Exam VI (TDC-II) ヽ(〃·◇·)ノ

¡Supera tus tareas y exámenes ahora con Quizwiz!

A therapist is providing ongoing therapy for a 13-year-old girl who was molested by her mother's ex-boyfriend. In assessing the impact the abuse has had on the client, the therapist should be most concerned with the client's: A. Physical health B. Family relationships C. Coping skills D. Psychosocial functioning

D. Psychosocial functioning Rationale: In assessing the impact of a traumatic experience the therapist is most concerned with the client's psychosocial functioning (D), which is an encompassing term that includes both coping skills (C), and client's relationships (B). As therapists, it is outside our scope of practice to assess physical health (A) and for that we would refer the client to an M.D. (Clinical Evaluation)

Sandra, a 54-year-old self-employed accountant, is referred to therapy by a close friend. Sandra has been feeling tense and nervous since her elderly mother moved in five months ago. Tearfully, she explains, "I just can't keep up with mom's round-the-clock needs on my own. It's overwhelming. The medications are so confusing. .Sometimes I just skip them for a couple days and then double up. I can't afford to hire a caregiver and I won't put her in a home. I don't know what to do. I can't take this much longer!" Sandra confesses that she has been taking her mother's muscle relaxant medication frequently to cope with stress. She appears fatigued and occasionally loses focus during the session. What interventions should a Psychodynamic therapist use in the middle stage of therapy if Sandra is unable to recognize the unhealthy defense mechanisms she is employing? A. Interpret Sandra's current need to take care of her mother as a response to unresolved issues stemming from early childhood; Assist Sandra in identifying times when she has been able to take care of her mother. B. Ask Sandra to invite her mother for a session to explore alternative living arrangements; Interpret Sandra's substance use as a way to avoid dealing with her anger toward her mother for being a burden. C. Monitor Sandra's safety and the safety of her mother; Interpret Sandra's current need to take care of her mother as a response to unresolved issues stemming from early childhood. D. Interpret Sandra's substance use as a way to avoid dealing with her anger toward her mother for being a burden; Provide referral to Narcotics Anonymous.

C. Monitor Sandra's safety and the safety of her mother; Interpret Sandra's current need to take care of her mother as a response to unresolved issues stemming from early childhood. Rationale: The first part of answer (A) is both consistent with a Psychodynamic perspective that takes into consideration early childhood experiences, and acts as an appropriate intervention for a middle stage of this theory. The second half of answer (A) is more fitting for a Solution Focused therapist rather than a Psychodynamic one, which means (A) is out. Answer (B) is wrong since the first part has us inviting Sandra's mother into the room and at this point, such intervention is not indicated. The second half of answer (B) is plausible, but the first half rules it out. Answer (C) is the best answer since monitoring both Sandra's and her mother's safety is something we are going to do throughout this case due to Sandra's hopelessness, drug use, and poor medication management of her mother. The second half of this answer has already been discussed as part of answer (A). Answer (D) is incorrect because it is assuming a particular emotion (anger) which may or may not be accurate and it's unclear if a referral to NA is appropriate at this time. (Treatment)

Natalia, a 47-year-old, and Igor, a 49-year-old, come in with their 16-year-old daughter Olga. They are referred to therapy by Olga's school. The initial assessment indicates that Olga has all the symptoms of anorexia. Natalia complains that all their attention is focused on Olga. Which of the following is the most appropriate treatment unit? A. The parents in order to strengthen their relationship in light of the mother's expressed concern about the family dynamics. B. The daughter alone since the assessment indicates she has symptoms of anorexia that must be addressed. C. The entire family in light of the mother's expressed concern about the family dynamics. D. The mother alone in light of her expressed concern about the family dynamics.

C. The entire family in light of the mother's expressed concern about the family dynamics . Rationale: One of the best ways to determine the unit of treatment is to see who is in the room with you. Since the whole family is here right now, and one of the presenting complaints is that the family's complete focus is on the daughter, it is safe to say that the entire family should be the unit of treatment. Seeing the daughter (B), the mother (D), or the parents (A) alone will not help the family deal with the complaint that was brought up. (Clinical evaluation)

Francisco, a 32-year-old construction worker, is referred by his EAP because of troubles at work. Francisco tells the therapist that co-workers are constantly conspiring against him. He is convinced they are discussing the most profitable projects behind his back and intentionally leaving him out of important meetings. "I do not trust anyone in that company; they are just like my family. They are out to get me," says Francisco at the end of the first session. His most likely diagnosis is: A. Delusional Disorder B. Paranoid Personality Disorder C. Schizophrenia D. Shared Psychotic Disorder

B. Paranoid Personality Disorder Rationale: According to the DSM 5, Paranoid Personality Disorder (B) is characterized by a pervasive distrust and suspicion of others such that their motives are malevolent. Since the symptoms appear to be pervasive ("they are just like my family"), the symptoms better describe Paranoid Personality Disorder rather than Delusional Disorder (A). The client is not exhibiting any psychotic symptoms; ruling out Schizophrenia (C) and the symptoms do not appear to be developing in conjunction with another individual, ruling out Shared Psychotic Disorder (D). (Diagnosis)

A 35-year-old lawyer is referred to therapy by his physician for symptoms of anxiety and panic. His physician wants the client to initiate therapy before prescribing medication. The client, however, believes that only medication will control his symptoms and that therapy is a "waste of time." Which of the following interventions would a Cognitive Behavioral therapist use to address the client's perspective on participation in therapy? A. Interpret the client's distorted cognitions and identify their connection to his current symptoms of anxiety and panic. B. Explore the client's disappointment with his physician and validate his automatic thoughts as a logical consequence. C. Examine the client's assumptions regarding treatment and collaborate with him to promote a shift in personal conclusions. D. Provide psychoeducation on the benefits of Cognitive Behavioral therapy and explore his previous experience with therapy.

C. Examine the client's assumptions regarding treatment and collaborate with him to promote a shift in personal conclusions. Rationale: The best answer is (C) as it attends to his views of therapy. CBT therapists don't interpret, so answer (A) is out. There isn't an indication of the client being "disappointed" with the doctor so that knocks out (B). You could do (D) after doing (C). (Treatment)

Betty, a 66-year-old woman, is referred for therapy by her Employee Assistance Program (EAP). She has been written up for poor job performance, multiple absences, and tardiness. She tearfully states that she was recently diagnosed with liver cancer, and has subsequently been feeling really down. She informs the therapist that her husband is unsupportive and demanding. She affirms that she has been sober for the last 6 years and says, "Although, lately, I have been thinking of how good a glass of red wine would taste right about now. I have a great AA sponsor and she helps me with my sobriety. So you do not have to worry about that. I just want you to help me deal with cancer from all angles." How should the therapist clinically manage the therapeutic implications of the EAP referral? A. Help the client prioritize the issues she is dealing with; Notify the client's employer when she has met her goals; Establish goals that are objective; Obtain a release to speak to Betty's boss B. Obtain a full family history; Establish a clear target date for achievement of goals; Establish goals based on the client's priorities; Explain what information will be shared with the EAP C. Ask the client to clarify what she means by "I just want you to help me deal with cancer from all angles"; Establish goals that are incremental; Provide all the information requested by the client's employer; Refer client to a psychiatrist for a medication evaluation D. Inform the client that treatment is going to be short-term and problem-resolution focused; Establish goals that are attainable and measurable; Review progress and provide necessary referrals; Explain what information will be shared with the EAP

D. Inform the client that treatment is going to be short-term and problem-resolution focused; Establish goals that are attainable and measurable; Review progress and provide necessary referrals; Explain what information will be shared with the EAP Rationale: In order to answer this question correctly it is important to keep in mind the specifics of working with EAP as it relates to this case. Answer (A) is wrong for several reasons. We would not notify the employer when the goals have been met, we would only share information for the purpose of reimbursement with the EAP. In addition, we are not after objective goals, but rather we would aim to create attainable and measurable goals. The final part is technically correct, we would need to obtain a release to speak with the client's boss, but there is nothing in the vignette to indicate we need to do this or that it would be an appropriate action to take. Answer (B) is wrong since we will not be obtaining family history as part of short term and problem-resolution focused EAP treatment. Answer (C) is out since not all parts relate to EAP treatment. Plus we would not provide information to the employer, we would only provide information to the EAP. All parts in answer (D) pertain to working as an EAP provider; thus it is the best answer. (Case Conceptualization and Planning)

A recent Chinese immigrant is seen in a hospital emergency room after several panic attacks. He is referred to a therapist but is resistant to receive mental health treatment. During the initial assessment, the therapist should consider all of the following as likely explanations for this refusal except: A. High incidence of anxiety disorder within the Chinese culture. B. Possible language barriers. C. Alternative treatments available in the culture. D. Cultural stigmas associated with treatment.

A. High incidence of anxiety disorder within the Chinese culture. Rationale: The correct answer is (A) since there is no evidence to support the fact that anxiety is highly prevalent in Chinese culture. Even if it is, it would not provide an adequate explanation as to why the client is refusing treatment, versus options (B), (C) and (D), which are all possible explanations for such refusal. (Clinical Evaluation)

A 15-year-old girl with a diagnosis of childhood diabetes is brought into therapy by her parents following a referral by the child's pediatrician. Recently, she has been fighting with her siblings and parents. Although she continues to receive good grades, she demonstrates an increasing need to be in control at home and is rebelling against family rules and her required diet. Which of the following conclusions should the therapist make based on the information shared by the family? Choose one A. The symptoms are a result of mood fluctuations caused by the possible misuse of prescribed medication. B. The symptoms are a result of normal adolescent behavior. C. The symptoms are a result of an irregular diet that is causing fluctuations in the client's mood and behavior. D. The symptoms are a result of the client's need to control whatever parts of her life she can.

B. The symptoms are a result of normal adolescent behavior. Rationale: The child's behaviors may represent a common response in children who feel they are lacking control in certain aspects of his/her life (D); however because they are recent, the behaviors are more likely representative of normal adolescent behavior (B). There is no indication that she is misusing her prescribed medication (A). There is a possibility that her diet is causing fluctuations in her mood (C), but the behaviors can be better explained by adolescence. (Clinical Evaluation)

A 28-year-old attends an intake session at a county mental health clinic. He was referred by his primary care physician to see a mental health practitioner due to symptoms of anhedonia. He reports feeling somber more often than not, although he notes that there are times his mood lifts. The therapist observes the client has an impoverished speech pattern, makes minimal eye contact and provides limited responses to questions asked by the therapist. What diagnosis would be most appropriate for this client? A. Major Depressive Disorder B. Bipolar 2 Disorder C. Cyclothymia D. Depressive Disorder Unspecified

D. Depressive Disorder Unspecified Rationale: Best answer here is D. This client clearly has depressive symptoms, yet they do not appear to be severe enough to impede his daily functioning. Answer A can be tempting, yet we have to consider the severity of symptoms when making this distinction. Answers B and C are out since there is no evidence of hypomania. (Diagnosis)

Sally, a 66-year-old widow who lost her husband five months ago, is self-referred. Sally recently retired from her job at the post office. Lately, she has lost her appetite and has been experiencing trouble sleeping and concentrating. She reports that the only thing she enjoys anymore is spending time with her 8-year-old granddaughter. She takes her to school and to ballet practice. Otherwise, Sally just feels useless and is concerned that she will become a burden to her daughter's family. She tears up when she says, "I'm having a hard time getting old. I hate being alone. I still pretend my husband is just at work. I keep praying that he'll come back. I now know why my husband used to refer to his drink as 'old faithful.' What initial interventions should the therapist consider in the case provided in the vignette? A. Acknowledge the changes that have occurred in her life and assess her safety. B. Identify potential volunteer opportunities and assess her level of alcohol use. C. Provide psychoeducation about life stages and assess her safety. D. Determine her level of social support and provide psychoeducation about grief and loss.

A. Acknowledge the changes that have occurred in her life and assess her safety. Rationale: Answer (A) is the best answer. You'd want to start by acknowledging what she's been through. In addition, since she makes several concerning comments, we want to prioritize an answer that assesses her safety (B) is a leap too far forward and does not address the more immediate concerns. This could be something that is considered later on in therapy, but it would not be the starting point. Psychoeducation about life stages could be done, but would only happen after acknowledging the changes (C). The client notes that she fears becoming a burden to her daughter, but is engaged with the family, as seen by her time with the granddaughter. Thus, the first part of answer D is not indicated. And, as noted earlier, psychoeducation would come after acknowledging the changes and assessing safety. (Crisis management)

William, age 59, brings his daughter Terrie, age 15, for therapy. William says, "Terrie has been cutting for a year, cries in her bedroom, and worries about everything." William's wife, Florence, age 58, is a Scientologist and against psychotherapy. She's been treating Terrie with homeopathic supplements and exercise. Terrie shares that she thinks about death and tried to hang herself a few years ago. She shows you the scars and fresh cuts on her right forearm. Terrie says, "I show my parents these and they do nothing about it, even when the cuts are deep and bleeding really badly." Terrie begins to cry and says to her dad angrily, "I don't want to be homeschooled! You guys shouldn't have adopted us. All you care about is your business." William shakes his head and says, "I'm too old to deal with this. How much is this all going to cost?" How would a Systems therapist incorporate human diversity into the treatment plan based on the case presented in the vignette? A. Ask William and Terrie to tell you how Florence's religious beliefs impact their family; explore how being older parents impacts the different family members; explore their experience of being an adoptive family and how it affects their understanding of the situation B. Remove Terrie's position of the identified patient by confronting Florence's religious beliefs; evaluate the validity of Terrie's beliefs that her parents do not care about her; explore the experience of being an adoptive family and how it affects their understanding of the situation. C. Educate them on how being an adoptive family may affect their understanding of the situation; Explore Florence's religious beliefs and how they affect Terrie's treatment; discuss how the other family members are impacted by Terrie's problems D. Explore how being older parents affects William's view of the family's issues; ask William and Terrie to tell you how Florence's religious beliefs impact Terrie's illness; determine whether homeschooling is the best option for Terrie

A. Ask William and Terrie to tell you how Florence's religious beliefs impact their family; explore how being older parents impacts the different family members; explore their experience of being an adoptive family and how it affects their understanding of the situation Rationale: The right answer for this question needs to respond to the issues presented, be consistent with the concepts of System's theory, and include issues of human diversity that are playing out in this case and are unique to this family. Answer (A) includes all of these components. The parts of the answer are geared towards all family members and include words like "explore" and "ask", which is consistent with System's theory. Florence's religious beliefs are certainly influential in this case as they are rather specific. Being older parents as well as an adoptive family has its own set of influences that not all families are dealing with. Answer (B) is wrong since a System's therapist is not going to confront Florence's religious beliefs. Doing so will not help to remove Terri's position of an IP. The second part of answer (B) only addresses Terry and a System's therapist would address the dynamic of the whole family. Answer (C) is not the best answer, since it has the therapist educating the family on the impact of being an adoptive family, as opposed to exploring their view on how it is impacting the situation. The family is going to know more about their experience than the therapist would. Answer (D) is not fitting since the first half deals only with William and leaves the rest of the family out. The second half of answer (D) targets Terrie as an IP, something a Systems therapist would avoid doing. In addition, the therapist is not going to determine whether homeschooling is the best option; that would be imposing the therapist's views onto the family. (Case Conceptualization and Planning)

A family is referred for therapy after the adoption of their 8-year-old nephew. The adopted child's mother was incarcerated for selling drugs and the father is deceased. The parents report that the child hoards food, is frequently aggressive with their other children, ages 9 and 6, and tends to reject their efforts to include or console him. Which of the following long-term goals should be included in the treatment plan for this family? A. Identify each family member's reaction regarding the recent adoption; Increase levels of trust between family members; Establish and maintain healthy family connections. B. Increase the level of trust between family members; Establish and maintain healthy family connections; Increase the family's ability to identify as a cohesive unit. C. Improve the adopted son's self-confidence and impulse control; Increase the family's ability to identify as a cohesive unit; Strengthen family cohesion. D. Improve the adopted son's self-confidence and impulse control; Identify each family member's reaction regarding the recent adoption; Establish and maintain healthy family connections.

B. Increase the level of trust between family members; Establish and maintain healthy family connections; Increase the family's ability to identify as a cohesive unit. Rationale: The question wants the "long-term goal" for the entire family so B is the best answer that addresses the whole unit and would be what the therapy should work toward. A, C, and D each include shorter-term goals as well as immediate interventions. (Case Conceptualization and Planning)

A 16-year-old client is referred by her teacher and is brought to treatment by her mother who complains that her daughter frequently skips school and has been found drinking by herself multiple times. The daughter sits quietly as the mother adds that her older brother died in a car accident a year ago and the daughter "hasn't been the same since." The daughter adds, "Tell that to dad, he's the one that needs help, everyone in our family tries to pretend everything is ok when it's not!" What should the therapist further assess in this case? A. Academic functioning; Frequency and quantity of alcohol use; Provide referral for psychiatric evaluation B. Provide medical referral for physical evaluation; Academic functioning; Social supports C. Complete a comprehensive substance use assessment; Appropriateness of teen grief group; Readiness for family therapy D. Appropriateness of teen grief group; Client's stage of change; Readiness for family therapy

C. Complete a comprehensive substance use assessment; Appropriateness of teen grief group; Readiness for family therapy Rationale: The best answer here is C. A substance use assessment must be in the answer here, since it is a critical issue. In addition, given the loss of her brother and comments the girl made about her family, we would also assess the appropriateness of family therapy and a grief group. Answer A can be eliminated since there is no indication of psychiatric issues. Answers B and D do not contain a substance use assessment, and can therefore also be ruled out. (Clinical Evaluation)

A family is referred to therapy by their son's high school. The parents are first generation immigrants from China. Their son was caught smoking pot during school. The father is a surgeon and shares that he is "very ashamed of his son" and he wants to know how the therapist will help them. How should the therapist incorporate human diversity into the assessment with this client? A. Determine the frequency and volume of substance use by the son and how each family member views drug use. B. Ask the father to share his cultural beliefs about drug use and how it reflects on the family. C. Explore the consequences of the son's drug use and the school's response on the family. D. Ask each family member about their perception of substance use and explore how each person's level of acculturation influences any differences.

D. Ask each family member about their perception of substance use and explore how each person's level of acculturation influences any differences. Rationale: The best way to incorporate human diversity into the assessment in this case is by exploring how each family member's level of acculturation is influencing their perception of substance use. So D is the best answer. Answer A focuses on the substance use assessment, but does not answer the question of human diversity. Answer B is out since it focuses solely on the father. Answer C does not mention a cultural aspect directly, so it too can be eliminated. (Clinical Evaluation).

A 38-year-old medical doctor seeks treatment for depression. The client states that her whole life she had worked to become a doctor and now that she is finally practicing independently she is miserable. She shares that she recently broke up with her partner and says she's afraid she'll never meet someone who can measure up to her expectations. In the initial phase of treatment, what interventions would a Cognitive Behavioral therapist utilize in treatment? A. A Cognitive Behavioral therapist would help the client identify her problematic beliefs. B. A Cognitive Behavioral therapist would educate the client's about her problematic beliefs C. A Cognitive Behavioral therapist would identify the client's problematic beliefs. D. A Cognitive Behavioral therapist would collaboratively explore the origins of the client's problematic beliefs.

A. A Cognitive Behavioral therapist would help the client identify her problematic beliefs. Rationale: The best answer here is (A). In the initial phase of treatment, a CBT therapist would help the client identify her beliefs. Answer (B) can be tempting; however, it can be ruled out since a CBT therapist would provide education about CBT and about the way beliefs work in general, but they would not teach clients about their problematic beliefs. Answer (C) is also incorrect, since the therapist is going to help the client identify their own beliefs, as opposed to doing it for the client. Answer (D) is incorrect because CBT therapists do not explore the origins of problematic beliefs.

A therapist at a community mental health center has been working with a family from Columbia to address issues with their teenage children. At the beginning of the final session, the mother gives the therapist a gift card and expresses her appreciation to the therapist for continued support of the family. How should the therapist address the gift with the client? A. Acknowledge the expressed gratitude and inform the client that you cannot accept gift cards. B. Determine the cash value of the gift card prior to accepting it. C. Consult with a colleague who is knowledgeable about Columbian culture. D. Explain to the family that it is your agency's policy not to accept gifts from clients.

A. Acknowledge the expressed gratitude and inform the client that you cannot accept gift cards. Rationale: Gift cards are equivalent to cash and that would be an inappropriate gift to accept, so the best answer is A. You don't need to find out the value or consult with a colleague. (Law and Ethics)

During an initial session, a mother tells a therapist that her three year-old son is stubborn, self-centered, doesn't listen, and refuses to help her until she starts doing something, and then he wants to help. He tantrums when she tries to show him how to do things properly and insists on doing it his way. Which would be the most appropriate initial response to the client's concerns? A. Acknowledge and explore the mother's feelings around parenting struggles and educate the mother about child development. B. Schedule a biopsychosocial evaluation of the child and refer mom to a parenting group for parents of children with behavioral issues. C. Review behavior modification techniques and examine the mother's expectations of parenting. D. Provide play therapy for the child and joint play therapy for mother and child.

A. Acknowledge and explore the mother's feelings around parenting struggles and educate the mother about child development. Rationale: The therapist's best response would be to acknowledge and explore the mother's feelings and provide the mother with education about normal development (A) since the symptoms described appear to be signs of normal child development. If, after receiving information regarding normal development, the mother continues to express concern or identifies behavior outside the norm, it could be appropriate for the therapist to schedule an evaluation (B), review behavioral techniques (C), or provide play therapy (D). (Treatment)

A client presents to therapy with symptoms of fearfulness to leave the house, an avoidance of buses, and anxiety around other people. She adds that while she's struggled with these issues her whole adult life it has gotten worse since her mother died five months ago. Which DSM 5 diagnoses should be considered in this case? A. Agoraphobia; Generalized Anxiety Disorder; V. Uncomplicated Bereavement B. Panic Disorder; Separation Anxiety; Adjustment Disorder C. Agoraphobia; Separation Anxiety; Adjustment Disorder D. Panic Disorder; Generalized Anxiety Disorder; V. Uncomplicated Bereavement

A. Agoraphobia; Generalized Anxiety Disorder; V. Uncomplicated Bereavement Rationale: Agoraphobia is our top answer here because she is afraid to go out. It is in (A) and (C). Both (B) and (D) have Panic Disorder and there is no indication of panic attacks so those are out. (C) has Separation Anxiety, which mainly happens with children and is not indicated here. (A) also has Generalized Anxiety Disorder, which is possible with the anxiety around other people, and V. Uncomplicated Bereavement is also possible with the death of her mother. So (A) is the best answer. (Diagnosis)

A therapist is working in a small rural town and is the only provider within the area. A neighbor from a nearby farm requests that her son receive counseling services. The therapist has seen the boy on several occasions in the local store but has minimal interaction with him or his mother. How should the therapist respond to the request? A. Agree to provide therapy to her son as the existing dual relationship is minimal and does not have a negative impact on the therapist's objectivity. B. Explain it's unethical to treat the son because he is your neighbor and entering a therapeutic relationship when any dual relationship exists is considered unethical. C. Follow a community standard of care and provide the neighbor with referrals to therapists in the next closest community. D. Given the request presents a clear conflict, the therapist should consult with their supervisor to determine if providing treatment is ethical.

A. Agree to provide therapy to her son as the existing dual relationship is minimal and does not have a negative impact on the therapist's objectivity. Rationale: It would be unethical for the therapist to treat the child if they frequently interacted with the child in a nonprofessional capacity since it could be detrimental to his treatment. However, the stem reports minimal interaction and the therapist is the only choice in the area. Thus (A) is the best answer. Answer B is incorrect since it falsely states that any dual relationship would be unethical; not all dual relationships are unethical.. You could do (C), but as it is a rural area, the next town might be very far away. Answer D assumes there is a clear conflict, but that is not accurate based on the information. Plus, as a licensed clinician, seeking consultation with other colleagues would be more appropriate than seeking supervision. (Law and Ethics)

A couple brings their adult daughter, age 24, to therapy. The daughter has been living with them since she graduated from college a few years earlier. The couple reports that their daughter has been experiencing episodes of decompensation. In formatting an initial treatment plan, what would be the initial goals of treatment in this case? A. An improvement in symptoms; increased ability to adapt to stress; ability to engage in everyday activities. B. An improvement in symptoms; maintenance of social relationships; medication compliance. C. Ability to articulate fears; willingness to leave the home; openness to trying new things. D. Ability to articulate fears; reduction in cognitive distortions; increased socialization.

A. An improvement in symptoms; increased ability to adapt to stress; ability to engage in everyday activities. Rationale: Episodes of decompensation include: a temporary worsening of symptoms, a loss of the ability to adapt to normal changes and stress, and difficulty with normal activities of life. In short, an episode of decompensation refers to a time when you experience an increase in symptoms and a loss of function. With this definition in mind the best answer is (A). Even without knowing the definition, (B) would be knocked out because of the medication compliance; we don't know if she's on medication. (C) and (D) are out because there's nothing about her being afraid, so "Ability to articulate fears" doesn't make sense. (Case Conceptualization and Planning)

A lesbian couple seeks pre-marital counseling. The women, one a 36-year-old Black high school teacher and the other a 34-year-old White graphic design artist, have been dating for three years and living together for the past year. When the therapist inquires about their reasons for seeking therapy, the teacher shares in a frustrated tone that their only issue is that the other woman smokes marijuana daily and refuses to quit despite promising to do so. In response to this, the other woman groans and says, "I don't understand why this is such an issue for you." How should the therapist further evaluate the couple's presenting issue? ? A. Ask the woman who uses marijuana to tell the therapist about the role marijuana plays in her life. B. Explore with the woman who uses marijuana when the problem with marijuana started and her views on how it has impacted their relationship. C. Further explore with the teacher in what ways the marijuana is a problem. D. Explore the teacher's past use of marijuana and how this affects her perception of the problem.

A. Ask the woman who uses marijuana to tell the therapist about the role marijuana plays in her life. Rationale: The best answer is (A). Answers (B) and (C) both focus on the marijuana use as a "problem" instead of exploring it from a neutral stance. One client does not see it as a problem; so using that wording can be harmful to the therapeutic relationship. This would make the therapist seem to be siding with the teacher. Answer (D) could be done at some point but doesn't address the main presenting problem. Answer (A) is a technique from motivational interviewing that explores the role the substance plays in the person's life instead of pushing them to see it as a problem. (Clinical Evaluation)

A young boy is brought to therapy by his parents. The 6-year-old boy has had several surgeries to correct a birth defect. The parents report the child is traumatized by his hospital experiences and expresses fear of going to the hospital or the doctor. Which of the following goals would a psychodynamic therapist establish for the middle phase of treatment? A. Child can recall the traumatic event without becoming overwhelmed with negative emotions. B. Child is able to go to the doctor with the support of the therapist and parents. C. Child is able to display a full range of emotions without experiencing a loss of control. D. Child is able to go to the doctor's office without exhibiting any resistance.

A. Child can recall the traumatic event without becoming overwhelmed with negative emotions. Rationale: (A) is the correct answer—that as the therapist works with the client there is a reduction in affect when recalling the traumatic event. (B) is out because typically the therapist would not accompany the child to the doctor. (C) is a goal for the end of treatment and this answer does not address the hospital/doctor issue that the child is working on. (D) is not a realistic goal, since it might be healthy for the child to exhibit some resistance in these circumstances. (Case Conceptualization and Planning)

An unemployed, pregnant, single woman who was recently hospitalized for observation after a car accident is referred to a community mental health clinic. She tells the therapist that she has not had any prenatal care, has not changed her diet, which she describes as basically "fast food," and doesn't want to exercise because it is "tiring." She also states that she enjoys going out with friends and drinking alcohol, but only on the weekends. What issues should the therapist focus on during the assessment phase of treatment? A. Client's feelings about being pregnant; determining client's intentions for therapy; understanding client's perception of herself and her self-worth. B. Providing referrals to community services; assessing for risk of suicidality; understanding client's perception of herself and her self-worth. C. Exploring the client's plans for her baby following delivery; Client's feelings about being pregnant; providing referrals to community services D. A need to make a report to Child Protection Services; assessing for risk of suicidality; exploring the client's plans for her baby following delivery

A. Client's feelings about being pregnant; determining client's intentions for therapy; understanding client's perception of herself and her self-worth. Rationale: As therapists, we do not make CPS reports regarding fetal abuse/neglect, thus (D) is out. Answers B and C both include providing referrals, which is something we might do in future treatments, but it's not clear at this time what services would be appropriate or if they are necessary. Plus, in answer B, the inclusion of assessing for risk of suicide is not indicated, ruling this answer out. Answer C is also a weak answer because exploring the mother's future plans for the baby might also be what we do in future treatment, but not indicated at this time. At this point, meeting the client where she is and understanding her presenting issues and goals for therapy will be the priority (A). Her lack of self care might be pointing to a low view of self, and/or to her potentially feeling resentful of being pregnant, so we would want to inquire about these issues. Assessing her motivation given her lack of desire to care for herself is also a good step to take, to make sure that she actually wants the help she is being offered. (Clinical Evaluation)

The parents of a 15-year-old boy consult with a therapist regarding their child's shift in behavior since entering high school. During the past year his grades have dropped dramatically, and he was recently placed on juvenile probation after being caught attempting to steal alcohol from a grocery store. He was adopted when he was two years old after he had been in several foster homes. Before this last year, his behavior and development had progressed normally. How should the therapist further evaluate the chief complaint? A. Conduct a comprehensive assessment of the entire family.. B. Refer the child for a medical evaluation. C. Assess the parents' disciplinary style and its congruence with the son's emotional needs. D. Assess the son for potential abuse or neglect due to marked change in behavior.

A. Conduct a comprehensive assessment of the entire family. Rationale: The strongest answer is A. The severity of the boy's behaviors warrants further assessment and if the option to engage with the entire family is available, we would pursue that first. Answer B might be done, but after a comprehensive assessment is completed by the therapist. Answer C is incorrect since some of the behaviors described in the vignette are extreme and do not reflect normal adolescent development. We do not see information in the vignette that supports assessing further for child abuse/neglect. (Clinical Evaluation)

A 45-year-old gay man seeks therapy because of a long history of depression. He states that "in most areas of my life things are fine, but I have never had an emotional relationship that felt intimate." He is estranged from his family, most of whom live in a different state and has few friends. He also reports being distanced from the gay community because he is not political and "after all, it is no different from being straight, except I have sex with guys." Which of the following actions should the therapist take to clinically evaluate this client? A. Conduct a comprehensive assessment to develop a treatment plan and determine need for referrals. B. Explore the client's estrangement from family, disconnection from the gay community and provide referrals C. Conduct a comprehensive assessment and refer to a psychiatrist for a medication evaluation to address depression. D. Normalize client's feelings and provide referrals for medication evaluation and a support group.

A. Conduct a comprehensive assessment to develop a treatment plan and determine need for referrals. Rationale: A comprehensive assessment (A) is the next step in this situation, before any kinds of explorations, referrals, or suggestions would be made. The therapist needs to have a comprehensive understanding of the presenting complaint prior to making treatment interventions. Answer (B) would be pieces of the comprehensive assessment, making A a better answer. Both (C) and (D) might happen after the assessment is complete, but we do not have enough information to warrant these referrals with the information currently provided in the vignette. (Clinical Evaluation)

A therapist in private practice receives a phone call from a potential client who states they were impressed by the therapist's website. The caller's presenting issue is a panic disorder, which falls outside of the therapist's scope of competence. The therapist declines to initiate treatment and provides the client with appropriate referrals. Shortly after the phone call, the therapist begins to notice concerning activities including odd messages on the therapist's email and cell phone, the client befriending mutual friends through Facebook, and a note left on the therapist's car from the client asking the therapist to reconsider their decision. As a result of the client's actions, the therapist is feeling uncomfortable and fearful. The therapist should: A. Document every stalking incident and consult with colleagues, as well as law enforcement. B. Consult with colleagues, as well as professional organizations and initiate the client's hospitalization. C. Document every stalking incident and provide the client with psychoeducation regarding appropriate boundaries. D. Consult with colleagues, as well as professional organizations and assess the client's appropriateness for hospitalization.

A. Document every stalking incident and consult with colleagues, as well as law enforcement. Rationale: The correct answer is A. In this scenario, the therapist is being threatened by an individual who is stalking her. The therapist did not initiate treatment with this person, so answers that include assessment or clinical interventions can be eliminated. Therefore, we can eliminate answers C and D. Answer B can also be eliminated because consultation with professional organizations would not be indicated and it's not clear the client requires hospitalization. Answer A allows the therapist to take protective measures by seeking consultation and contacting law enforcement. (Crisis Management)

A 52-year-old female client is in therapy with a male therapist for help with her relationships with her grown sons. The therapist perceives that a good working relationship has been established, but in the eighth session, the client announces that she wants a new therapist. She states that the therapist is too young and inexperienced to help with her particular problem. The therapist should: A. Encourage the client to discuss her feelings about their relationship. B. Review his professional experience to reassure the client. C. Help the client to differentiate the therapist from her grown sons. D. Validate the client's feelings and offer a list of older therapists.

A. Encourage the client to discuss her feelings about their relationship. Rationales: The most appropriate intervention to do when the client expresses resistance or a sudden desire to leave therapy is to discuss the situation directly with the client and clarify the client's feelings and concerns (A). Reassuring the client of the therapist's experience (B), referring the client out (D), or differentiating between the therapist and client's sons (C) are all "fix-it" interventions that will miss the opportunity to deepen the client's understanding of self in working with her transference. (Ethics)

At a low fee clinic, therapists receive new clients based on availability. A new client who identifies as gay is assigned to a younger male therapist whom the client assumes is heterosexual. The client, who has an extensive history of receiving mental health counseling adamantly insists on working with a therapist who also identifies as gay. Which actions should the therapist take to address the client's request? A. Explore the client's reasons for this request and determine how to proceed. B. Validate client's concerns and provide referral to an openly gay clinician, if one is available. C. Explain to the client that while the therapist understands the client's request, the therapist's sexual orientation will not affect their work together. D. Explore the client's concerns, disclose the therapist's sexual orientation, and discuss how the therapist and client can most effectively work together.

A. Explore the client's reasons for this request and determine how to proceed. Rationale: The therapist must first understand why the request was made by the client (A) in order to determine the appropriate course of action. A referral may be made (B) and the therapist could choose to disclose their own sexuality answer D, but only after understanding the client's reason for the request. We do not even know the therapist's sexual orientation based on the vignette. Answer C, explaining that the therapist's sexual orientation is irrelevant would inappropriately dismiss the client's concern. (Clinical Evaluation)

A therapist meets with a married couple for an initial intake session. The woman shares that she was born with ambiguous genitalia and as an infant her parents opted for her to have surgery to, "avoid any confusion." She had not told her husband until recently when her obstetrician informed her that due to her anatomy, she is not able to conceive. The wife shares that she feels ashamed and the husband says he feels angry and betrayed that she did not tell him this sooner. How should the therapist assess this couple? A. Explore the husband's feelings of betrayal and anger and explore the wife's reasons for not sharing this information earlier in the relationship. B. Explore each partner's understanding of intersex dynamics and the meaning each makes of it. C. Refer the couple to a fertility doctor to explore alternative options for pregnancy. D. Assess the wife's history as an intersex youth and adult and the husband's sexual identity.

A. Explore the husband's feelings of betrayal and anger and explore the wife's reasons for not sharing this information earlier in the relationship. Rationale: The best answer available is A. This is a subject you may know a lot or possibly little about, which can sometimes happen on the exam. However, you always want to consider what you do know. We know that answer C jumps ahead and ignores the presenting issues, allowing it to be eliminated. Answer D similarly ignores what has been stated in the room regarding the dynamic currently playing out in the room. This leaves us with answers A and B, which are both appropriate options. Answer A more directly addresses the issues presented by exploring the husbands expressed emotions and tapping into the wife's feelings of shame. Answer B is not as strongly connected to what is noted in the vignette, making it a weaker option(Clinical Evaluation)

A 32-year-old woman seeks therapy for depression. In the initial session she discloses that her girlfriend of two years is physically abusive and controls all their money. The patient says she knows that her depression is part of the problem and she wants to make it work with her girlfriend. In addition to discussing a safety plan with the client, which of the following interventions should the therapist consider initially when evaluating this client? A. Find out what the client means by "depression"; Provide referrals for domestic violence resources; Identify coping skills. B. Refer to couples therapy; Explore how the client has coped with girlfriend; Identify social support. C. Explore client's ambivalence about leaving; Encourage client to join a domestic violence support group; Refer to a psychiatrist. D. Find out what the client means by "depression"; Refer client to an MD; Identify social support.

A. Find out what the client means by "depression"; Provide referrals for domestic violence resources; Identify coping skills. Rationale: The correct answer is (A) because it adequately addresses all points presented in the stem. When working with domestic violence, we would create a safety plan, refer the client to DV resources, and identify and subsequently strengthen coping skills. This client mentions depression, thus it is our obligation to explore what this means, as it can have a different meaning for the client when compared to the clinical use of the term. Answer (B) is incorrect because it is clinically contraindicated to start by engaging in therapy with couples when domestic violence is present. Answer (C) is incorrect because the client did not express ambivalence, we do not have enough information for a psychiatric referral, and as therapists we provide DV resources, not encourage clients to take advantage of specific ones. Answer (D) is incorrect because it is unclear at this point whether the client needs medical attention, and answer (A) better accounts for all necessary steps. (Clinical evaluation)

A therapist sees a college-aged client who was raised in a family with a physically and emotionally abusive alcoholic mother. The client tells the therapist that she and her siblings regularly avoid getting angry or upset around their mother, because they are afraid of how she will react. The client also says she tries to keep her mother happy and that she feels this is the only way to do so. Which of the following short-term goals should the therapist consider in this case? A. Help the client understand characteristics of codependency in order to help free her from a burdensome relational dynamic with her mother. B. Assist client in expressing her feelings to her mother in an attempt to reinstate a parent - child hierarchy. C. Help the client access and express her anger since doing so can be cathartic for the client. D. Prepare the client for family therapy in order to facilitate an honest conversation between all family members where the children can express the impact of mother's behavior on them.

A. Help the client understand characteristics of codependency in order to help free her from a burdensome relational dynamic with her mother. Rationale: The stem describes a codependent situation, so the therapist needs to first help the client understand this behavior (A). Assisting the client in expressing her feelings (B) or helping the client access and express her anger (C) address only one element of the broader issue. Family therapy (D) is not a realistic option at this point. (Case Conceptualization and Planning)

44-year-old Angela comes to therapy with her 47-year-old husband, Ron, and their 14-year-old son. The wife has a bruise on her cheekbone and further questioning reveals the presence of domestic violence in the family. The son comments, "My parents have been fighting ever since I was little. They hit each other but never hurt me or anything." As the session progresses, Ron becomes more and more agitated and begins to yell at Angela while she is describing her view of the situation. The best course of action clinically would be: A. Interrupt the current discussion; conduct separate individual assessments with each family member; refer the parents to individual therapists. B. Get up and open the door of the office; refer the wife to an anger management group; refer the husband to a support group for batterers. C. Interrupt current discussion; refer the son to a support group for victims of DV; treat the couple conjointly. D. Get up and open the door of the office; refer parents to individual therapists; make a child protective services report.

A. Interrupt the current discussion; conduct separate individual assessments with each family member; refer the parents to individual therapists. Rationale: When there is a history of DV and the situation becomes heated in the session, the first thing a therapist can do to clinically manage the situation is to call for a timeout to halt the current discussion. Since this is still in the assessment stage, the presence of DV calls for separating family members and conducting individual assessment. Couples therapy is not recommended with couples that experience DV. Thus, answer (A) contains all of the correct options. There is not enough information to suspect child abuse, thus (D) is wrong. A child witnessing DV is not enough to make a CPS report. The stem needs to indicate that the child is suffering as a result of witnessing DV. For instance, they are now depressed or suicidal. Treating the couple conjointly is not recommended, thus (C) is out. Opening the door would only be necessary if the time out technique did not work. Thus (B) is out as well. (Crisis Management)

A therapist is working with a senior who recently moved into an assisted living facility after a lifetime of living in one house. Before the move, the client was outgoing but is now reclusive and refusing to attend meals or other activities. In the Bowen-Extended Family Systems therapy, the appropriate sequence of treatment would be: A. Normalize the client's feelings and allow the resident time for grieving; teach "I" statements; map out the family tree as part of the life review. B. Offer to attend events together with the client; gather family information; request a psychiatric evaluation. C. Offer to attend events together with the client; normalize the client's feelings and allow the resident time for grieving; suggest bibliotherapy. D. Normalize the client's feelings and allow the resident time for grieving; find another facility to transfer the resident; teach the client the difference between thoughts and feelings.

A. Normalize the client's feelings and allow the resident time for grieving; teach "I" statements; map out the family tree as part of the life review. Rationale: In Bowenian therapy, normalizing a client's feelings, teaching "I" statements and mapping out the family (genogram) would be the appropriate steps for a therapist to take (A). Answers (B) and (C) include the therapist attending events with the client, which would be considered boundary violations, ruling out each of these answers. Answer (D) includes transferring the client to a different residence, which is not indicated, and teaching the difference between thoughts and feelings is a cognitive approach. (Treatment)

A new client contacts the therapist to inquire about possible treatment. During the initial phone consultation the client asks whether it is OK to have questions for the therapist. The two schedule an initial session. The client comes in with a list of questions written out on a piece of paper. Since the first session was focused on history gathering there was not enough time to go over the client's questions. The therapist offers to schedule another session. The client reluctantly states: "I hope there is more time during the next session to go through my questions and answers." What is the provisional diagnosis the therapist should consider in this case: A. Obsessive Compulsive Personality Disorder. B. Borderline Personality Disorder. C. Oppositional Defiant Disorder. D. Generalized Anxiety Disorder.

A. Obsessive Compulsive Personality Disorder. Rationale: The correct answer is A. The strongest answer in this question can be found most easily through the process of elimination. We can easily rule out oppositional defiant disorder since there is nothing in the vignette to indicate the client has difficulties with authority. Answer D can also be ruled out since we do not see any signs of anxiety or excessive worrying. Answer B could be tempting, but we do not see any signs of impulsivity or instability in relational dynamics. This leaves us with answer A, obsessive compulsive personality disorder, which is supported by the client's orderliness with the long list of questions and desire to have some control in dictating the course of the session. (Diagnosis)

A therapist in a school setting is meeting with a 16-year old student who identifies as a female. She complains that at least one week a month, usually before her period, she feels irritable, tearful, and completely plagued with self-deprecating thoughts. She also has no interest in hanging out with friends or going to school. She just wants to stay in bed and sleep all day because she feels lethargic and bloated on those days. What DSM diagnosis should the therapist consider in this case? A. Premenstrual Dysphoric Disorder B. Depressive Disorder not otherwise specified. C. Dysthymic Disorder D. Major Depressive Disorder

A. Premenstrual Dysphoric Disorder Rationale: The symptoms described in this vignette meet the criteria for Premenstrual Dysphoric Disorder, which includes symptoms that often mimic depression, but occur in the week before the onset of menstruation. The symptoms also start to improve within a few days after the start of prior to menses. Since the symptoms align with the client's menstruation cycle, we can rule out all other depressive diagnoses.

A therapist is working with an elderly man who lives in assisted living housing. When the therapist first approached the client he was resistant and unwilling to talk. "You are so much younger than me. What is it that you have to offer me?" he asks the therapist. During the third session, when the therapist asks the client to look back at his life and share any highlights that come to mind, he suddenly perks up and willingly shares a number of stories. In working with the elderly, what is the objective of employing a Life Review as an intervention? A. Provide clients with an opportunity for closure; communicate a sense of respect; and help clients recognize the sense of meaning developed over the course of their lives. B. Help clients recognize the meaning developed over the course of their lives; assess clients for possible signs of dementia; provide clients with an opportunity for closure. C. Develop appropriate goals for treatment; provide clients with an opportunity for closure; communicate a sense of respect D. Communicate a sense of respect; review client's movement through various developmental stages; help clients recognize the meaning developed over the course of their lives

A. Provide clients with an opportunity for closure; communicate a sense of respect; and help clients recognize the sense of meaning developed over the course of their lives. Rationale: A Life Review is a supportive tool for client's mental health. It is an opportunity to communicate a sense of respect and reverence to the clients regarding their past accomplishments and sorrows. It provides clients with a possibility to gain closure. Thus (A) is the correct answer. A Life Review is not used as an assessment tool, thus (B) and (D) are out. Goals for treatment (C) might come out of the Life Review, but that is not the main function of such an intervention. (Treatment)

A therapist is providing crisis counseling to second graders at a local elementary school. A classmate was recently hit by a car and killed on the way to school. The students report feeling scared to come to school and anxious that something bad might happen to them. When meeting with students, the therapist should first: A. Reassure students that their feelings are normal and meet with students individually to assess severity of symptoms. B. Develop a student support group to provide a safe space for classmates to process their grief and to normalize their feelings of fear. C. Provide students and their parents with education on how to cope with Acute Stress Disorder and grief. D. Develop a list of referrals for grief counseling and encourage parents to link students with mental health treatment.

A. Reassure students that their feelings are normal and meet with students individually to assess severity of symptoms. Rationale: The correct answer is (A) because the first thing to do is to let the students know that feeling scared is completely normal and to be expected. This way the students do not have to worry that something is wrong with them. We would also want to assess the varying needs of students, as the type of support needed will vary. It is only after completing this step that the therapist might choose to follow through on options (B), (C), or (D). It's important to note that students' needs will vary, so the different options available in answers (B-D) could be appropriate for one subset of students, but not all. (Crisis Management)

A therapist working in the psychiatric hospital is asked to complete an intake with a newly admitted patient. The 54-year-old woman was admitted to the hospital in the middle of the night under WIC Section 5150 after law enforcement received an anonymous call requesting a welfare check . When the therapist meets with the woman, she presents as confused and unable to answer the therapist's questions. How should the therapist evaluate this client? A. Seek permission from the client to contact relatives; continue working with the client to gather information; collaborate with the medical team. B. Refer the client for psychological testing; collaborate with the medical team; wait for the woman to stabilize before completing the intake. C. Refer the client for psychological testing; interview the officers who invoked the 5150; gather as much information from the client as possible. D. Seek permission from the client to contact relatives; interview the officers who invoked the 5150; wait for the woman to stabilize before completing the report.

A. Seek permission from the client to contact relatives; continue working with the client to gather information; collaborate with the medical team. Rationale: People are admitted to the hospital for serious medical reasons. If they are unable to provide intake information, the next best source of information could be their family or significant others (A). Seeking permission to make these contacts would be an appropriate and ethical thing to do. The client might not be lucid enough to sign a release allowing us to talk with their relatives, but that does not mean that we wouldn't try. In addition, we would continue our efforts to work with the client and it would be important to collaborate with the medical team to understand any underlying biological concerns. Answer B is not as strong because it's premature to recommend psychological testing and it's not clear how long it would take for the client to stabilize. Answer C is incorrect because we would focus our attention on the medical needs rather than the officers or psychological assessment. Answer D has similar components to B and C also making it incorrect. (Clinical Evaluation)

Eric, a 38-year-old Caucasian, and Naomi, a 31-year-old Asian-American, are referred to therapy by Naomi's minister. In order to keep his job, Eric recently completed a 30-day drug and alcohol residential treatment program. Naomi states that while Eric was away, she felt safe for the first time in years. She was able to attend church and see her friends. Her situation has changed since Eric returned home. She appears nervous in session and complains of insomnia, nightmares, and difficulty concentrating at work. She fears that they both might lose their jobs. Eric states, "I don't know why I'm here. I don't need any more therapy. I'm done drinking! What's the problem? Naomi should just see you alone." Using Cognitive-Behavioral Therapy, which of the following interventions would achieve Naomi's goal of increasing her self-care skills based on the case provided in the vignette? A. Teach Naomi relaxation techniques to use when she awakens from a nightmare; test Naomi's assumptions that she might lose her job B. Enable Naomi to identify the origins of her self-defeating thoughts; teach Naomi to extinguish her fears about seeing her friends C. Enable Naomi to challenge her isolation as a result of not seeing friends; teach Naomi thought-stopping techniques regarding Eric's drinking D. Encourage Naomi to monitor her thoughts when she is feeling nervous; enable Naomi to identify the origins of her self-defeating thoughts

A. Teach Naomi relaxation techniques to use when she awakens from a nightmare; test Naomi's assumptions that she might lose her job Rationale: The question is specifically focusing on Naomi's goal of self-care, as well as CBT therapy. Both parts in (A) address that goal and are appropriate CBT interventions. Answer (B) talks about the origins of her thoughts and CBT does not pay attention to the origins. Plus the therapist will not teach the client how to extinguish her fears, as that is not an existing intervention. Answer (C) is out since CBT therapists would not be enabling the client to challenge isolation, rather thoughts that are resulting in isolation. And teaching the client ways to stop thinking about her husband's drinking is not an appropriate intervention. If he drinks, she should be thinking about it. Answer (D) talks about the origins of her thoughts, but CBT does not pay attention to the origins. (Treatment)

A therapist meets with the parents of an 18-year-old boy. The father complains that their son is too rebellious and he wants him to move out of the house immediately. The mother disagrees with her husband, defending her son by claiming he is just being a normal teen and complains the husband is too strict. The husband tells the therapist he is offended when his son talks back, leaves the house a mess, and ignores any directions given. Which of the following actions would a Systems therapist use to address the conflict presented in this case? A. Teach the parents how to discuss and address problems between themselves and their son. B. Teach the entire family how to discuss and address problems to avoid maintaining homeostasis. C. Assist the entire family to better understand how circular communication exacerbates the son's negative behaviors and increase positive interactions. D. Utilize role play to increase the parent's level of empathy toward each other.

A. Teach the parents how to discuss and address problems between themselves and their son. Rationale: This question highlights the importance of identifying the treatment unit. While Systems focuses on the entire family, the therapist's actions would still need to be directed toward the parents since they are the ones in the room. Teaching the parents how to communicate and problem solve between themselves and also their son (A) would be the main goal of a systems family therapist. Plus, they are complaining of having problems with their son, so teaching them how to resolve them would address their concerns. Answer (B) assumes the son is part of the treatment unit, which is not accurate, so we would rule it out. In addition, while homeostasis is a term used with systems therapy, the stem does not talk about how family members are maintaining homeostasis- it only talks about the problem they face. The exam will sometimes include the keywords linked to a theory, but inappropriately so, to ensure you are understanding the question and not automatically choosing an answer solely because it includes key terminology of the theory. Answer C is also addressing the wrong treatment unit. Answer D is not as strong as answer A because it is only focused on the couple while disregarding the role of the son in the presenting issue. (Treatment)

A therapist sees a client who is concerned about excessive Internet use. The client worries about not being able to control online activity and feels it is harming interpersonal relationships outside of the Internet. In order to assess the impact of the client's Internet use, the therapist should determine: A. The degree to which the client's life is affected. B. The duration of time spent online each day. C. How many online relationships the client has. D. What kind of websites the client is visiting.

A. The degree to which the client's life is affected. Rationale: The therapist would need a complete picture of the client's presenting problem and its impact on the client's life, which must include an evaluation of how Internet use affects the client's life (A). Options (B), (C), or (D) would be done after the therapist has an understanding of the extent of the problem. (Clinical Evaluation)

A 6-year-old girl is brought to therapy by her parents. The parents have noticed that their daughter is much quieter and less interactive at home and with family. The girl's teacher has also notified the parents to tell them she is concerned about their daughter because in the last 6 months, she seems much more withdrawn and isolated at school. The parents feel that something may have happened to their little girl that they don't know about and that she is not telling them. After completing the assessment, what would the initial goals be in working psychodynamically with this child? A. To build rapport and a safe environment for the child; for the child to express feelings through projective techniques such as art, play, or sand play. B. For the child and parents to verbalize an accurate understanding of PTSD; to develop coping strategies when she is feeling isolated. C. For the parent and child to develop the capacity to discuss the nature of the problem; to increase positive interaction between child and peers. D. To obtain a history of the child's early childhood behavior; to create a holding environment for discussing past events with the child.

A. To build rapport and a safe environment for the child; for the child to express feelings through projective techniques such as art, play, or sand play. Rationale: Answer A is the correct answer for initial goals in working with the child psychodynamically. (B) is out because there is not enough in the stem to assume there is PTSD. The goals identified in answer C may come later but first the therapist needs to build rapport and understanding. The question stem notes an assessment has been completed, so answer D is incorrect because obtaining history is part of the assessment. (Case Conceptualization and Planning)

A structural family therapist has been working with a family of six for several weeks. In the most recent session, Stephanie, the second to youngest member of the family tells the therapist about the "blow out" that happened the night before. Apparently two of the siblings got into a fight and the mom was trying to help them resolve it while bathing the youngest child and helping Stephanie with her homework. In the meantime, the dad was downstairs watching the football game. The therapist asked the family to replay the scenario from last night and encouraged the dad to participate in the management of the fight more actively. What was the purpose behind such intervention? A. To help the family modify the problematic interaction and see it through to a satisfying completion. B. To observe circular causality and communication patterns. C. To increase "here-and-now" awareness. D. To challenge family's definition of the problem.

A. To help the family modify the problematic interaction and see it through to a satisfying completion. Rationale: The intervention described in the stem is re-enactment. It is frequently used by structural family therapists in order to help the family change problematic behaviors and experience a different, more effective outcome (A). Answer (B) describes a Systems family therapy intervention. Answer (C) describes a Gestalt intervention. Answer (D) does not fit because, even though challenging the definition of the problem is an early stage structural therapy intervention, in this case the therapist did not challenge the family's definition of the problem but asked them to re-enact the problem instead. (Treatment)

A therapist sees a client who was referred after they were charged for engaging in embezzlement. The client has an extensive history of gambling and shares that he is worried for his family and their future. How should the therapist assess appropriate referrals in this case? A. Utilize motivational interviewing to determine stage of change and explore client's access to legal representation B. Utilize motivational interviewing to determine stage of change and explore familial support systems C. Explore client's access to legal representation and most recent medical examination D. Explore client's expectations for therapy and assess frequency of client's engagement in gambling activities.

A. Utilize motivational interviewing to determine stage of change and explore client's access to legal representation. Rationale: The best answer here is A. Since the client was charged and is now part of the legal system, assessing his access to legal representation is going to be a high priority. In addition, accurately determining the stage of his change will help us with creating a treatment plan. The first part of answer B is good, however, the second part regarding familial support is not completely relevant to the stem, nor is it going to be prioritized over legal support. Answer C is incorrect since there are no medical issues indicated in the vignette. Answer D could be tempting, however, there is no indication in the stem that he has inappropriate expectations for therapy, thus we can rule out this answer. (Clinical Evaluation)

A 42-year-old woman seeks therapy after the birth of her second baby. She reports difficulty sleeping, feeling agitated and crying a lot. She tells the therapist that she is afraid she can't be a "good mom." The therapist assesses for risk of self-harm and the client reports that she thinks about killing herself on a daily basis. In addition to determining whether the client has a plan and the means, what else should the therapist assess? A. What has stopped her from killing herself; Any past attempts; Preparatory acts B. Her history of mental illness and current substance use; Any past attempts; Lethality of the means C. Family history of suicide and social supports; Any self-injurious actions; Availability of the means D. Safety of her children and her willingness to go on medication; Preparatory acts; Family history of suicide and social supports

A. What has stopped her from killing herself; Any past attempts; Preparatory acts Rationale: The correct answer needs to be oriented towards this woman and her current suicidal state. The first part in answer (B) includes client's history of mental illness, which is not as relevant during a suicide assessment. Answer (C) similarly talks about her family history, which is not relevant at the moment. Answer (D) addresses her children, which is not a focus of the question being asked. The best answer here is (A) since it addresses her personal history of passed suicide attempts, preparatory acts (steps taken in advance of committing suicide such as creating a will, finding caretakers for her children), as well as the resources that are currently available to her and that have been keeping her alive so far. (Clinical Evaluation)

A therapist is meeting with a 37-year-old computer programmer. The client tells the therapist that he is very content at his job and actually is in charge of a big project at his firm. "The improvement I am working on would reinvent the entire system of how we do things at the company. This is a huge responsibility and people are looking to me to develop the best way to move forward. So I have been working around the clock." During an intake interview a therapist also learns that the client has 6-7 alcoholic beverages a night. In addition, the client tells the therapist that he has been experiencing headaches and blurry vision. Which of the following referrals is most important in this case? A. Alcoholics Anonymous because the client is abusing alcohol regularly and is not indicating any level of denial. B. A general medical practitioner because the client is abusing alcohol and is experiencing physical symptoms. C. An outpatient substance abuse treatment facility because the client is abusing alcohol, but is functional at work. D. A neurologist to assess for head trauma because the physical symptoms listed are commonly linked to head trauma injuries.

B. A general medical practitioner because the client is abusing alcohol and is experiencing physical symptoms. Rationale: AA might be an appropriate referral after dealing with the physical issues (A). The client's symptoms require immediate medical attention and most likely in-patient detox so (C) is out. If the client had bizarre symptoms, such as smelling a tangerine all the time, then we would refer him to the neurologist, but he is not having those symptoms so (D) is out. Thus, the correct answer is (B), which will include sending the client to an emergency room if he does not have a regular doctor. The main thing is for him to see a doctor due to the symptoms of headaches and blurry vision. (Clinical Evaluation)

During a session, a woman complains about ongoing conflict with her husband regarding their finances. She reports that two days ago, after a heated argument about a credit card bill, her husband slapped her. She informs the therapist that this was not the first time a verbal argument turned physical. She is feeling more optimistic because he bought her flowers and offered to start couples therapy with her. The client informs the therapist she would like the husband to join in future sessions. How should the therapist initially proceed to address the physical violence presented in the case? A. Acknowledge client's hope for change; encourage client to develop a safety plan; inform client that couple's counseling is contraindicated in cases of domestic violence. B. Acknowledge client's hope for change; provide the client with psychoeducation regarding the cycle of violence; develop a safety plan with client. C. Provide the client with psychoeducation regarding the cycle of violence; refer husband to attend anger management group; develop safety plan with client. D. Provide client with referral to domestic violence support group; provide psychoeducation regarding the cycle of violence; inform client that couple's counseling is contraindicated in cases of domestic violence.

B. Acknowledge client's hope for change; provide the client with psychoeducation regarding the cycle of violence; develop a safety plan with client. Rationale: Answer (B) is the strongest answer available. We want to start where the client is, so acknowledging the client's hope for change is a good place to start. We would also want the client to be aware of the cycle of violence since the vignette clearly indicates they are in the honeymoon stage. In addition, developing a safety plan with our client is essential to address potential future risk. Answer A highlights the importance of paying close attention to the wording--we would want to collaboratively develop a safety plan with the client, not encourage the client to develop one. This wording makes answer A incorrect. Answer C is not as strong as (B) because our priority is focusing on our client, not resources for the husband. Answer (D) is not as strong as (B) because the referral to a DV support group is not as important as safety planning. (Crisis Management)

A 13-year-old boy who has been in foster care for three months is referred to therapy by the court. The referring social worker reports that he does not follow direction at his foster home, often leaves home for extended periods of time, and has come home smelling like marijuana and cigarettes. He is arguing with teachers and skipping school on a regular basis. What would be the provisional diagnoses for this client? A. Oppositional Defiant Disorder, Conduct Disorder, Intermittent Explosive Disorder B. Adjustment Disorder, Oppositional Defiant Disorder, Substance Use Disorder C. Major Depressive Disorder, Conduct Disorder, Substance Use Disorder D. Oppositional Defiant Disorder, Reactive Attachment Disorder, Substance Use Disorder

B. Adjustment Disorder, Oppositional Defiant Disorder, Substance Use Disorder Rationale: The correct answer is (B) as it is possible the behavior could be a result of recent home displacement; he is also showing signs of ODD and substance use. Every other answer includes conduct disorder, which is not indicated by the information included in the stem. (D) is out because there is not enough for a diagnosis of RAD. (Diagnosis)

A therapist is called to a hospital emergency room to assist with a woman who was brought in with her deceased baby. The baby has just died of sudden infant death syndrome. The mother, who has no history of mental illness, is refusing to let go of the baby and insists that the baby is still alive. What actions should the therapist take in this case? A. Ask the doctors to sedate the woman while the baby is removed. B. Allow the woman to let go of the baby in her own time. C. Admit the woman to the psychiatric unit. D. Involve family members in helping to reorient the woman.

B. Allow the woman to let go of the baby in her own time. Rationale: The client is engaging in denial, which is the first stage of grieving. Allowing the mother to let go of the baby in her own time (B) would allow the mother to engage in this stage of grief. Asking the doctor to sedate the woman (A) or admitting the woman to the psychiatric ward (C) would be overreactions based on the information in the stem. The therapist could choose to involve family members (D) but only at a later stage. (Treatment)

Ana, a Mexican-American single mother of two young children is referred to therapy by her minister. Ana shares that she has been very overwhelmed lately. Her parents were visiting for three months but recently went back to Mexico. She says, "They really helped me with the kids. Childcare is so expensive and my kids are in different schools now. My boss at work is getting irritated that I have recently been arriving late to work . I am worried I am going to lose my job." She adds that she's been having difficulty sleeping and bad headaches. What human diversity issues should the therapist consider when developing an initial treatment plan in this case? A. Ana's health concerns; Her low socioeconomic status; Being a single mother. B. Ana's spiritual beliefs; Her experience as a Mexican-American; Being a single mother. C. Ana's spiritual beliefs; Her low socioeconomic status; Being a single mother. D. Ana's sexual orientation; Being a single mother; Her experience as a Mexican-American.

B. Ana's spiritual beliefs; Her experience as a Mexican-American; Being a single mother. Rationale: In answering this question we need to think about this client's unique life circumstances that are shaping her presenting issues. In the stem we see that she is a Mexican-American, is a single mother, and she was referred by her minister. All three of these things set her aside from other people with similar presenting issues, thus (B) is the best answer. Answer (A) is out since lots of people share similar health concerns and that is not a human diversity issue. Plus, we would be making an assumption if we were to state that her low socioeconomic status is at play, since the stem doesn't explicitly state that she has a low socioeconomic status. Answer (C) is also out due to the assumptions regarding her low socioeconomic status. And (D) is wrong because we do not know her sexual orientation. (Case Conceptualization and Planning)

A therapist meets with a 19-year-old woman who is in her first year of college. She expresses feelings of depression and loneliness. She explains that she had a very close group of friends in high school, played on the soccer team, and received straight A's. Since starting at school, she has met several people but has not developed many good friendships and her grades are low. She says, "If things don't get better, I'm not sure what I'll do." Which of the following actions should the therapist take to assess this client? A. Explore client's goals and identify possible obstacles; identify healthy coping mechanisms; determine support networks available to the client. B. Ask the client if she has thoughts of self-injury; identify healthy coping mechanisms; determine how long symptoms have been present. C. Explore the client's goals and identify possible obstacles; refer to a medical doctor to assess biological factors; determine how long symptoms have been present. D. Identify healthy coping mechanisms; determine support networks available to the client; identify client's internal strengths.

B. Ask the client if she has thoughts of self-injury; identify healthy coping mechanisms; determine how long symptoms have been present. Rationale: The most important factor to consider in this case is the potential for self-harm. The only answer that assesses for this is (B). The therapist may choose to use the interventions identified in answers (A), (C), and (D) but the priority is to focus on immediate safety. (Clinical Evaluation)

Filip, a 27-year old real estate agent, seeks treatment for persistent anxiety. In the initial interview he tells the therapist that he worries about everything. "It is like a constant tape playing in my head. I worry that the sale is not going to go through. I worry that I will not have enough money to pay my rent. I worry that I will never find a mate. And then I worry about missing being single if I do find someone." He has agreed to complete thought records in between sessions with the therapist. In the next two sessions the client says, "I didn't get it done." How should a Cognitive Behavioral therapist respond to the client's lack of follow through? A. Interpret behavior as a sign of resistance and explore the client's motivation for treatment. B. Ask the client what interferes with completion of homework and identify strategies to address barriers. C. Remind the client of the importance of homework and discuss the limitations of CBT without the client's completion of assignments. D. Acknowledge that homework can be burdensome and restructure the assignment to make it easier to complete.

B. Ask the client what interferes with completion of homework and identify strategies to address barriers. Rationale: Meeting the client where they are in a neutral, curious way and working to address the barriers identified is the most appropriate way to approach this situation (B). Interpreting the client's behavior as resistance is a little bit premature, so (A) is incorrect. Reminding the client of the importance of homework and limits of CBT as a consequence might come across as authoritative and even judgmental, so (C) is out. Acknowledging that homework can be burdensome and restructuring assignments (D) does not make sense since the client never said that the reason they did not complete it was because it was burdensome and we can't restructure without first understanding barriers. (Treatment)

A mother of two young daughters meets with a therapist. The woman shares that she recently remarried, and her new husband also has two young children. Her daughters live with them permanently and the husband's boys are there every other week. The woman complains that none of the kids get along and that their house is "total chaos" when they are all together. The woman shares that she is miserable with this arrangement and does not know what to do. Which of the following actions are representative of how a humanistic-existential therapist would address the woman's presentation? A. Ask the woman to elaborate on her emotional reaction when all of the children are together and the chaos ensues. B. Ask the woman to notice and describe the physical sensations she is experiencing in her body, in the moment, as she describes the familial dynamics. C. Encourage the woman to envision her blended family in the therapy room with her, and express her frustrations and what she hopes for in her home. D. Ask the woman to imagine what a perfect home would look and feel like and to describe that vision with the therapist.

B. Ask the woman to notice and describe the physical sensations she is experiencing in her body, in the moment, as she describes the familial dynamics. Rationale: A humanistic- existential therapist would keep the client in the here and now and help her feel and be present with the experiences she is describing. Thus B is the best answer. Answer A is a good, yet generic answer, not specifically pertaining to this theoretical orientation. Answer C might come after the action step described in B. And finally, answer D is associated more closely to solution focused therapy and can also be eliminated. (Treatment)

Tasha, age 63, is referred to therapy by her doctor. Tasha was recently in a car accident that resulted in her losing her left leg. In the first session she appears clean but disheveled and has a hard time maneuvering her wheelchair. She tells the therapist, "I am too old for this. I don't want to be a burden on my family. Now I can't even take care of my grandkids. What is the point of being alive when I can't really live?" How should the therapist manage the potential crisis issues in this case? A. Assess Tasha's potential for self-harm and consult with her physician regarding in-home support needs. B. Assess Tasha's potential for self-harm and collaborate with her family and medical providers. C. Initiate a "No Harm" contract and refer for occupational therapy. D. Assess Tasha's risk of self-harm and potential neglect of grandchildren.

B. Assess Tasha's potential for self-harm and collaborate with her family and medical providers. Rationale: The correct answer here is (B). Since there are a number of risk factors present in the vignette, assessing Tasha's potential for self harm is going to be a priority. Plus answer (B) focuses on all of Tasha's available support, which is important in crisis situations. Answer (A) is a strong possibility; however, it only involves Tasha's physician and leaves her family out. Engaging all of her available supports is crucial in supporting the client. Answer (C) can be ruled out since the actions it describes do not address the crisis concerns in this case. Answer (D) is also incorrect since child neglect is not indicated in the vignette. (Crisis Management)

A therapist is working with a client who has a history of substance abuse. He informs the therapist that although he has been through several treatment programs, he has continuously relapsed. He says that nothing has worked for him and he doesn't think anything can help him. The client thinks he will become just like his father, who has a long history of substance abuse. What should the therapist further evaluate when determining initial interventions? A. Ask the client about his experiences in the different treatment programs. B. Assess the client's current use and his motivation to change his behavior. C. Explore the client's willingness to involve his family in his recovery. D. Refer the client for a medical evaluation.

B. Assess the client's current use and his motivation to change his behavior. Rationale: In working with any client, it is important to begin the work with assessment. The client's level of use and motivation to change the addictive behaviors is the most important factor to consider when assessing probable effectiveness of substance abuse treatment. Answer (B) includes both of these important components. It is after such steps are complete that the therapist might decide to involve the client's family (C), refer the client for a medical evaluation (D), or ask about previous experiences (A). (Clinical Evaluation)

Tamara, a 38-year-old Baptist minister, and her 28-year-old husband, Michael, who are both African American, self-refer for couples therapy. Tamara recently gave birth to a son with Down Syndrome. Tamara states, "Michael doesn't understand my stress. I have a congregation that relies on me, but I am so worried about everything that I can't sleep and can't think clearly." She complains that Michael drinks nightly and doesn't get out of bed all day. "I can't trust him with our baby." Michael responds, "I begged you not to go through with the pregnancy when we found out the baby was defective. Don't blame me for your child care problems while you keep working 16 hours every day." Throughout the initial appointment, Tamara compulsively checks her cell phone for messages, while Michael sits rigidly with his arms crossed and stares at the floor. How would a Cognitive-Behavioral therapist incorporate the couple's religious beliefs into the treatment plan based on the case presented in the vignette? A. Enable the couple to identify the positive value of the marital dyad as embraced by their religious beliefs; encourage Tamara and Michael to explore alternative spiritual interpretations for their marital discord. B. Assist the couple in using their religious beliefs to reframe the perception of the child as "defective"; facilitate dialogue between Tamara and Michael about the impact of Tamara's ministry on their marital discord. C. Explore the thoughts that trigger Michael's desire to drink; promote dialogue on how potential divorce would be received within the religious community. D. Recommend that Tamara and Michael explore alternative spiritual interpretations for their marital discord; challenge the couple's insight into their spiritual differences regarding their baby.

B. Assist the couple in using their religious beliefs to reframe the perception of the child as "defective"; facilitate dialogue between Tamara and Michael about the impact of Tamara's ministry on their marital discord. Rationale: Answer (A) is wrong since it doesn't specifically include CBT interventions and neither part of the answer is indicated in the stem. The value of the marital dyad is not being questioned in the stem, and there is no need to explore alternative interpretations of the marital dyad, as much as there is a need to determine the couple's understanding of the marital discord. Answer (B) has all the necessary parts to it. It describes CBT interventions that are relevant to the case, and religious beliefs are incorporated. Answer (C) is out since the first part only deals with Michael and ignores the couple, and the second part brings up a divorce, which was not mentioned in the stem. The first part of answer (D) has already been discussed, and the second part is not appropriate since a CBT therapist is not going to challenge the couple's insight but will teach the couple how to challenge their own thoughts. (Case Conceptualization and Planning)

A school-based therapist is leading a small ongoing group for high school students with conduct problems. The goal of their weekly meetings is empathy development. In addition, the therapist attempts to aid the students in advancing a sense of responsibility for their actions. The group has now been meeting for 5 consecutive weeks and it is clear that one of the students is an informal group leader. He tends to intimidate the others and tries to control the discussion by glorifying his past crimes. Each week he comes in boasting about things he has done without any consequences. How should the therapist address the group dynamic? A. Remain in the observing role without getting involved and allow the group to work out the issues of power and control. B. Assume a leadership role and ask the group how they feel about this student's stories and comments. C. Maintain a neutral stance and model a trusting relationship by listening to the leader. D. Demonstrate the concept of therapeutic confrontation, by talking to the student leader about his past and current behavior.

B. Assume a leadership role and ask the group how they feel about this student's stories and comments. Rationale: A group member who takes over the leadership of a therapy group destroys the therapeutic process. The therapist can best re-assert professional leadership by providing a safe environment for the other members to express their feelings (B).(A) is not the best answer since the group has been dealing with it for 5 weeks now and they have not yet been able to confront the situation. So they need help and modeling to deal with this situation. (C) is not the best answer since it would be empowering this student in the wrong way and letting the group work through this situation. (D) is not a good answer since we need to help the group express the impact this student's behavior has had on them, instead of giving more air time to the student. He needs to be able to hear what it was like for them to have him in the group, since the goal of this group is to develop empathy and a sense of responsibility. (Treatment)

A therapist receives a phone message from a client's psychiatrist who has been providing medication monitoring for the client's depression. The psychiatrist informs the therapist that the client gave him permission to contact the therapist and he would like to discuss concerns about the client's diagnosis and treatment. What should the therapist do first? A. Call the psychiatrist to inform him that you will request the client sign a release in the next session. B. Call the psychiatrist and request a faxed copy of his release before acknowledging you see the client. C. Call the client to request permission to speak with the psychiatrist. D. Call the psychiatrist to discuss concerns regarding the client's case since the client has signed release.

B. Call the psychiatrist and request a faxed copy of his release before acknowledging you see the client. Rationale: Legally we are allowed to speak to another health care provider without a release for the purpose of treatment and diagnosis, however, it is always best practice to secure a release from the client and have it on file. For instance if the conversation with another provider moves away from those two topics and onto the client's history, as it often does, we are covered. Ethically, we would also want a release of information. Thus (B) is the best answer here. Calling the client and receiving a verbal consent would not be sufficient, thus (C) is incorrect. You could do (A); however it is time consuming. (D) is incorrect since the therapist would be breaking the client's confidentiality without having a written release in hand. (Law and Ethics)

Jill, age 45, and John, age 47, are referred by Jill's therapist for therapy to help with their blended family. The couple recently married and Jill's children, Tammy, age 13, and Jake, age 10, moved in with John and his daughter, Blair, age 5. John complains, "Tammy is such a handful. She doesn't listen to anyone. We really need help with her." Jill adds, "The transition has been hard on everyone. It doesn't help that I have been working a lot and the kids spend a lot of time with sitters." What ethical responsibilities does the therapist have based on the case provided in the vignette? A. Manage countertransference issues that may arise; Obtain a release from Jill to consult with her therapist; Maintain appropriate boundaries with the family. B. Clarify the unit of treatment; Manage issues of confidentiality if seeing members of the family alone; Discuss their expectations of treatment. C. Maintain appropriate boundaries with the family; Discuss their expectations of treatment; Obtain informed consent from the family. D. Clarify the unit of treatment; Maintain appropriate boundaries with the family; Provide a referral for parenting classes.

B. Clarify the unit of treatment; Manage issues of confidentiality if seeing members of the family alone; Discuss their expectations of treatment. Rationale: Answer (A) is ruled out since the first part is not relevant to the vignette. There is no indication in the stem that the therapist is having issues with countertransference. The second half is a legal responsibility, not an ethical one, and the third part is also not indicated in the stem. Answer (B) has all parts that are ethical in nature and that are relevant to the case. It would be important to identify the unit of treatment, to discuss their expectations, and to manage confidentiality if members of the family are seen alone. Answer (C) is out since the first half is not as relevant to the case presented, as we would need the answer to be, and the third part is legal in nature. Answer (D) has a good first half, but the second part, as discussed, is not relevant to the case. The third part only addresses the parents and the transition has been hard on the whole family. (Law and Ethics)

Shana, an overweight 31-year-old financial analyst, is referred to therapy by her EAP due to problems at work. She has been coming in late, leaving early, not completing paperwork on time, and being absent-minded in the meetings. She shares that lately she feels like she is on a rollercoaster with her moods, particularly when she gets hungry. She adds "On the bright side, I've lost 10 pounds this month without even trying." How should the therapist further assess the presenting issues in this case? A. Complete psychological testing; Provide a referral for Shana to see a psychiatrist; Explore irrational beliefs regarding body size. B. Complete biopsychosocial assessment; Provide a referral for Shana to see an MD; Assign the client to keep a journal of activities, thoughts, and feelings. C. Complete an assessment interview; Refer Shana to a career counselor; Assign the client to keep a journal of activities, thoughts, and feelings. D. Complete a biopsychosocial assessment; Provide a referral for Shana to see a nutritionist; Identify irrational beliefs regarding body size.

B. Complete biopsychosocial assessment; Provide a referral for Shana to see an MD; Assign the client to keep a journal of activities, thoughts, and feelings. Rationale: Answer (A) is incorrect since there is no indication for psych testing for this client, her symptoms of quick weight loss and hunger-related mood swings point to the need for her to see a doctor, not necessarily a psychiatrist. And there is no indication of irrational beliefs about her body size in the stem. Answer (B) contains good initial stage interventions that are also relevant to the case. A biopsychosocial assessment should be completed, she should be referred to an MD, and a journal assignment to keep track of her feelings, thoughts, and activities is a good way of helping both the therapist and the client gain awareness about what might be causing her presenting issues. Answer (C) has a good first part, but the second part is not appropriate since it is not clear that it is work stress that is the matter here. Answer (D) has us referring Shana to a nutritionist and that is a bit premature. We would first need to know that it is diet that is causing her weight loss and mood swings. (Clinical Evaluation)

57-year-old Nolan and 54-year-old Luisa seek treatment due to marital discord. Luisa complains "Nolan is always quiet when he gets home from work. It's as if we have nothing to talk about, nothing in common." Nolan utters "Ever since the kids went off to college, all Luisa does is rearrange their rooms and tries to rent them out on Airbnb. She is bored and disinterested with me." How should the therapist further evaluate the presenting problem? A. Conduct an inventory of the couple's attachment style; ask the couple what brought them together; figure out what conflict looks like between the two of them B. Conduct an inventory of the couple's attachment style; assess the impact of couple's life stage transition; ; gather information regarding their relational history C. Ask the couple to each put themselves into the shoes of their partner and describe their experience; figure out what conflict looks like between the two of them; conduct an inventory of the couple's attachment style D. Schedule individual sessions with each member of the couple; provide psychoeducation to the couple about the life stages; ask the couple what brought them together

B. Conduct an inventory of the couple's attachment style; assess the impact of couple's life stage transition; ; gather information regarding their relational history Rationale: The best answer here is (B) since it addresses the impact of the life stage and answers the question that is geared towards assessment. Answer (A) includes various issues we would want to explore, but it does not address the issue of the life stages, which makes (B) a stronger answer. The first part of answer (C) is not an assessment technique, but rather an intervention used later in therapy after the assessment is complete. The second part of answer (D) is not an intervention that does not allow for information gathering and is therefore incorrect for an assessment question (Clinical Evaluation)

A Middle Eastern family consisting of father Abir, mother Narin, mother's sister Gulnar, 12-year-old Nadina, and her little brother Arman seek family therapy. They state that recently Nadina's behavior has been a bit strange. She has become withdrawn, uninterested in things she used to love, and almost completely disengaged in family activities. "We decided to try family therapy this time, since last time, when Nadina was in therapy by herself it seemed to have made matters worse," said Abir. When the therapist turns her attention to Nadina, she appears very shy and uncomfortable talking in front of her entire family. The therapist suggests an individual assessment during the first half of the next session. During that time, Nadina tells the therapist that her previous therapist used to masturbate in front of her. "He also said that I should also play with myself in front of him in order to loosen up and know my body better. So I did that a few times and now I feel completely ashamed of myself." What action should the therapist take? A. Inform Nadina that her parents must be told about the incident immediately. B. Contact protective authorities. C. Provide the brochure from the BBS and discuss the option of reporting the previous therapist to the licensing board. D. Maintain confidentiality and deal with this as a clinical issue only.

B. Contact protective authorities. Rationale: The best answer is (B). The client is a minor and the therapist exposed himself to her. This constitutes a mandated report. (A) is wrong because the parents don't HAVE to be informed, though the therapist may do it. (C) would be fine if the client were not a minor. And (D) is wrong because this is a case that qualifies as a mandated report. (Crisis Management)

A therapist has been working with a client for 4 years. The client has a history of major depression, with periods of sustained remission. The client excitedly informs the therapist that she just learned she is 7 weeks pregnant with her first child and intends to stop taking her medication because she and her husband researched the potential negative effects it could have on the fetus. The therapist is concerned for the client to stop her medication because she experienced significant decompensation last time she went off of them. Which of the following actions should the therapist take in this case? A. Encourage the client to consider staying on medications while recognizing the client's right to self-determination. B. Contact the psychiatrist and share concerns to ensure proper collaboration between psychiatrist and therapist. C. Inform the client that the possible side effects do not outweigh the risk of going into a depression. D. Invite the client's husband for a collateral session to help the couple process the pros and cons of taking the medication.

B. Contact the psychiatrist and share concerns to ensure proper collaboration between psychiatrist and therapist. Rationale: The issue presented is outside the therapist's scope of practice, so it would be important to consult with the psychiatrist to discuss concerns and coordinate care, making B the best answer. This could lead to answer A, but we would want to ensure that we work within our scope of practice and avoid providing medical advice--if this is what the psychiatrist recommends, we could convey what the psychiatrist stated with the client. Answer C is outside the therapist's scope of practice and should come from her MD. If the client noted a desire to bring the husband in for a collateral session, the therapist could do so, but based on the information provided, this step is not indicated. (Treatment)

Kyle, age 6, is brought to therapy by his mother, Lynn, because he has been acting out at school. The school told Lynn that Kyle won't stay in his seat and frequently interrupts the class. Lynn shares that she and Kyle recently moved into a new apartment after separating from Kyle's father. She has sole legal custody of Kyle, but on alternate weekends Kyle visits his father at their old house. After these visits, Kyle becomes more disruptive at home and at school. Lynn says that Kyle's father gives Kyle energy drinks and takes him to watch cage fighting. Lynn appears very anxious in session and shares timidly, "I'm having a really hard time with him. I'm frustrated and don't know what to do anymore. His dad and I are always fighting; I think it has taken a toll on him." As Lynn reports this, Kyle is throwing toys around in the office. What initial treatment goals should the therapist consider in this case? A. Increase family supports; Set clear limits and consequences for Kyle; Improve consistency in parenting styles B. Decrease Kyle's behavioral problems at school; Reduce mother's anxiety; Improve Kyle's transitions between homes C. Decrease Kyle's behavioral problems at school; Increase father's involvement in Kyle's life; Improve communication between the parents D. Determine whether Kyle has ADHD; Reduce mother's anxiety; Improve communication between the parents

B. Decrease Kyle's behavioral problems at school; Reduce mother's anxiety; Improve Kyle's transitions between homes. Rationale: The goals in (B) align with the presenting issues. (A) can be ruled out since there is no indication that there is a lack of social support. (C) is out because it is not clear that increasing the father's involvement would be beneficial. (D) is out because determining whether Kyle has ADHD is part of the assessment, not a goal. (Case Conceptualization and Planning)

A 34-year-old single man seeks therapy for conflict at work and with his family. During the assessment, he shares that he becomes easily agitated with colleagues, frequently explodes at the smallest slights and " isn't sure where the anger is coming from." He tells the therapist that he's gotten a verbal warning at work after the last incident where he snapped at his co-worker during a staff meeting. "And my brother and I have not spoken in weeks. He is upset with me after I yelled at him in front of the whole family," states the client. After carefully listening to the client's presenting complaints, a Psychodynamic therapist would? A. Explain the process of therapy; explore the client's feelings about his week and the intensity of emotions; offer interpretation of the client's anger as it relates to conflicts from the past. B. Determine if the client has a certain level of psychological sophistication to engage in a therapeutic process; explain the process of therapy; wait silently and attentively for the client to relate whatever comes to his mind. C. Explain the process of therapy; remind the client of his treatment goals and ask what he'd like to address first; observe the client's characteristic defense mechanisms. D. Determine if the client has a certain level of psychological sophistication to engage in a therapeutic process; explain the process of therapy; inquire about the client's early childhood relationship.

B. Determine if the client has a certain level of psychological sophistication to engage in a therapeutic process; explain the process of therapy; wait silently and attentively for the client to relate whatever comes to his mind. Rationale: The question is asking about the way in which a Psychodynamic would proceed. Psychodynamic therapists are insight oriented, so they make sure that the client is psychologically minded and sophisticated enough to gain insight from therapy. Psychodynamic therapists are also neutral and non-directive, so they will not initiate the conversation but wait for the client to start once they describe the way therapy works. Thus (B) is the best answer. The therapist's actions in answers (A) and (C) are too direct and involved for them to be attributed to a Psychodynamic therapist. Answer (D) might happen during the later stages, but the therapist is not going to start with inquiring about early childhood. (Treatment)

Nicholas, a 36-year-old lawyer, and his partner Gary, a 43-year-old accountant, are self-referred for couple's therapy. The intake interview reveals that both Nicholas and Gary are in long-term recovery for cocaine addiction. They report having frequent arguments and misunderstandings with each other. "Every single conversation seems to turn into a fight," said Nicholas. "Gary has been tremendously defensive lately," he adds. "Oh it is all on me, isn't it?" retorts Gary. "How about you? You have been completely absent lately. I cannot connect with you at all." As part of the initial intervention the therapist should: A. Teach the couple communication skills; examine relational history; discuss recent stressors B. Discuss recent stressors; assess for relapse in either partner; teach the couple how to be empathetic with each other C. Assign the couple intimacy exercises as homework; teach the couple how to be empathetic with each other; discuss recent stressors D. Teach the couple how to be empathetic with each other; examine relational history; discuss recent stressors

B. Discuss recent stressors; assess for relapse in either partner; teach the couple how to be empathetic with each other Rationale: Given the fact that substance abuse history is present in both clients, assessing whether either partner has relapsed is the most appropriate intervention to start with. Increased distance in the relationship and frequent arguments may be signs that one partner has relapsed, putting stress on the relationship. The only answer that includes assessment for relapse is answer B. The rest of the interventions are also good to do at some point, but checking for relapse would need to be part of the right answer. (Treatment)

A mother brings her 8-year-old son to the clinic for an evaluation. The mother reports he has tantrums at least 4-5 times a week. He is also frequently irritable and difficult to be around. The mother adds that she was deployed for up to six months twice when he was younger but she will be stationed here for a while. During the session the mother and child rarely interact and when they do it is when she scolds him. Which initial diagnoses should the therapist consider? A. Disinhibited Social Engagement Disorder, Major Depressive Disorder B. Disruptive Mood Regulation Disorder, V. Parent Child Relational Problem C. Oppositional Defiant Disorder, V. Parent Child Relational Problem D. Intermittent Explosive Disorder, Separation Anxiety Disorder

B. Disruptive Mood Regulation Disorder, V. Parent Child Relational Problem Rationale: The symptoms point to Disruptive Mood Regulation Disorder with his frequent tantrums and irritability so answer (B) is the immediate frontrunner. There is also an indication that the Parent Child relationship would be a focus of treatment so that adds to the strength of answer (B). (A) is out because he isn't demonstrating attachment issues with others. (C) is out because he isn't arguing with authority and (D) is out because there is no indication of Separation Anxiety. (Diagnosis)

A 19-year-old college student initiates therapy and reports that the previous night she went to an off-campus party. She states that she only had one drink, then blacked out and woke up naked in a bedroom. Which of the following actions should the therapist initially take in this crisis situation? A. Re-establish the client's feelings of control by encouraging her to file a police report to protect other women from the perpetrator. B. Evaluate the client's support systems and coping skills and refer her to an emergency room for an examination. C. Develop the client's trauma narrative of the event to desensitize emotional impact. D. Explore how the client has dealt with similar trauma and notify the university of the assault.

B. Evaluate the client's support systems and coping skills and refer her to an emergency room for an examination. Rationale: The best answer is B. In a crisis you assess coping and support and provide needed referrals -- in this case the ER because it will have a rape kit for testing. Answer A pushes the client to take a particular action. Answer C could be done, but only after we take the steps in answer B. Answer D breaks the client's confidentiality and assumes she's been through something like this before. (Crisis Management)

Molly, a 23 year-old college student, comes to treatment complaining of constant and frequent mood swings. During the assessment phase, the therapist learns that Molly has been diagnosed with diabetes but has not been getting proper treatment for it. She exclaims: "If I monitor what I eat and maintain a proper diet, I am OK. I do not have health insurance and it is too expensive to pay for doctor's visits out of pocket. Could you just help me figure out the number of sugar calories that I consume?" What would the treatment goals be during the initial stages of treatment for a Narrative therapist? A. Separate the client from the problem; explore the story she is telling herself around her illness; assess the client's motivation for treatment. B. Explain that you are not trained to help her with managing her diabetes; assist the client in finding medical resources; identify ways in which Molly's problem with diabetes impacts her life C. Identify ways in which Molly's problem with diabetes impacts her life; provide the client with specific tools that can help Molly monitor her diet; record progress of the client throughout treatment. D. Identify factors that enhance the problem; help the client reduce harm inducing behaviors; talk about the ways diabetes tricks Molly into not maintaining a proper diet.

B. Explain that you are not trained to help her with managing her diabetes; assist the client in finding medical resources; identify ways in which Molly's problem with diabetes impacts her life Rationale: The symptoms being described and the goals of the client are outside the scope of practice for a therapist. The client requires medical services and the therapist's initial goal would be to connect the client with those services. In addition the answer is appropriate because a Narrative therapist would initially focus on gathering a problem-saturated story from a client (B). Although externalizing the problem is part of Narrative Therapy (A), it is not an appropriate intervention, because it does not address the client's medical needs. Providing the client with tools to help her monitor diet (C) and identifying factors that enhance the problem (D) are outside the scope of practice of the therapist. (Case conceptualization and Planning)

A teacher consults with a school-based therapist regarding a six-year-old student who repeatedly arrives at school with the same injuries. The student tells the teacher that the injuries are a result of playing with the family's dog and participating in outdoor sports. The teacher reports the student is increasingly distracted during class, spending more time alone, is being aggressive with classmates, and is reluctant to leave the classroom at the end of the day. What should the therapist do next? A. Discuss the client's injuries with the school nurse. B. Explain to the teacher the need to report abuse. C. Contact the parents to inquire about the child's unusual behaviors. D. Meet with the student to complete an assessment.

B. Explain to the teacher the need to report abuse. Rationale: The information provided in the stem is enough to reasonably suspect child abuse. It is the responsibility of a mandated reporter to notify the appropriate authorities when abuse is suspected (B). Both the teacher and the therapist would not need to make a report. In this case, the teacher would make a report, since he/she has witnessed the child's behavior first hand and the therapist would document that such a report was made. It is the responsibility of the appropriate authorities to investigate whether abuse exists. So (A), (C), and (D) are out. (Crisis Management)

A therapist meets with a client who describes feelings of depression that began soon after her partner moved into the home. The client replies with a flat "fine" when asked about the new arrangement, but also mentions offhandedly that she has to go straight home after the session because the partner monitors the car mileage and time spent away from home. The client seems tense. Which of the following actions should the therapist take to address the relational dynamics discussed in the vignette? A. Explore the possibility of a joint session to address the recent changes in the couple's relationship. B. Explore how the couple has dealt with conflict over the course of their relationship. C. Role-play with client to enhance her ability to set boundaries with her partner and assert her needs. D. Explore how the couple deals with conflict and develop a safety plan.

B. Explore how the couple has dealt with conflict over the course of their relationship. Rationale: The therapist's best course of action would be to start by exploring how the couple deals with conflict (B). This would allow the therapist to gather additional information to determine if domestic violence is present in the relationship. The therapist should not suggest a joint session (A) or role play on how to be more assertive with partner (C) if there is a likelihood of domestic violence. The therapist might want to collaboratively develop a safety plan (D), but only after determining it is appropriate for the client. (Crisis Management)

Loreen, an adult client with developmental disabilities, attends therapy sessions at a community mental health agency. She has been making progress in addressing the impact of her disability both on her life as well as on her family. In addition, she has been learning various tools that help her cope with her disability. In the last session Loreen confides to the therapist that the receptionist at the community center she frequents often treats her like a child and she doesn't know what to do. What actions should the therapist take to help the client address the receptionist's behavior? A. Document instances of inappropriate behavior and process need to report the receptionist's behavior. B. Explore past interactions with the receptionist and role-play alternative responses. C. Discuss the client's concerns with the receptionist privately and provide psychoeducation to the receptionist. D. Help the client process this and any other instance when the client was mistreated.

B. Explore past interactions with the receptionist and role-play alternative responses. Rationale: The client clearly states that she does not know what to do. The situation described in the stem provides the therapist with an opportunity to help the client develop important social skills and role playing would achieve this task (B). Documenting inappropriate behavior (A) or confronting the receptionist privately/providing psychoeducation (C) would not directly help the client. Helping the client process this and other instances of mistreatment (D) does not give the client tools to deal with this situation. (Treatment)

A couple in their late twenties meets with a therapist regarding relational issues. The husband states that the two of them have not had sex in over a year and he is growing more and more resentful each day. His wife reports having severe pain during vaginal intercourse, which has led to reduced sexual desires. "How come it doesn't hurt when you use a tampon, but suddenly it hurts when we have sex?" the husband asks angrily. What diagnosis should the therapist consider for the female partner? A. Female sexual arousal disorder Z code: Relationship Distress With Spouse or Intimate Partner B. Genito-pelvic penetration disorder Z code: Relationship Distress With Spouse or Intimate Partner C. Female orgasmic disorder Zcode: Phase of Life Problem D. Unspecified sexual dysfunction Z code: Phase of Life Problem

B. Genito-pelvic penetration disorder Z code: Relationship Distress With Spouse or Intimate Partner Rationale: The correct answer is B. The wife clearly states that she has severe pain during vaginal intercourse, which meets the criteria for genito-pelvic penetration disorder. The wife's reduced sexual desire and husband's expressed disbelief of the underlying reasons for lack of sexual intimacy point to the Z Code: Relationship distress with spouse or intimate partner. Answer A can be ruled out because the wife is specifically experiencing vaginal pain when attempting to engage in sexual intercourse, this is distinct from a lack of sexual arousal. Answer C is similarly incorrect because the wife does not report problems with orgasms and there is nothing to indicate the couple is entering a new phase of life. Again, answer D includes phase of life, which is not indicated in the vignette and since we clearly see signs of genito-pelvic penetration disorder, unspecified sexual dysfunction can be ruled out.

A therapist meets with a couple that has been married for 52 years. They are seeking treatment because they are struggling with sexual intimacy and both agree they would like to work on "fixing it." The husband states that lately he feels that they are too old to be sexually intimate. The wife shares that she feels unattractive in her body as it has aged, but desires sexual intimacy. Which of the following should the therapist assess in this case? A. The husband's physical health status and willingness to try sexual enhancement products and the wife's ability to achieve climax. B. How the couple defines sexual intimacy and each partners' sexual desires. C. The couples' sexual history and their understanding of unique attributes of geriatric sexual intimacy. D. How each partner perceives beauty and the understanding of geriatric sexual limitations.

B. How the couple defines sexual intimacy and each partners' sexual desires. Rationale: the best answer here is B. Given the presentation, we are first going to assess each partner's sexual desire level and their definition of intimacy. Answer A can be eliminated since evaluating the husband's status of physical health is outside of our scope of practice. Plus, the wife's inability to achieve climax is not indicated in the vignette. Answer C is a bit too generic and not applicable. Getting to know this couple's sexual history is not as relevant to the current presenting complaint. Answer D could be tempting, however "geriatric sexual limitations" are not indicated in the stem and we would be jumping to conclusions here if we were to steer the conversation in that direction. (Clinical Evaluation)

Kim, a 7-year-old girl, is brought to therapy by her mother, Jane. They were referred to therapy by Kim's teacher because Kim had started crying excessively at morning drop-off and is continually asking when her mom is going to pick her up. Jane also reports that Kim is refusing to sleep alone. Jane shakes her head and says, "I just don't understand what is wrong with her. She was never like this before." Which of the following actions should the therapist take to assess this client? A. Provide psychoeducation about separation anxiety; identify familial communication patterns; identify somatic symptoms B. Identify recent changes in family; identify coping strategies; identify familial communication patterns C. Provide psychoeducation about separation anxiety; identify coping strategies; identify somatic symptoms D. Identify recent changes in family; identify coping strategies; identify support systems

B. Identify recent changes in family; identify coping strategies; identify familial communication patterns Rationale: The goal of assessment is to gather information, not provide psychoeducation. Answers A and C can immediately be ruled out as options because the therapist would focus on information gathering, rather than educating the client about separation anxiety. Answer D is incorrect because the 3rd part of the answer - identifying support systems is not as relevant, based on the information provided in the vignette, to understanding the presenting issue as family communication patterns. (Clinical evaluation)

A therapist attends an intensive two-day workshop on treating sexual disorders. Besides this workshop, the therapist has no other training or background in sexual disorders. After attending the workshop, the therapist lists Treating Sexual Disorders as an expertise on his website. This action is: A. Legal since it falls within scope of practice, but unethical due to misrepresentation of scope of competence. B. Illegal and unethical due to the misrepresentation of the therapist's scope of competence. C. Legal since it falls within the therapist's scope of practice and ethical since the therapist attended an intensive training to treat sexual disorders. D. Legal and ethical as long as he cites the specific training he attended.

B. Illegal and unethical due to the misrepresentation of the therapist's scope of competence. Rationale: The actions taken by the therapist are both illegal and unethical (B). The actions violate advertising laws and ethical guidelines by misleading potential clients of the therapist's scope of competence. The actions are neither ethical nor legal (A) or (C). It is not sufficient for the therapist to cite his specific training (D); regardless, it is unacceptable to list false expertise. (Law and Ethics)

A 12-year-old Chinese-American student is referred to therapy because she's been acting out in school. During the second family therapy session, the student angrily states that her mother slapped her across the face and she shows you a mark. The father confirms this occurred but explains the daughter is frequently defiant and verbally abusive toward the mother. Which of the following actions would a Structural Family Therapist use to address the family dynamics described by the daughter and father? A. Demonstrate cultural competency by exploring the family's cultural beliefs related to parenting and discipline. B. Inform the family that a child abuse report term-3must be filed and process their reactions. C. Determine the nature and severity of the slap and file a report. Dterm-3. File an anonymous report after the completion of the session.

B. Inform the family that a child abuse report must be filed and process their reactions. Rationale: In this question, the best answer is B. Ethically, therapists should inform the family when they will report and process this with the clients. This ethical responsibility would be necessary regardless of theoretical orientation. As a mandated reporter, we would not file the report anonymously, ruling out answer D. There is enough to trigger a report so both answers C and A can be ruled out. (Crisis Management)

Kevin, a 48-year-old male, is in treatment for issues of depression. At the end of the sixth session, Kevin angrily states that his 18-year-old stepson is getting on his nerves and it is time Kevin showed him "who is the man in the house." Two days after the session, the therapist receives a phone call from Kevin's wife. She is concerned because Kevin left to go meet her son at the cabin in the woods for a "stern talking to." His wife states that Kevin was very irritable when he left the house. The therapist's responsibility is to: A. Encourage Kevin's wife to call law enforcement. B. Maintain confidentiality. C. Call the police and warn Kevin's stepson. D. Contact Kevin and find out whether he has any means of hurting his stepson.

B. Maintain confidentiality. Rationale: The therapist's responsibility in this case is to maintain confidentiality of the client. Kevin's wife is not giving us enough information to substantiate Tarasoff. Therefore, on the phone with her, we are going to maintain confidentiality and not even acknowledge to her that Kevin is our client (B). Based on the information provided in the stem, the threat of harm is unclear. Encouraging the wife to call 911 (A), calling the police and warning Kevin's stepson (C), or contacting Kevin to find out whether he has means of hurting step son (D) are overreactions based on the information provided. (Crisis Management)

Maricel, a 37-year-old single woman who is Filipino, is referred to therapy by her mother's doctor. Recently, her elderly mother suffered a stroke and is currently in a coma on life support. Maricel has decided that she wants to terminate life support. However, her uncle, who is her mother's brother, is against it. Tearfully, Maricel shares, "In front of the doctor, he says that I have the final say, but in private he is against me doing it. I'm sure that all his siblings are going to take his side in this matter. He doesn't care that this is what my mom wanted." She adds, " I'm crying all the time, not sleeping, and I'm tense. I just don't know what to do. I don't want to get out of bed in the morning." Which of the following human diversity issues should the therapist consider when developing the comprehensive treatment plan in the case presented in the vignette? A. Maricel's cultural and religious views of death and dying and the impact of being a single woman and sole caregiver of mother. B. Maricel's spiritual beliefs regarding end-of-life issues and the influence her Filipino culture plays in this situation. C. Maricel's departure from traditional Filipino roles and the impact of being a single woman and sole caregiver of mother. D. Filipino attitudes towards Western medicine and the role of religion and spirituality in Maricel's family.

B. Maricel's spiritual beliefs regarding end-of-life issues and the influence her Filipino culture plays in this situation. Rationale: When developing a treatment plan, it is important to take into consideration matters that are both relevant to the presenting issues and that address diversity issues, making (B) the best answer. Maricel's spiritual beliefs regarding end-of-life issues are important to take into consideration, since the presenting complaint is her mom's dying wishes. In addition, what sets this case apart from other similar cases is the fact that Maricel is heavily influenced by her culture and her Filipino family in making this decision. Answer (A) is not the best answer because we do not see evidence of Maricel's religious views in the stem or that her being a single woman is somehow impacting her decision making process in this case. Answer (C) is out since there is no evidence in the stem that Maricel has departed from traditional Filipino roles. There are no indicators of answer (D) in the stem either, as there is no mention of Filipino attitudes towards Western medicine or the role of religion in Maricel's caregiving. (Case Conceptualization and Planning)

A therapist in private practice is conducting a weekly support group for postpartum mothers. During a group discussion about the challenges new mothers confront, one of the mothers begins to sob and says she is struggling to get through each day. She notes that she has lost interest in daily activities, spends most of the day in bed and feels disconnected from her partner and baby. How should the therapist clinically manage the client's disclosure? A. Provide psychoeducation to the group about postpartum blues; explore if other group members feel the same; refer to individual therapy. B. Meet with client after group; assess for suicide; refer to individual therapy. C. Normalize the client's experience; explore if other group members feel the same; refer to individual therapy. D. Provide psychoeducation to the group about postpartum blues; help the client identify supports; explore ways to improve connection to family.

B. Meet with client after group; assess for suicide; refer to individual therapy. Rationale: What can we eliminate? (A) and (D) are out because this is more than "blues." That leaves (B) and (C). (B) is the best because it meets with the client privately, assesses for risk, and refers her to individual therapy, which is indicated. (C) is problematic because it normalizes something that, while common, is not "normal." It also invites others to say if they have the same thing and there is a good chance that no one else is feeling as bad as this; so it could increase the client's shame. (B) is the better option as it responds to the severity of the woman's complaint. (Crisis management)

A therapist is planning to see a client at a residential facility for weekly sessions. The therapist is aware that the client is a conservatee and has been appointed a conservator by the court. The client informs the therapist that the conservator is responsible for paying for treatment. What are the next steps in proceeding with treatment? A. Discuss the fee for treatment with the conservator before the onset of treatment; Discuss the progress of the treatment on a regular basis with the conservator; Ask the conservator to sign consent for treatment. B. Obtain a copy of the conservatorship order; Discuss fee for treatment with the conservator before the onset of treatment; Become familiar with the parties involved and their respective rights and responsibilities. C. Send all bills for treatment to conservator; Include the conservator in treatment protocols; Invite the conservator to attend sessions. D. Contact conservator for verification of payment provider; Have conservator involuntarily hospitalize client if necessary; Send all bills for treatment to conservator.

B. Obtain a copy of the conservatorship order; Discuss fee for treatment with the conservator before the onset of treatment; Become familiar with the parties involved and their respective rights and responsibilities. Rationale: (B) is the best answer in moving forward in treating this client. You would not discuss the details of treatment unless that was stated in the order which would be unlikely; therefore (A) and (C) are out. You may do all of the things mentioned in (D) but only after treatment is underway, might you call upon the authority of the conservator to involuntarily hospitalize the client. It is not always necessary for the conservator to consent for treatment, but that could be stated in the order. (Law and Ethics)

A therapist specializing in sex therapy is meeting with a new client referred by his wife. The client, a 53-year-old man, shares that he and his wife have experienced intimacy issues that developed approximately 5 months earlier. The man explains that he never had problems in bed, but started to experience difficulty sustaining an erection. The therapist asks if anything in their relationship changed, but the client cannot identify anything noteworthy. What is the most important factor the therapist should consider during the initial assessment with this client: A. Determine if the client had prior difficulties with past partners or in his current relationship. B. Refer client for medical evaluation to rule out any biological basis for sexual dysfunction. C. Conduct a collateral session with client's partner to evaluate sexual communication and any potential problems. D. Evaluate the client's social and sexual history for potential trauma-induced dysfunction.

B. Refer client for medical evaluation to rule out any biological basis for sexual dysfunction. Rationale: This question is asking what the most important factor that should be considered in assessing this client's sexual dysfunction. It is always important for a therapist to rule out medical reasons for any physical symptoms reported by clients (B). After ruling out a biological basis for the sexual dysfunction, the therapist would gather information included in answers (A), (C), and (D). (Clinical Evaluation)

A therapist is in the middle phase of treatment with a family. The family sought therapy after the daughter and her son moved back home with her 66-year-old father and 65-year-old mother. The daughter had lost her job due to issues with alcohol and moved in after completing a treatment program. At the start of the session, the mother reports that three days ago the daughter relapsed and became violent when they confronted her. What action should the therapist take first? A. Explore each family member's perspective of the incident and create a safety plan. B. Remind the family that the therapist is a mandated reporter and obtain more details about the violent episode. C. Recommend that the daughter seek alcohol treatment immediately and create a safety plan. D. Obtain more details about the nature of the violent episode and file an APS report if necessary.

B. Remind the family that the therapist is a mandated reporter and obtain more details about the violent episode. Rationale: The code of ethics states that therapists must inform the clients of the limits of confidentiality at the start of treatment and then remind them throughout the course of treatment when necessary. We are never going to trap the client into telling us the information that later can be used to make a report. So we are going to tell the client: "I am going to stop you right there and remind you that I am a mandated reporter." Doing so goes in accordance with the code of ethics. In this scenario, there are two elders and a minor. If the violent episode was directed at one of them, a report would be required. So the best answer is (B) remind them of reporting duties then further assess. Neither (A) nor (C) acknowledge the possibility of reporting. (D) Has only APS and it could be CPS. (Crisis Management)

A 24-year-old young man walks into a therapist's office wearing a colorful headband, long purple scarf, and a fur coat. He states that he is happy with who he is, but he would like to better understand personal messages he receives from the newspaper and magazine articles. He adds that his strongly developed telepathic skills allow him to know what other people are thinking about him; thus he warns the therapist about the consequences of lying to him. What diagnosis should be considered initially? A. Schizoid Personality Disorder B. Schizotypal Personality Disorder C. Delusional Disorder D. Histrionic Personality Disorder

B. Schizotypal Personality Disorder Rationale: This client is exhibiting odd, eccentric behavior, ideas of reference, and magical thinking. In addition, he is happy with who he is, which points to the egosyntonic nature of the disorder. All of these symptoms are best accounted for by answer (B). Neither Schizoid PD, nor Histrionic PD have magical thinking or ideas of reference as their symptoms. Thus (A) and (D) are out. Since his behaviors are present across all aspects of his life, it cannot be Delusional Disorder (C). (Diagnosis)

A therapist meets with a new client who reports ongoing headaches and stomachaches. Their doctor has conducted multiple medical tests and has been unable to identify any underlying medical issues. The client continues to schedule doctors appointments. What diagnosis is most appropriate in this case? A. Illness Anxiety Disorder B. Somatic Symptom Disorder C. Factitious Disorder D. Obsessive Compulsive Disorder

B. Somatic Symptom Disorder Rationale: The answer that accounts for the presented symptoms the best is B. The client actually has physical ailments, they are not associated with any medical condition, but the person is clearly preoccupied with them to the point of it having an impact on their life. Answer A is out since the client does exhibit somatic complaints. Answer C is incorrect since there is no evidence of deliberately creating or exaggerating symptoms. Answer D can be eliminated due to the fact that we do not see the presence of obsessions or compulsions in the stem. (Diagnosis)

A therapist working as a school counselor receives copies of illustrations drawn by a 9-year-old student who is currently in the 3 grade. His teacher was concerned because the illustrations include depictions of children being shot at and a bomb exploding in a scene that resembles a hallway of the school. In addition, the teacher informs the counselor that the child has exhibited aggressive behaviors in the school yard and has told the teacher that he "hates his classmates who all think they are better than him." What is the most appropriate response in this case? A. Contact the child's parents to gather additional information. B. Speak directly with the child to assess for threats of violence. C. Encourage the teacher to monitor the child's behavior in the classroom and in the schoolyard to intervene as needed. D. Report the child's illustrations and threats to the police, principal and parents.

B. Speak directly with the child to assess for threats of violence. Rationale: The best place to start here is described in answer B. Since we have not yet met with the student, that would be a place to start in order to properly assess the situation. Answer A could follow the action described in B. Answer C does not adequately respond to the risk presented in the scenario. And answer D is a bit of an overreaction. It might or might not follow after we meet with the child and assess the level of threat.

A therapist received a phone call from a concerned mother regarding her 15-year-old son. She states that he is withdrawn and is exhibiting bizarre symptoms like staying up all night, not eating, and grinding his teeth constantly. What is the most likely diagnosis? A. Bipolar II B. Stimulant abuse C. Unspecified Depressive Disorder D. Cannabis abuse

B. Stimulant abuse Rationale: The combination of indicated symptoms, particularly grinding teeth, point to stimulant abuse (B). There are no symptoms, such as impaired coordination, or sensation of slowed time to indicate Cannabis use (D). There are not enough symptoms to indicate hypomania, thus (A) is out. One might consider a Depression NOS diagnosis (C), due to impaired sleeping and eating, but teeth grinding would point towards stimulant use. (Diagnosis)

A therapist is treating Nora, a 32-year-old chiropractor. Nora sought therapy to help her cope with a horrendous incident that happened to her a couple of months ago when she became a victim of a physical and sexual assault outside of her home. The therapist spent a dozen of initial sessions establishing safety and diversifying Nora's coping strategies. Recently, the therapeutic work has become quite intense. Suddenly, the client begins to say that the event never happened. What is the most likely reason for the client's change of opinion and the best course of action for the therapist to proceed? A. Client's behavior is a sign of repression of the traumatic experience and the therapist ought to normalize the fact that talking about the incident might be rather daunting. B. The client may be avoiding the painful subject and upon venturing such a guess the therapist should return to reestablishing a container of safety. C. The client is redirecting threatening emotions against herself rather than the attacker and the therapist ought to prevent such phenomena by calling it out. D. The client is identifying with the aggressor as evidence by her protecting the negative information against him and the therapist should normalize the process as one that helps the client regain control over the situation.

B. The client may be avoiding the painful subject and upon venturing such a guess the therapist should return to reestablishing a container of safety. Rationale: Based on the information provided in the stem, avoidance (B) is the most plausible reason for the client's change of opinion. There is no evidence in the stem to indicate the client is redirecting threatening emotions against herself (C) or is identifying with the aggressor (D). Repression (A) does not happen from one week to the next, it is instead a defense mechanism that is usually found in kids who unconsciously repress early childhood memories. An adult woman who remembered the incident last week or last month is not likely to repress it between sessions. (Treatment)

A therapist is working with Jhumpa, 42 and his wife Keiko, 38. They immigrated to the United States over twenty years ago from Japan. During the assessment, Jhumpa explains that they live under the same roof as their children and Keiko's mother. "My mom is having trouble sleeping. We think she might be depressed," Keiko notes. The wife and her husband are experiencing ongoing conflict about whether or not to move the mother into a nursing home. What issues should the therapist initially consider when working with this couple? A. The decreased risk of suicide among elderly parents living with their children and Keiko's mother level of depression. B. The couple's values and beliefs about caring for elderly family members and ways in which the couple handles their disagreements. C. Keiko's mother's level of depression and the increased risk of elder abuse due to conflict between the couple. D. The couple's cultural beliefs regarding the appropriate treatment of the mother's depression and ways in which the couple handles their disagreements.

B. The couple's values and beliefs about caring for elderly family members and ways in which the couple handles their disagreements. Rationale: Answer (B) is correct because the therapist should begin by understanding the values and beliefs around the issues and understanding how the couple handles conflict. (A) is incorrect because there is not enough information to determine if there is risk of suicide, and Keiko's mother is not part of the Tx unit. There is also no indication of elder abuse, so (C) is incorrect. (D) could certainly be considered, but is not why the couple is seeking help. (Clinical Evaluation)

Adam, a 52-year-old man, was court-mandated to attend 24 individual therapy sessions after pleading guilty to physically assaulting his partner. To date, he has attended half of his required sessions. He has acknowledged and taken responsibility for verbally and physically abusing his ex-wife. He has also disclosed to the therapist that as a child he was frequently beaten by his father and still has occasional nightmares about the abuse. When the therapist attempted to explore Adam's past trauma and current symptoms in the beginning of treatment, he would shut down and tell the therapist that his court-order requires him to only deal with issues regarding DV. What interventions should the therapist consider at this stage of treatment? A. The therapist should provide psychoeducation to the client about the link between being a victim of abuse and perpetrating violence against others. B. The therapist should consider the increased level of rapport and re-attempt to explore the client's understanding of his personal history of abuse and his relational dynamics in adulthood. C. The therapist should honor the client's expressed disinterest in exploring his past abuse and focus on strengthening coping skills to avoid future perpetration of violence. D. The therapist should continue to strengthen the client's coping skills and inform him of the need to address his diagnosis of PTSD.

B. The therapist should consider the increased level of rapport and re-attempt to explore the client's understanding of his personal history of abuse and his relational dynamics in adulthood. Rationale: The best answer here is B. Since Adam has now been through 12 sessions, the rapport between the client and the therapist is most likely greater than at the beginning. Thus, the therapist should revisit the topic of childhood abuse and help the client understand the impact it most likely had on his adult relationships. Answer A could be tempting, but would have taken place at the beginning of treatment. Answer C might have to come after taking the actions described in answer B, if the client refuses again. Answer D can be ruled out since it is not explicitly stated in the vignette that the client has PTSD. (Treatment)

A 14-year-old boy is referred to the school therapist after fighting with a classmate. The student reports that he has been involved with a gang since he was 12-years-old and has witnessed many fights and shootings. He states that many of his friends have been killed before they turned 18. His symptoms include irritability, aggression and weight loss. What initial diagnosis should the therapist consider for this client? A. Conduct Disorder, Acute Stress Disorder B. Unspecified Depressive Disorder, PTSD C. Conduct Disorder, Unspecified Anxiety Disorder D. Unspecified Anxiety Disorder, PTSD

B. Unspecified Depressive Disorder, PTSD Rationale: According to the DSM 5, the symptoms of irritability, aggression and weight loss, in addition to history of witnessing traumatic events meet the criteria for Unspecified Depressive Disorder and PTSD (B). The symptoms and behaviors described in the stem do not meet the criteria for conduct disorder or anxiety disorder (A), (C), or (D). While we know he has been involved in a gang, we do not have any information about his behaviors that would lend themselves to a diagnosis of conduct disorder. In addition, he does not indicate any excessive worrying. (Diagnosis)

A 27-year-old African American man is court-ordered to therapy as a condition of his parole. He served three years in prison for a drug related charge. The therapist is Caucasian. During the initial session the client is reluctant to engage with the therapist. And finally says, "You're just part of the system of oppression here to tell me what to do." How should the therapist respond? A. "I am not sure what you mean by that; can you be more specific." B. "I am here to help you; it is your choice whether to use therapy or not." C. "I can see your point of view. I imagine you've experienced a lot of oppression and racism in your life." D. "Tell me what you want to work on in therapy."

C. "I can see your point of view. I imagine you've experienced a lot of oppression and racism in your life." Rationale: Clinically, the most effective way to build trust and rapport with a client who is mandated to be in therapy and is suspicious of the therapist's intentions is to meet the client where he is. The only answer that does that is (C). Answer (A) is missing the client and the sentiment he is expressing. Answer (B) also does not address the suspicion the client has just expressed. Answer (D) also ignores the statement the client just made and the feeling behind it. (Treatment)

A therapist working with a same sex couple receives an email from one of the partners. In the email, they ask the therapist to help them tell their partner about their decision to transition to the opposite gender. The therapist helps them process this matter and together they come up with a way to reveal this information to their cisgendered partner. After helping the couple process their reactions, what referral sources should the therapist consider in an attempt to support the couple? A. A group for cisgender spouses of trans mates; Resources that focus on family rights and responding to discrimination related to family relationships B. Support guide for parents of transgender and gender non conforming people; A New York Times resource list for transgender people / youth C. A group for cisgender spouses for trans mates; A peer support group for transgender and gender noncomforming folks. D. A New York Times resource list for transgender people / youth; A community based website with listing of all LGBTQIA+ rehab centers in the area

C. A group for cisgender spouses for trans mates; A peer support group for transgender and gender noncomforming folks. Rationale: The best answer here is C. The question has a very specific focus--identifying referral resources. Since the couple is our client, we want answers that address both partners, which answer C does well. Answer A is incorrect because there is nothing in the vignette that indicates an immediate need for resources that address discrimination related to family relationships. Answer B is incorrect because the support needed is for the couple, not their parents. The NYTimes resource list could be an acceptable answer, but the first part of B makes this answer incorrect. Answer D again includes the NYTimes resource, but the second part of the answer is incorrect, as there is no need for rehab centers. Answer C includes resources that are more directly related to the concerns noted in the vignette. (Clinical Evaluation)

A therapist is working with a client who was in a physically and emotionally abusive relationship for several years. It took the client four attempts before finally leaving the partner. As a result of being emotionally abused over many years, the client exhibits signs of low self-esteem. They constantly express self-defeating thoughts regarding their inability to leave their ex sooner and struggle to move beyond the feeling that they wasted so much of their life on such a poor relationship. They report feeling depressed, overwhelmed, and hopeless about the future. The therapist should do all of the following except: A. Collaborate with the client to develop goals for behavior change. B. Address the client's dysfunctional thinking patterns. C. Accept the client's views of self and circumstances. D. Ask questions to identify client strengths and successes.

C. Accept the client's views of self and circumstances. Rationale: Accepting the client's view of hopelessness would not be appropriate (C). Collaborating with the client on goal development (A), addressing dysfunctional thinking patterns leading to the client's view (B), and identifying client's strengths (D) would all be fitting interventions. (Treatment)

A 24-year-old Asian-American woman is referred to a therapist by her doctor. She tearfully reports that six months ago she was diagnosed with Lupus. She shares that her boyfriend of two years broke up with her a few months ago because he couldn't deal with the news and that she has been having a hard time getting out of bed to get to work. She recently moved in with her parents and states, "My parents say that I am depressed but my doctor says it is part of the disease." She appears clean but disheveled. What action should the therapist take first: A. Provide the client with psychoeducation about how Lupus causes depression. B. Obtain a signed release of information to consult with the referring doctor. C. Ask how the client views the impact of Lupus on her mental health. D. Explore how Lupus is viewed in Asian-American culture.

C. Ask how the client views the impact of Lupus on her mental health. Rationale: The client has indicated that her parents and her doctor have different views of her symptoms. The best place to start would be asking her views on the problem (C). The client has had a lot of changes in the past few months that are likely to affect her mental health in addition to the Lupus. (A) sides with the doctor without taking in to account the other issues. (B) could be done as some point, but not when the therapist is initially connecting with the client. (D) could also be done at some point once the therapist is cleared on how the client views her situation. (Treatment)

A family of three is referred for therapy by their minister. The couple reports that they have found their 14-year-old son drunk on two occasions. One parent shares, "One time I walked in on him drinking alone in his room "and another time he came home smelling of alcohol after his baseball practice." "This is unacceptable to us," adds the other parent. The son responds, "I hate school, I get bullied every day. You guys don't care; you just pray." Which interventions should the therapist consider in this case? A. Refer for substance abuse treatment; obtain a release to speak to school; obtain a release to speak to the referring minister. B. Assess for suicide risk; refer to MD; refer for substance abuse assessment. C. Assess for suicide risk; refer for substance abuse assessment; explore how he is bullied. D. Explore how he is bullied; refer for substance abuse assessment; obtain release to speak to the referring minister.

C. Assess for suicide risk; refer for substance abuse assessment; explore how he is bullied. Rationale: Since there are a few pressing issues presented, speaking to the minister might become relevant at some point, but it is not a priority, thus (A) and (D) are out. There are no indications of any medical issues, thus (B) is incorrect. Answer (C) addresses all of the critical issues presented in the stem, making it the right answer.(Crisis Management)

An unmarried couple, Howard, an Asian American age 39, and Kate, a Caucasian age 24, are referred by Howard's health insurance panel. They share that they recently moved in together. Kate tearfully tells of Howard's lack of affection over the past five months. According to her, he can't control the amount of time he spends on the Internet at work and at home. Howard avoids eye contact and nervously states, "I don't know what I'll do if my boss finds out about my problem." Kate's voice breaks as she says, "I can't live like this! He is spending all our money on porn sites. We can't pay our bills, and today we can't even pay our $10 co-pay. Can we defer that until you cure Howard?" How would Cognitive Behavioral Therapy be used in the case presented in the vignette? A. Explore Howard's early sexual experiences; Assign homework on active listening to improve communication; Clarify Howard and Kate's sexual needs and how they communicate those needs to each other. B. Explore Howard's needs for emotional gratification; Discuss Howard's and Kate's beliefs about their body images; Explore the underlying assumptions that come from Howard's parents. C. Assign homework on active listening to improve communication; Explore Howard's belief system about sexual addiction relating to shame; Learn what automatic thoughts are triggered by those underlying assumptions. D. Ask the couple to describe what their relationship would be like without this problem; Explore the underlying assumptions that come from Howard's parents; Explore Howard's belief system about sexual addiction relating to shame.

C. Assign homework on active listening to improve communication; Explore Howard's belief system about sexual addiction relating to shame; Learn what automatic thoughts are triggered by those underlying assumptions. Rationale: Answer (A) is immediately ruled since CBT theory does not dive into early experiences. Answer (B) is also out since a CBT therapist will not be exploring the needs for emotional gratification or assumptions that come from the client's parents. There is no indication in the stem that Howard and Kate are having body image issues. Answer (C) is a perfect answer since assigning homework, exploring a belief system, and learning more about automatic thoughts are all interventions a CBT therapist would do. Answer (D) is out since it starts with an intervention that is more fitting for a Solution Focused therapist. (Treatment)

Addison, a single mother of two, seeks treatment at a community clinic following the death of her aunt. She tells the therapist that her aunt died very suddenly of a brain aneurysm. "She was there one day and gone the next," Addison states tearfully. "She used to take care of my boys," adds Addison, "they absolutely loved her. She would take them to the library and to the playground. And now I have to take a leave from my job in order to take care of them. That is going to cost me a pretty penny, but there is no one else I can leave them with." The client complains that she is always tired and feels she no longer has time for herself. In creating a treatment plan, which of the following actions should the therapist prioritize? A. Providing the client with grief counseling as her aunt's death seems to be the underlying determinant of her chief complaint. B. Referring the client for a medical evaluation since she is tired all the time and it is important to rule out any possible medical issues according to the standard of care. C. Assisting the client in identifying her immediate needs due to the diverse nature of the issues she is presenting with. D. Referring her to childcare services in order to help the client feel less overwhelmed and better supported.

C. Assisting the client in identifying her immediate needs due to the diverse nature of the issues she is presenting with. Rationale: Based on the information provided in the stem, the immediate needs of the client are unclear and the therapist should therefore assist the client in identifying her immediate goals (C). Once the client's needs are identified, the therapist might find it appropriate to provide grief counseling (A), refer the client for medical evaluation (B), or refer her to childcare services (D). (Case Conceptualization and Planning)

A therapist has been treating a client with schizophrenia who lives in a group home for those with severe mental illness. In one session, he states, "I feel like my conservator controls all of my life and financial decisions and I don't like that; he is stealing my money." How should the therapist proceed? A. Maintain confidentiality and process with your client how he can no longer be under the control of the conservator. B. Call the conservator and express your concern about how he manages your client's financial matters. C. Become familiar with parties involved and their respective rights and responsibilities as ordered by the court. D. Ensure your client that his conservator is not stealing his money and practice reality testing techniques.

C. Become familiar with parties involved and their respective rights and responsibilities as ordered by the court. Rationale: (C) is the best answer. When working with a client who is a conservatee, you will need to work with both the conservator and conservatee. It is good to know what powers the conservator was granted by the court, as this can vary on a case-by-case basis. (A) is out because it isn't the client's choice to have a conservator or not. (B) is breaking your client's confidentiality (a client with a conservator still has the right to confidentiality). (D) is out because there is no way we would not want to dismiss the claim outright without first receiving additional information. (Law and Ethics)

A man is referred to therapy to address work issues he has been struggling with over the past few months. During the initial assessment, the client talks about his difficult childhood, physical abuse by his parents and his two divorces. He is afraid that his current girlfriend is cheating on him and attributes his recent job problems to his chaotic relationship with her. He acknowledges drinking alcohol most nights of the week to help him relax but does not feel it is a problem. Which of the following issues are most important for the therapist to assess? A. Client's referral source; client's potential risk of job loss; psychological response to childhood physical abuse. B. Range of current and past coping skills; psychological response to childhood physical abuse; client's potential risk of job loss. C. Client's referral source; The quantity and frequency of client's substance use; range of current and past coping skills D. Range of current and past coping skills; quantity and frequency of client's substance use; problems with his relationship.

C. Client's referral source; The quantity and frequency of client's substance use; range of current and past coping skills Rationale: This is an example of a question where several answers are okay, but one is stronger as a whole when compared to the others. Two issues that would most strongly influence our answer choice would be the referral source--if the client was self-referred or referred by the girlfriend, our treatment plan would be different than if the client was referred by his EAP. Since answers (B) and (D) do not include this option, they can be eliminated. The other important issue in this case is substance use--if a client indicates potential abuse, we would need to understand this first and foremost when developing our treatment plan, as this would need to be addressed before other issues can be addressed. This allows us to eliminate answer (A). The best answer remaining is answer (C). (Clinical Evaluation)

A 29-year-old graduate student is recommended to therapy by her roommate. She states that she has never been to therapy before and does not want it to last too long. The therapist and the client make a decision that short-term therapy is most appropriate. During the intake interview, the client's chief complaint is general dissatisfaction with life. During the first few sessions the therapist attempts to narrow down the focus of therapy, but the client skirts answers to questions and avoids issues. Each time the therapist picks a topic the client changes the subject. Upon observing these behaviors, the therapist finds himself to be rather annoyed and dreads sessions with the client. Which of the following interventions should the therapist take to address the observed dynamic? A. Explore how the client deals with problems outside of session and document personal feelings in the client's record. B. Respect the client's pace and style of relating and seek consultation in order to prevent personal feelings interfering with treatment. C. Confront the client with regards to this pattern and seek consultation regarding countertransference. D. Process personal feelings via journaling and remain silent in order to allow the client to bring up the material.

C. Confront the client with regards to this pattern and seek consultation regarding countertransference. Rationale: This is a treatment question where there is no crisis issue. Thus we are going to meet the client where they are at, as that is the best way to handle a clinical concern. Short-term therapy indicates a limited number of sessions to work on issues with the client. Because the client is being avoidant, the best intervention would be to confront the client regarding this pattern and process feelings of countertransference in consultation (C). Attempting to explore how the client deals with problems outside of session (A) would be ineffective based on client's avoidance. Answers (B) and (D) could be appropriate for long-term but not short-term therapy. (Treatment)

A young male therapist in a mental health agency conducts an initial interview with a gay client who seeks treatment for depression and problems in his relationship with his partner. The therapist feels uncomfortable when the client discusses his sexuality. At the conclusion of the session, the client expresses a desire to continue the therapeutic relationship with the therapist. What steps should the therapist take to address the ethical obligations noted in this case? A. Since this is a first session, and the therapist has not established rapport with the client, it would be most appropriate to refer the client to another therapist in the agency. B. Continue meeting with the client on a trial basis and process the discomfort with colleagues to identify and address the root cause. C. Consult with colleagues to discuss the root cause of the discomfort and determine if a referral or continued service would be in the best interest of the client. D. Process with client the therapist's lack of experience working with homosexuals and provide the client with referral options to receive the most appropriate care.

C. Consult with colleagues to discuss the root cause of the discomfort and determine if a referral or continued service would be in the best interest of the client. Rationale: This question asks what the therapist should do to manage the ethical obligations. In this case, the ethical obligations are to 1. determine if the therapist can work effectively with the client and 2. if the client would benefit from treatment with the therapist. In this question, the cause of the discomfort the therapist is experiencing is unclear--it could be due to lack of experience, it could be a value conflict, or a different issue altogether. Until the therapist understands the cause of the discomfort, they would not be able to determine if they can or cannot work effectively with this client. The best way to approach this situation and figure out the cause of the discomfort would be to consult with colleagues (C). Following consultation, the therapist may choose to refer the client to another therapist (A). Continuing to work with the client on a trial basis (B) would create instability and would be detrimental to the client's treatment and disclosing lack of experience (D) would cause the therapist to focus on self rather than the client and the vignette does not make clear the therapist's experience working with same sex couples. (Law and Ethics)

A couple in their 70s, Elise and Jim, come to therapy due to conflict over their 26- year-old granddaughter, Jessica. A year ago, after suffering a near-fatal car accident, Jessica, along with her unemployed husband Matt, age 25, moved in with them. Jim complains, "Jessica lays around doing nothing all day, even though she recovered from her injuries months ago. They should both be out looking for work. They can't stay with us forever!" Elise appears upset and states, "My grandbaby can stay with us as long as she needs to! I just thank God that she's alive and in one piece." Jim says he suspects his grandson-in-law is an alcoholic. He reports that he heard the couple fighting violently last night in their room but was afraid to intervene "after what happened last time." What interventions would an Object Relations therapist use in the initial treatment plan in the case described in the vignette? Choose one A. Create the space in which the couple can share their anxieties regarding the current situation and serve as a transitional object for Jim and Elise as they move towards differentiation. B. Interpret the mental representations of Jessica and her husband that Jim and Elise each hold and enable the couple to act as if they are on the same page about their multigenerational household. C. Create the space in which the couple can share their anxieties regarding the current situation and interpret and explore the mental representations of Jessica and her husband that Jim and Elise each hold. D. Facilitate a dialogue between Jim and Elise about the impact of Jessica's car accident on their family and enable the couple to act as if they are on the same page about their multigenerational household

C. Create the space in which the couple can share their anxieties regarding the current situation and interpret and explore the mental representations of Jessica and her husband that Jim and Elise each hold. Rationale: An Object Relations therapist would indeed create the space in which the couple can share their anxieties regarding the current situation in the initial stage of treatment. So the first half of answer (A) is good. The second half, however, is a mumbo jumbo combination of both Bowenian and Object Relations terms. Besides, Jim and Elise have not indicated the desire to differentiate. The first half in answer (B) is a good initial stage Object Relations intervention, but the second half talks about a multigenerational household, which is a Bowenian term. Answer (C) is correct since both parts are good initial stage Object Relations interventions. Answer (D) is out due to the second half that has been discussed in (B). (Treatment)

A therapist is currently treating a 13-year-old child who has a diagnosis of PTSD. His 20-year-old brother, who has a history of substance abuse, physically abused the client. The child informs the therapist that the older sibling recently moved back into the house after losing a job. The parents assure the therapist that they informed social services of the plan and received approval for him to move back into the home. Which of the following actions should the therapist take to manage the client's safety concerns in this case? A. Schedule a joint session with the entire family to assist in the transition and safety plan with the child. B. Contract with the client to report any further abuse and continuously assess for signs of abuse. C. Develop a safety plan with the child and continuously assess for signs of abuse. D. Appeal the decision made by social services and request removal of the perpetrator from the house due to the client's diagnosis of PTSD.

C. Develop a safety plan with the child and continuously assess for signs of abuse. Rationale: Safety is the main objective for the child; thus the development of a safety plan should take priority (C). Answer (A) is out because it is contraindicated to have a victim and a perpetrator in the same session. Options (B) and (D) could be done, but safety is foremost. (Crisis Management)

At the end of the second session with a new client, the client hugs the therapist and tells him that she is so happy to finally find a therapist who knows what he's doing and is helping her to feel better. Between sessions, the client leaves multiple messages on the therapist's voice mail with updates. Which of the following actions should the therapist take in this case? A. Document all interactions in the client's psychotherapy notes; tell the client that, due to ethical concerns, such behavior is problematic; set appropriate boundaries for the therapeutic relationship at the next session B. Recognize the client's need for positive affirmation to establish trust with the therapist; adjust the treatment plan to account for such need; record personal feelings and interpretations as part of the psychotherapy notes C. Document all interactions in the client's medical record; set appropriate boundaries for the therapeutic relationship at the next session; record personal feelings and interpretations as part of the psychotherapy notes D. Interpret the hug and messages as an indications of the client's comfort with the therapist; record personal feelings and interpretations as part of the psychotherapy notes; document all interactions in the client's medical record

C. Document all interactions in the client's medical record; set appropriate boundaries for the therapeutic relationship at the next session; record personal feelings and interpretations as part of the psychotherapy notes Rationale: Based on the behaviors presented in the stem, it is likely that the client has Borderline Personality Disorder. Establishing boundaries (C), particularly with clients with BPD, is an important therapeutic intervention. In addition it is important to write down factual information about the client's behavior in the medical records and therapist's personal interpretations in a separate psychotherapy note. Enabling the poor boundaries by casting the behavior as a way to establish trust (B) or an example of client's comfort (D) would be detrimental to the client's treatment. There is no need to state that such behavior is unethical, since that could be perceived as authoritative and/or judgmental (A). Plus the documentation of client - therapist interaction belongs in the medical record. (Law and Ethics)

A therapist has been meeting with a family for a couple of months. The family started therapy after the father spent 3 months in a rehab facility following a disturbing incident in which he drove home drunk, hit a tree, completely destroyed a family car and got badly injured himself. The family participated in some family therapy sessions in rehab. During those sessions the family learned that there has been a lot of lying going on in the last several years. At this point they continue to work on building trust. During one of the sessions with this therapist, the parents reveal that they have been reading their 16-year-old daughter's emails and have become aware of her sexual activity. The daughter begins crying while the 13-year-old son appears disengaged. Which action should the therapist take to address the parental admission? A. Acknowledge the inappropriateness of reading daughter's email and explore the parents concerns regarding their daughter's sexual activity B. Empathize with the daughter and advise the parents that they were wrong to invade her privacy. C. Encourage the daughter to respond to her parents' disclosure and engage the entire family. D. Schedule a separate session with the daughter and parents to discuss their disclosure and the daughter's sexual activity.

C. Encourage the daughter to respond to her parents' disclosure and engage the entire family. Rationale: The daughter's privacy has been violated and she should be provided with an opportunity to express her feelings and talk about the impact the parental disclosure had on her (C). In addition, the entire family is addressing issues of trust so engaging everyone is important. Answer A is incorrect, because the therapist should allow the daughter to respond to the disclosure. Answer B is incorrect because the therapist would be aligning with the daughter and would likely prevent the family from processing the disclosure. Answer D excludes the son, which would undermine efforts to build trust in the entire family. (Treatment)

Justin is referred to therapy by his EAP for 6 sessions to work on issues of social isolation and poor job performance. Initial assessment indicates the fact that Justin will need more than 6 sessions to deal with his chief complaints. After contacting Justin's EAP provider and advocating for additional sessions, the therapist learns that they will still only allow 6 sessions. How should the therapist proceed? A. Focus on the underlying causes of behavior during 6 sessions then refer out or discuss payment options if continued support is indicated. B. Explain the EAP's restrictions to the client and provide the client with referrals to a different therapist. C. Explain the EAP's restrictions to the client and collaboratively identify appropriate treatment goals for short-term therapy. D. Explain the EAP's restrictions and process referral limitations with client.

C. Explain the EAP's restrictions to the client and collaboratively identify appropriate treatment goals for short-term therapy. Rationale: (A) would not be the way to proceed because we would want expectations to be clear at onset of therapy, not following the completion of 6 sessions. Answer (B) is incorrect because it would still leave him with the same amount of sessions with just a different therapist. Answer (D) is not as strong as the correct answer because we are just processing the limitations with the client rather than working within the clearly defined parameters noted in the vignette. Thus, talking to Justin and developing goals that are of appropriate depth and length (C) is the most optimal solution to the situation. (Law and Ethics)

A therapist meets with a 30-year-old man who is having relationship problems. He reports that he's been dating his partner for almost a year and is feeling conflicted over the direction of the relationship. He found the therapist through his insurance panel and would like to use his insurance to pay for his sessions. After completing an assessment, the therapist informs the client that his presenting issues will not be covered by his insurance. The man becomes upset, tells the therapist he has to use insurance or he cannot afford therapy, and asks the therapist to help him make it work. What ethical and legal obligations does the therapist have in this case? A. Discuss treatment options and encourage him to find a low or no fee clinic. B. Diagnose the client with adjustment disorder and submit a claim to insurance. C. Explain why insurance cannot be used and offer him a list of low fee providers. D. Offer a sliding scale fee to the client based on his ability to pay.

C. Explain why insurance cannot be used and offer him a list of low fee providers. Rationale: Since the client is insisting on using his insurance, we have to respond to the client's request. So we are going to do (C) first. (A) can be done after the therapist provides an explanation. (B) would constitute fraud since it is illegal to make up a diagnosis simply due to reimbursement needs. (D) can also be done, but only after the explanation regarding insurance has been given. (Law and Ethics)

A therapist working on a multidisciplinary team meets with a young adult male who is hospitalized following a failed suicide attempt. The client is agitated and complains that too many staff members know the details of what led to his attempt. He tells the therapist that he is tired of being asked the same questions over and over again. He acknowledges that he "lost it" with a staff member but defended his action as a response to being belittled in front of other patients. After acknowledging the client's feelings, how should the therapist proceed? A. Reassure the client that the staff understands the client's frustration and can deal with his anger. B. Suggest ways the client can better manage his anger. C. Explore how the client thinks the therapist can help with the situation. D. Tell the client that his feelings will be discussed at the next staff meeting

C. Explore how the client thinks the therapist can help with the situation. Rationale: The client is expressing frustration regarding lack of control in his hospitalization, and exploring how the therapist can help (C) allows the client to feel some sense of control. Reassuring the client of the staff's understanding and ability to deal with his anger (A) could be misleading and gives the client permission to continue acting inappropriately with staff. Suggesting anger management techniques (B) does not address the underlying reason for the client's behavior and telling the client that his feelings will be discussed with staff (D) would perpetuate the client feeling a lack of control. (Treatment)

Maria, a Mexican American, brings her 68-year-old mother, Ana, in for therapy. Maria says that she is very concerned about her mother, because she has not been the same since her husband died four weeks ago. Maria relays that her mother hasn't been able to get out of bed. Ana reports seeing and hearing her husband everywhere. According to Maria, her mother never had any type of psychological symptoms previously. How would a therapist demonstrating cultural competency assess this client? A. Refer Ana to a general medical practitioner for a physical evaluation. B. Refer Ana to a psychiatrist for evaluation due to symptoms of delusional disorder. C. Explore the family's cultural views about grief and dying. D. Explore the family's religious views on death and the after life.

C. Explore the family's cultural views about grief and dying. Rationale: The question at the end of the vignette is asking us to assess this client and to be culturally mindful during the process. The only answer that adequately demonstrates cultural competency is C. Exploring family's cultural views about grief and dying (C) should be part of the assessment process. If communicating with the spirit of the dead is a normal part of the culture, then referring Ana to an M.D. (A) or a psychiatrist (B) for the sake of an evaluation would not be necessary and we risk pathologizing behavior that is normal in the cultural context. Answer D is incorrect because it focuses too narrowly on religious views. We do not know if the client is religious or not and if she is, answer C would take that into account while also incorporating other cultural considerations. (Clinical Evaluation)

A mother brings her 7-year-old son for treatment following the advice of his school psychologist who recently diagnosed the child with ADHD. In the initial session, the mother shares that her husband thinks the diagnosis is "bogus" and thinks the child just needs stricter parenting. She adds that she and her husband have always differed on their parenting approaches. How should the therapist initially assess the presenting issue in this case? A. Explore the mother and father's knowledge of ADHD and the school psychologist's initial impressions of the child. B. Explore the differing parenting styles and provide a referral for a medication evaluation for the child. C. Explore the mother and father's concerns about their child and different parent styles and request to speak with the school psychologist and teacher. D. Explore the stimuli that trigger the child's ADHD behavior and provide a referral for medication evaluation for the child.

C. Explore the mother and father's concerns about their child and different parent styles and request to speak with the school psychologist and teacher. Rationale: The best answer is C as it is important to make an effort to include the father and the therapist should be in communication with the school to find out more about this child's particular issues. (Clinical Evaluation)

A couple, David and Xavier, seek therapy because they find themselves at a crossroads in their relationship. Xavier reports feeling unhappy in the relationship because his partner seems emotionally unavailable. David shares that he feels that Xavier seems so needy and insecure in recent months, always questioning his commitment to their relationship. Which of the following interventions would an Emotionally Focused therapist use to address the couple's presenting concerns? A. Explore how Xaiver's underlying insecurities lead to David's lack of emotional attachment and availability. B. Reframe David's exhaustion as a sign that he cares for Xavier and encourage the couple to express their feelings and relationship desires. C. Explore the underlying attachment needs that are contributing to Xavier and David's expressed feelings. D. Teach the couple more effective communication styles that would allow for greater vulnerability and recognition of their emotions.

C. Explore the underlying attachment needs that are contributing to Xavier and David's expressed feelings. Rationale: The best answer here is C. An EFT therapist is going to help the couple go a layer deeper and identify attachment needs that act as a root cause of their emotions. Answer A could be tempting, however, it is not specific enough as to how an EFT therapist is going to go about such exploration. Answer C is more descriptive and specific. Answer B focuses on one partner only in the initial part of the answer, thus it can be ruled out. Answer D is incorrect, since it is unclear that the couple is struggling with vulnerability or recognition of emotion. In fact the stem points to the contrary. Plus, communication skills teaching is going to come after deep exploration that is described in answer C. (Treatment)

A therapist meets with a client who recently retired. The client shares that he had a union job and they offered incentives for early retirement, so he jumped at the opportunity since he was dissatisfied with work. Now that he has entered retirement, he's not sure what to do with his time and is feeling a sense of emptiness. When asked about his daily activities, the client notes that he doesn't engage in any regular activities because he feels too lethargic, and mostly sits around his house. Which of the following actions should the therapist take to assess this client? A. Explore changes in substance use; Identify activities of interest to the client; Refer the client for a medical evaluation B. Administer the Beck Depression Inventory; Inquire about the client's cognitive functioning, sleeping patterns and levels of motivation; Investigate types of support systems available to the client C. Explore what work meant for him prior to retirement; Administer the Beck Depression Inventory; Refer the client for a medical evaluation D. Explore support systems; Identify and contact collateral sources of information; Inquire about the client's AOD use.

C. Explore what work meant for him prior to retirement; Administer the Beck Depression Inventory; Refer the client for a medical evaluation Rationale: The best answer is C. The client is indicating some symptoms of depression, and additional investigation of his depression should be done first. Beck Depression Inventory is a good assessment tool for someone who is exhibiting signs of depression. Since the client reports lethargy a medical evaluation is also a priority (C). Option (A) is out, since there is no indication of drug or alcohol use (AOD). Answer (B) can be a tempting option, however between assessing the client's support system and referring him out for a med eval, the latter is going to take priority given the symptoms. (D) is out since AOD use is not indicated and we will conduct our own assessment of the client first before talking to any collateral sources of information. (Clinical evaluation)

A new client begins therapy at the suggestion of a co-worker who spoke very highly of CBT therapy. In an initial interview the client reveals feelings of tension and trouble concentrating. "I always anticipate the worst in every situation," they state. They also mention having guilt and hopelessness about the future. "Nothing will ever work out for me as it does for others," they note. "I am not meant to be happy." The initial diagnostic impression of generalized anxiety disorder is reached. The client further relates that they frequently get angry with her partner for no reason. After an initial medication evaluation, a Cognitive Behavioral therapist would most likely focus treatment on: A. Assisting the client in identifying the historical origin of their feelings of anxiety; utilizing a self-report symptoms measure to identify baseline functioning; orienting the client to CBT B. Identifying precipitating events that compound the client's difficulties; teaching relaxation skills; utilizing a self-report symptoms measure to identify baseline functioning C. Exploring the client's underlying thoughts and beliefs about themselves and others; orienting the client to CBT; identifying precipitating events that compound the client's difficulties D. Orienting Eileen to CBT; identifying the defense mechanisms she uses to cope with her anxiety; exploring her underlying thoughts and beliefs about herself and others

C. Exploring the client's underlying thoughts and beliefs about themselves and others; orienting the client to CBT; identifying precipitating events that compound the client's difficulties Rationale: Answer (A) is wrong since a Cognitive therapist does not care about the historical origin of feelings. Answer (B) is out since relaxation skills training is more of a middle stage intervention. Answer (C) is the best answer among the ones provided, since a Cognitive therapist would orient the client to CBT, would focus on the client's underlying thoughts and beliefs about themselves and others, and it is consistent with the initial stage of treatment. Identifying events that trigger the client's struggles is also an appropriate Cognitive Behavioral intervention. Answer (D) includes an intervention appropriate for a Psychodynamic therapist, not a CBT one, CBT therapists do not care about defense mechanisms. (Case Conceptualization and Planning)

A 54-year-old woman is referred by her doctor after a recent panic attack. The woman shares that she has been having issues with co-workers and is afraid she may get fired. She states that her husband is her main support, but he is a corporate lawyer who travels a lot and they rarely see each other during the week. She has two grown children who are attending university out-of-state. She has few friends and shares that it has always been hard for her to trust people. Besides the client's self-report about her issues, what additional source of data would an Object Relations therapist use to formulate a complete clinical assessment? A. A report from the husband. B. A report from the doctor. C. How the client and the therapist interact. D. A questionnaire filled out by the client and other family members.

C. How the client and the therapist interact. Rationale: Object-Relations is an offshoot of psychoanalytic theory where the relationship between a therapist and a client acts as one of the most important sources of information. Thus, (C) is the best answer, as it goes along with the main tenet of this theory. The rest of the answers are other good sources of information, but they are not as specific to Object-Relations theory as is answer (C). (Clinical Evaluation)

A therapist is seeing a client with a diagnosis of Major Depressive Disorder. In discussing the treatment plan, the therapist suggests certain goals and interventions. The client quickly agrees, stating, "I'll try anything. I just want to feel better." The client reports a history of failed counseling experiences. In developing a realistic treatment plan, how should the therapist initially proceed? A. Empower the client to take the lead in developing treatment goals to counter lack of motivation among clients experiencing depressive symptoms. B. Explore effective treatment protocols for depression by consulting with colleagues and peer reviewed journals. C. Identify obstacles that hindered earlier treatment efforts since the client has an awareness of what was ineffective in the past. D. Clarify the client's understanding of the treatment process to ascertain client's expectations and identify appropriate interventions.

C. Identify obstacles that hindered earlier treatment efforts since the client has an awareness of what was ineffective in the past. Rationale: Past barriers to reaching treatment goals must be identified first so that the therapist and client can be successful moving ahead (C). The rest of the options listed may be appropriate actions to take but will not be effective until the therapist understands what past treatment was like and what hindered progress in the past. Also, answer A is weak because it would be best for the client and therapist to collaborate on treatment goals, not have one or the other do them on their own. (Case Conceptualization and Planning)

A therapist working in an elementary school is approached by a teacher in the hallway. The teacher asks the therapist for professional advice about a six-year-old student who recently joined the school. The student had been diagnosed with ADHD at his previous placement. "He just does not get along with anyone," states the teacher, "and his classmates do not seem to like him very much. Do you think this is because of ADHD or just his personality?" What steps should the therapist consider given this information? A. Let the teacher know that it is likely a combination of the student's diagnosis and personality that are at play; Discuss possible options with the child's parents; Enroll the child in an after-school social club. B. Let the teacher know that it is likely a combination of the student's diagnosis and personality that are at play; Contact the child's parents; Refer the child for a medication evaluation. C. Inform the teacher about ADHD and implication of such diagnosis; Discuss possible options with the child's parents; Refer the child to a social skills group. D. Inform the teacher about ADHD and implication of such diagnosis; Inquire about the child's academic performance; Recommend the parents request an aide to assist with the child in class.

C. Inform the teacher about ADHD and implication of such diagnosis; Discuss possible options with the child's parents; Refer the child to a social skills group. Rationale: It is beneficial for children diagnosed with ADHD to participate in social skills groups (C), especially when it is noted that the child is having difficulties getting along with other students. An after-school social club (A) might not be appropriate until the child participates in a group that will help him build skills. The referral for a medication evaluation (B) and the need for an aide (D) might be something the therapist deems appropriate in the future, but the first step would be a referral to a social skills group. (D) is also out since the teacher does not mention the child is struggling with academics. (Case Conceptualization and Planning)

Steven, a 14-year-old, is referred to therapy by his pediatrician. He is brought in by his parents, Jen and Bill, who share that Steven has diabetes but hasn't been taking his medication regularly since they moved three months ago. Steven says angrily, "Why do you care? You guys are always working." Jen ignores the comment and adds, "I think he misses his friends, he's been having a hard time fitting in at his new school." What should a family therapist do to initially engage and assess this family? A. Obtain a release to talk to the pediatrician about the reason for the referral and a separate release to talk to Steven's school in order to gain a clearer picture of his social difficulties B. Normalize Steven's non-compliance with his medication as a response to the recent move and inquire about the impact of the move on his social life. C. Invite each family member to share what they think the family issues are and what they hope to get from therapy. D. Observe how the family members engage with each other and reenact their issues in the session.

C. Invite each family member to share what they think the family issues are and what they hope to get from therapy. Rationale: Answer (A) is incorrect, since it is not answering the question that is oriented towards engaging and assessing the family, as getting releases is not a way to engage the family. Answer (B) does not deal with the entire family but only with Steven, so the question is not answered. Answer (C) addresses the whole family and gets every member involved in the conversation about what they see happening and what they want to accomplish. This is the best answer. Answer (D) has the therapist observing, but the question asked us to engage them; this is a wrong answer as well. (Clinical Evaluation)

A 75-year-old woman is referred to therapy by her pastor following the loss of her spouse of 35 years, as well as the loss of her sister. In the initial interview, the client reports she is experiencing recent difficulties with memory loss and confusion, aches in her body, sadness, feelings of loneliness, and a lack of purpose. What should the therapist initially assess? A. Severity and frequency of memory loss and other biological complaints. B. Understanding of the grief process and type of relationship she had with her spouse and sister. C. Level of depression and suicide risk due to symptoms and increased risk factors. D. Understanding of the grief process and interest in attending a support group for recent widowers.

C. Level of depression and suicide risk due to symptoms and increased risk factors. Rationale: Based on the multiple losses, the client's age, and her own reports on her psychological functioning, an evaluation for depression and suicide is the therapist's first course of action (C). An assessment of her memory loss and biological complaints (A) may be appropriate, but that would not be the therapist's first action. An exploration of the relationships she had with her loved ones (B) does not adequately address the immediate concerns. Assessing the client's understanding of the grief process and her interest in attending a group (D) may be appropriate, but like (A), that would not be the therapist's priority. (Clinical Evaluation)

Rose, a 70-year-old woman, is referred to therapy by her doctor. On the phone with the therapist, Rose discloses that for some time now she has been experiencing trouble recalling recent events or recognizing people and places. "I cannot remember what I ate yesterday or where I put my keys for the life of me. I am so frustrated all the time. And I am really terrified that there will come a day when I will not recognize my husband or my kids. What is going to happen to me?!" she exclaims with tears in her voice. The doctor gave her the diagnosis of dementia. Rose is upset and scared. She tells the therapist that she would like to involve her entire family. What should the therapist do first? A. Conduct research on dementia in order to gain knowledge of the disease; make an initial appointment just for Rose; consult with her doctor regarding the best course of treatment B. Obtain a release to speak with her doctor and determine the stage of her disease; make an initial appointment for Rose's entire family; make sure that this case is within the therapist's scope of competence C. Make an initial appointment just for Rose; make sure that this case is within the therapist's scope of competence; obtain a release to speak with her doctor during the initial appointment D. Make an initial appointment for Rose's entire family; obtain a release to speak with Rose's doctor; conduct research on dementia in order to gain knowledge of the disease

C. Make an initial appointment just for Rose; make sure that this case is within the therapist's scope of competence; obtain a release to speak with her doctor during the initial appointment Rationale: Meeting just with the client (C) would be what the therapist should do first in order to complete assessment and come up with a treatment plan. It would also be important to make sure that the therapist has experience working with someone with such a diagnosis. Since there is another provider involved, it is best practice to obtain a release right away to coordinate care. Both answers (B) and (D) will be out since we are going to meet with our client first before involving her family. She was not referred for family therapy, so we will see her individually first, build rapport, conduct assessment and then determine the best way and time to involve the family--she does not specify how she wants the family involved so this does not interfere with self-determination. Answer (A) is not the best here, since we will initially meet with Rose, conduct our own assessment, and only then consult with her doctor. Plus the option of having Rose sign a release that is present in (C) is better than the option of speaking with her doctor right away. One would come before the other. (Treatment)

A 5-year-old child is referred to a therapist for an assessment as part of a custody hearing. The custodial parent has been accused of coaching the child to describe physical abuse by the non-custodial parent. How should the therapist assess the possibility that the child has been coached? A. Ask the child details of the event and observe his ability to correctly answer when the events occurred. B. Observe for the presence or absence of repetitive violent acts and horror scenes during play therapy. C. Observe the child's emotional response when describing the abuse. D. Ask the to retell the story several times to ensure consistency in the narrative

C. Observe the child's emotional response when describing the abuse. Rationale: The correct answer is (C) because the disconnect between the emotion and the information shared is the biggest red flag of all the options presented. In addition, one can train the child how to play, what to say, and how to say it; however, it is difficult to teach the child to emote. (Clinical Evaluation)

A 54-year-old Black man seeks therapy for feelings of low self-worth. His wife encouraged him to "talk to someone." He shares that he is a partner at a large law firm and has been quite successful. However, he finds that the partners and staff at the firm treat him differently because of his race. He shares that they often make comments that they do not realize offend him and make him feel like an "other." What actions should the therapist take in this case? A. Explore the client's understanding of microaggressions and their psychological consequences and teach the client how to effectively address them in the workplace. B. Provide psychoeducation regarding the link between racism and self-worth and explore client's desire to find alternative employment options. C. Process the consequences of microaggressions on the client's mental health and explore how the client can effectively address his concerns at his law firm. D. Explore the client's cognitive distortions and encourage the client to maintain an automatic thought record to challenge harmful thinking.

C. Process the consequences of microaggressions on the client's mental health and explore how the client can effectively address his concerns at his law firm. Rationale: The correct answer is C. Answer A can be eliminated since it is less about exploring his experience with microaggressions and is more of an intellectual activity. It is about processing the impact of microaggressions on his mental health. Plus, engaging the client in a discussion of how he thinks microaggressions should be addressed (C) is better than teaching him ways to address them, which is in answer A. The wording in answer C empowers the client and draws on his own expertise and knowledge. Answer B is ruled out since the client did not express any desire to find alternative employment. Answer D can be easily ruled out since there is no indication in the stem of the presence of cognitive distortions and this could be seen as discounting his lived experience. (Treatment)

A therapist meets with a 48-year-old male with a chief complaint of depressive symptoms, including lethargy and a lack of interest in usual activities. The client reports a history of regular recreational cocaine use, although he reports refraining from use for several days and has a desire to maintain sobriety. What initial action should the therapist take in this case? A. Refer client to psychiatrist to evaluate severity of depressive symptoms and to determine appropriateness of an SSRI. B. Encourage the client to attend Narcotics Anonymous since he is in the action stage and recommend he participate in individual therapy to address depressive symptoms. C. Provide the client with psychoeducation regarding common withdrawal symptoms linked to cocaine withdrawal, including depressive symptoms. D. Affirm the client's desire to maintain sobriety and help identify support systems to assist the client with his stated goal.

C. Provide the client with psychoeducation regarding common withdrawal symptoms linked to cocaine withdrawal, including depressive symptoms. Rationale: The correct answer is (C) because prior to referring the client to NA (B) or for a psychiatric evaluation (A), and affirming the client's motivation to maintain sobriety (D), the therapist should first educate the client on the nature of his/her symptoms. Understanding that repeated use of cocaine depletes the brain of the neurotransmitters necessary to sustain a normal mood, which results in depression will help the client grasp and deal with the reality of the situation. (Treatment)

A therapist meets with two young female girls, ages 4 and 7, who were referred to therapy by their legal counsel. The girls arrived at the US border a year prior and were separated from their parents shortly after their arrival. They spent several months in a detention facility and were subsequently placed with a family. Their parents have not been located. What should the therapist initially assess? A. Manifestations of post traumatic stress due to separation from parents; Current living arrangement and psychosocial supports; Efforts to reunite children with their biological parents. B. Manifestations of post traumatic stress due to separation from parents; Access to internal and external coping skills; Verbal and nonverbal communication patterns. C. Psychological and biological consequences of migration; Current living arrangement and psychosocial supports; Presence of adaptive coping skills for each child. D. Psychological and biological consequences of migration; Access to internal and external coping skills; Efforts for reunification of children with their biological parents.

C. Psychological and biological consequences of migration; Current living arrangement and psychosocial supports; Presence of adaptive coping skills for each child. Rationale: Initially, the therapist should focus on assessing the consequences of migration, available supports, and the presence of coping skills (C). Answer A can be ruled out since it assumes that the girls have post traumatic stress, but it is not yet determined according to the stem. While the scenario described appears to be traumatic, we still need to see symptoms of PTSD described when considering the diagnosis. In addition, the first part of answer C is more comprehensive. Answer D can be ruled out since the stem clearly states that the girl's biological parents have not been located. They would first need to be located before reunification efforts can begin. Answer B is out both because of the reference to PTSD and because their communication patterns have not been mentioned in the stem. (Clinical Evaluation)

A six-year-old child lives with a foster family. His father is in prison and his mother is in residential treatment for alcohol dependence. The child is small for his age, often has temper outbursts, and has difficulty completing schoolwork. The therapist notes that his speech is immature. How should the therapist assess this client? A. Request the child's file from the social worker; work with the foster parents on a behavior modification plan; obtain a release to speak to the teacher B. Suggest that the child's teacher refer him for special education placement; have the parents sign a release authorizing the therapist to speak to a social worker; obtain a release to speak to the teacher C. Refer the child for assessment for fetal alcohol syndrome; gather child's history; request the child's file from the social worker D. Refer the child for assessment for fetal alcohol syndrome; work with the child's biological mother toward reunification; suggest that the child's teacher refer him for special education placement

C. Refer the child for assessment for fetal alcohol syndrome; gather child's history; request the child's file from the social worker Rationale: Proper assessment is going to inform the treatment plan developed by the therapist. The child's mother is in treatment for alcohol dependence and the child is exhibiting symptoms that might be accounted for by Fetal Alcohol Syndrome. Therefore, the best first step would be to refer the child for assessment to identify or rule out this disorder, gather the child's history and get a hold of the child's file. Thus, answer (C) is correct. Answer (A) jumps to creating a behavior modification plan, but this cannot be done without first gathering additional information. Answer (B) ignores the therapist's responsibility for proper assessment given the child's presenting symptoms. Answer (D) is not indicated, since it is not stated in the stem that his biological mother wants reunification. (Clinical Evaluation)

An adult female seeks therapy for depression. During the initial intake she tells the therapist that following a car accident several months ago she was prescribed painkillers and has been taking them daily. She admits to the therapist that she tried to taper off of them when the pain eased, but the physical and psychological effects were "unbearable." How should the therapist further assess the client in this case? Choose one A. Refer the client to a psychiatrist for a medication evaluation in order to mitigate symptoms of withdrawal. B. Refer the client to an inpatient substance abuse treatment program. C. Refer the client for a comprehensive medical evaluation. D. Utilize the motivational interviewing to determine the client's stage of change.

C. Refer the client for a comprehensive medical evaluation. Rationale: The best answer is C, as it is the most appropriate referral given the severity of the situation, medical origin of the substance use problem and the presence of physical symptoms during withdrawal. Answer A can be eliminated since a more comprehensive medical evaluation is going to take priority. Answer D will take place after the medical evaluation, while answer B might take place afterwards. However, the first step is going to have to be a referral to a physician who can then be on board, supporting and monitoring the client while she deals with her physical and mental health. (Clinical Evaluation)

A therapist is treating a client who is 5 months pregnant. The woman experienced postpartum depression after the birth of her first child and fears she is likely to experience it again once her second child is born. She recently informed her employer that she was pregnant and planning to take maternity leave following the birth of the child. Shortly after sharing her intentions, she received a written notice of termination from employment due to "personnel conflicts," which surprised her as she recently received an excellent review. What action should the therapist take in this case? A. Encourage the client to request disability insurance and refer to a psychiatrist to address risk of postpartum depression. B. Explore what coping skills helped the client after birth of first child and refer to psychiatrist C. Refer the client to legal services regarding termination and explore appropriate support for postpartum depression. D. Encourage her to confront the manager about her termination and refer for legal services.

C. Refer the client to legal services regarding termination and explore appropriate support for postpartum depression. Rationale: The client has presented a legal issue to the therapist, which is outside the therapist's scope of practice. Therefore, the therapist would best serve the client by encouraging her to seek legal advice by someone qualified to provide those services. Answer C addresses both the legal needs and postpartum depression making it the best answer. Encouraging the client to request disability insurance does not address the potential legal issues that arise in this case and the psychiatrist may or may not be appropriate at this time (B). Confronting the manager could be detrimental to the client if she is to pursue legal action (D). (Treatment)

A therapist meets with an 11-year-old girl for an intake session. Her parents are having difficulty managing her behaviors at home and her teacher has concerns about her lack of social interactions at school and disruptive behavior in class. What should the therapist prioritize during the assessment phase of treatment? A. Referral for psychological testing; Ability to complete school assignments; Cultural explanations of behavior. B. Need for parents to obtain therapeutic support services; Refer for a medical evaluation; Assess for level of impairment due to ADHD. C. Refer to medical evaluation; Parents openness to engage in home behavioral exercises; Ability to complete school assignments D. Appropriateness of a social skills group; Implement a reward system to reinforce positive behaviors at home and school; Refer parents to a support group for parents of children with ADHD.

C. Refer to medical evaluation; Parents openness to engage in home behavioral exercises; Ability to complete school assignments Rationale: The best strategy to use here is the process of elimination. Answer A can be eliminated since it includes cultural explanations of the behavior, yet the vignette indicates that the parents are also struggling at home, so there is no disconnect between home and school. Answer B is out since it assumes that the client has ADHD and that has not yet been confirmed. Answer D is out since it includes an intervention, which would not happen in the assessment phase. Thus the only answer we are left with is C. (Clinical Evaluation)

A therapist sees a 13-year-old child for therapy and suspects the mother is physically abusing the child. The therapist had difficulty building rapport with the child who, until recently, was unwilling to participate in sessions. The therapist is concerned that if a report is made to the child protection agency, the therapeutic relationship will be damaged. What actions should the therapist take to manage the legal and ethical issues presented in this case? A. Meet with the child individually to discuss concerns regarding safety and to assess for abuse. B. Refer the child for a medical evaluation to assess the client's physical well-being and potential effects of physical abuse. C. Report the suspected abuse and process the report with the child. D. Report the suspected abuse and process the report with the child and mother.

C. Report the suspected abuse and process the report with the child. Rationale: Legally, therapists are mandated to immediately file a child abuse report if they suspect abuse and should process the need to report with the client(C), once we have reasonable suspicion, we must report regardless of the effect on the therapeutic relationship. Answer (A) is incorrect because we do not need to assess further once we suspect the abuse. Following the report, we may choose to refer the child for a medical evaluation (B). Our obligation is to process the report with our client, it's not clear how involved the mother is in treatment or if it would be appropriate to discuss with her as well, so (D) is not as strong as answer (C). (Law and Ethics)

A therapist sees an 18-year-old teenager with a diagnosis of bulimia nervosa and a recent pattern of cutting. The teenager's pediatrician believes her bulimia in conjunction with recent self-harming behaviors make hospitalization necessary. The parents have taken their daughter to a psychiatrist who determined the client has a comorbid condition of major depressive disorder and recommends intensive outpatient treatment. The client and her parents are confused and frightened. How should the therapist ethically manage the client's needs in this case? A. Explore the pros and cons of inpatient versus outpatient therapy with the client and her parents and empower them to make a decision. B. Provide psychoeducation to the family about the client's comorbid diagnosis and complete a Beck Depression Inventory to determine if hospitalization or outpatient treatment is indicated. C. Schedule a case conference with the professionals involved to collaboratively determine what is in the client's best interest and discuss the outcome with the family. D. Meet with the client and her parents to process their fears and confusion, and help concretize what steps would be most appropriate for the daughter considering her diagnosis and immediate safety concerns.

C. Schedule a case conference with the professionals involved to collaboratively determine what is in the client's best interest and discuss the outcome with the family. Rationale: The client's behaviors can be considered high-risk and the most appropriate course of action should be coordinated among all the professionals working with the client (C). This would be the best course of action, since the code of ethics emphasizes the importance of collaborating with all health care providers involved in our clients' cases. This way the providers can come to a unified solution and present it to the client without confusing the client. Following the case conference, the therapist could take the actions outlined in answers (A), (B) or (C). (Law and Ethics)

Allison, a 25 year-old woman is referred to therapy by her co-worker. She tells the therapist that in recent days she has been having visits from her grandmother who was very dear to Allison and died 4 years ago. She reports a poor appetite, excessive crying, and general apathy. During the initial assessment, she states, "I am not always sad. There are weeks that my grandmother visits and cheers me up. During those times I do not feel blue at all, but it always comes back." What provisional DSM diagnosis should the therapist consider? Choose one A. Schizophrenia B. Major Depressive Disorder with psychotic features C. Schizoaffective Disorder depressive type D. Persistent depressive disorder

C. Schizoaffective Disorder depressive type Rationale: The correct answer is C. Answer A is incorrect because we see a mood disorder present when psychotic symptoms are not present. With schizophrenia, we would see psychotic symptoms present most of the time and the mood disorder coming and going. In order to select answer B, the depression would be pervasive and the psychotic symptoms would occur at the same time as the MDD. Answer D can be ruled out because it does not take into account the hallucinations described in the vignette. Answer C is the strongest, we see psychotic features present and then disappear and we see depressive symptoms present and fade. (diagnosis)

A woman explains to the therapist that her family recently survived a major car accident. She is worried because every time there is a loud noise, her six-year old daughter becomes terrified and hides in the closet. The daughter looks frightened as she clings to her mother. How will the therapist know it is time to terminate with the child? A. The child is able to express her feelings related to the car accident. B. The child is able to ride in cars without becoming scared. C. The child no longer becomes terrified when hearing loud noises. D. The child is able to reenact the accident through play therapy without becoming aroused.

C. The child no longer becomes terrified when hearing loud noises. Rationale: One of the ways to determine that it is time to terminate is when the client is symptom free. Since this client's presenting complaint is around loud noises, once the little girl is no longer terrified of them, it would be time to terminate (C). (Case conceptualization and Planning)

Melanie, a 60-year old widow, reports that her husband passed away approximately a year ago and since that time, she has been unable to work due to ongoing health problems. In addition she reports feelings of frustration and disappointment. She does not want to talk to her friends or to her daughter about her feelings since she does not want to bring anyone down along with her. In creating a treatment plan the therapist should first consider: A. The client's need for bereavement counseling; if the client has an adequate support system; client's ability to pay for services B. The client's level of depression; the client's need for bereavement counseling; if the client has an adequate support system C. The client's need for bereavement counseling; the nature and severity of her health problems; if the client has an adequate support system D. The client's level of depression; the client's need for bereavement counseling; client's ability to pay for services

C. The client's need for bereavement counseling; the nature and severity of her health problems; if the client has an adequate support system Rationale: In determining any client's treatment plan, it is important to rule out substance abuse and any medical issues. In this stem, it is indicated that the client is experiencing health problems; thus the first thing to consider is the nature and the severity of such concerns (C). After ruling out medical conditions, and thus any possibility of health risks, the therapist can move on to evaluating the client's support system and ability to pay (A), level of depression (B), and/or the need for bereavement counseling (D). (Case conceptualization and planning)

A therapist sees a couple, Sandra and Josephine, shortly after the birth of their first child. The two explain that they went through several years of finding a donor and in vitro fertilization before Sandra became pregnant. Now that they finally have a child, they are feeling very stressed by the demands of parenthood and guilty that they are not thoroughly enjoying what they worked so hard to achieve. They find themselves arguing over who should complete various tasks related to child rearing and general household chores. In addition, they share that both of their families live out of state so they cannot rely on them to help with childcare. What information is most important for the therapist to consider when developing a treatment plan? A. The couple's understanding of how changes in a family's life cycle can exacerbate stress levels; their ability to communicate feelings of guilt and the emotional strain caused by the birth of their baby; their ability to properly care for the newborn. B. The type of relationship the couple had prior to the birth of their child; their ability to communicate feelings of guilt and the emotional strain caused by the birth of their baby; the couple's social support and access to resources. C. The couple's understanding of how changes in a family's life cycle can exacerbate stress levels; their ability to communicate feelings of guilt and the emotional strain caused by the birth of their baby; their social support and access to resources. D. The couple's willingness to engage in treatment; the type of relationship the couple had prior to the birth of their child; their ability to communicate feelings of guilt and the emotional strain caused by the birth of their baby.

C. The couple's understanding of how changes in a family's life cycle can exacerbate stress levels; their ability to communicate feelings of guilt and the emotional strain caused by the birth of their baby; their social support and access to resources. Rationale: The therapist would need to collaborate with the couple and understand how the couple views the issues causing stress, how communication plays a role in their relationship and ability to cope with the stress and guilt; and since the couple notes their families are far away, identifying supports or lack thereof would be helpful (C) when developing a treatment plan addressing their specific needs and concerns. Answer (A) is incorrect because there is nothing to indicate an issue with their ability to care for the newborn. Answer (B) is not as strong as (C) because the relationship they had prior to the birth of their child is not as important as their current ability to communicate. It is important to pay attention to what the stem is discussing. The stem doesn't indicate a lack of willingness to participate in therapy (D). The stem specifically notes stress caused by the birth of the first child, arguing, and geographical distance from family, so that is where the therapist should start when trying to develop a treatment plan. (Case Conceptualization and Planning)

A 17-year-old male client is referred by his immigration lawyer. The lawyer is actively working with the client as he seeks asylum. The client is from Saudi Arabia and was forced to leave the country and his family after he was caught engaging in sexual acts with another man. What should the therapist initially consider when conducting a clinical evaluation in this case? A. The need for a qualified interpreter; the client's understanding of therapy and confidentiality; the manifestations of PTSD in relation to the client's level of functioning. B. The need for a qualified interpreter; the client's understanding of therapy and confidentiality; the likelihood of the client obtaining asylum. C. The role of culture in the manifestation of mental health problems; the need for additional adjunctive resources; the client's current relationship with his family of origin. D. The role of culture in the manifestation of mental health problems; the need for additional adjunctive resources; the client's understanding of therapy and confidentiality.

C. The role of culture in the manifestation of mental health problems; the need for additional adjunctive resources; the client's current relationship with his family of origin. Rationale: The best answer here is C. Given the man's situation, understanding the role of his culture and his current involvement with his family as it relates to mental health would be important. Plus we would want to see if he needs adjunctive resources, such as an interpreter, housing, religious or social connections, or vocational support, given that he is trying to establish himself in this country. Answer A can be ruled out since there is no indication of PTSD in the stem and he might need access to greater resources than just an interpreter, so answer C is more comprehensive. Answer B is also out since concerning ourselves with the likelihood of him obtaining asylum is not going to be our priority in this case. Such matter lies outside of our scope of practice and does not have any current implications on his mental health as evidenced in the stem. Answer D could be tempting, however, the stem does not state that the client does not understand therapy or the concept of confidentiality, thus this answer could be ruled out. (Clinical Evaluation)

A therapist is working with a family consisting of a mother, father, and 15-year-old daughter. The mother and daughter are arguing with each other about her curfew while the father sits in silence. The daughter says, "Every time I come home a little bit late, my mom yells at me. I can't take it anymore." The dad says to the therapist, "See what I have to deal with?" Which of the following questions demonstrates the Systems theory concept of circular causality? A. To the mother, "What feelings arise for you when your daughter yells at you?" B. To the daughter, "When you arrive late, what happens after your mother yells at you?" C. To the father, "How do you respond when your wife yells at your daughter?" D. To the mother, "How do you think your yelling affects your husband?"

C. To the father, "How do you respond when your wife yells at your daughter?" Rationale: Circular causality, an intervention used in family therapy, tries to determine patterns in family dynamics. By asking the father how he responds to the wife yelling at the daughter (C), the therapist is able to understand how the entire system functions together. Asking the mother what it feels like when her daughter yells (A), asking the daughter what happens when she comes home late (B), or asking the mother how her yelling affects the father (D), does not allow the therapist to understand how the different parts of the system work together. (Treatment)

Elizabeth, a 19-year-old college student, comes to therapy complaining of body image issues. She is average weight, which makes her unhappy and she wishes she were skinnier. "I am constantly on a diet," she states "and once a week I feel so bad about eating even a little bit of food that I have to throw up to feel better." In addition she shares that this behavior has been going on since the academic year began, which was 5 months ago now. Elizabeth's likely diagnosis is: A. Anorexia nervosa B. Bulimia nervosa C. Unspecified eating disorder D. Body dysmorphic disorder

C. Unspecified eating disorder Rationale: Since Elizabeth is of average weight, the diagnosis of anorexia nervosa is out (A). Elizabeth does not meet full criteria for bulimia nervosa (B) because for such diagnosis to be present one must engage in binging and compensatory behaviors at least once a week for three months. Thus, the correct answer is (C). (D) is out because her symptoms are better accounted for by the diagnosis of unspecified eating disorder. (Clinical Evaluation)

A recently divorced mother and her teenage daughter are self-referred for therapy. The mother complains that since she divorced, the daughter constantly talks back, stays out past curfew, and "sneaks" her 17-year-old boyfriend into the house. The mother states, "It's tough enough to raise a daughter alone. I can't even get her to go to school." The daughter replies, "You and your religion make a big deal out of everything. I hate being home when you're always running those bible study groups. Just back off!" Outside of the session, the daughter informsthe therapist that she is two months pregnant and is considering having an abortion. She asks that her mother not be told about the pregnancy. Assuming the mother and daughter are the unit of treatment, what interventions would a Solution Focused therapist use in the initial stage of treatment? A. Instruct them to describe what their relationship would be like if they were getting along; identify their automatic thoughts about one another. B. Ask them to identify a time when they were getting along; draw a family map to understand their roles. C. Utilize scaling questions to clarify the nature of their situation; ask them to identify a time when they were getting along. D. Tell them to resist getting better; ask them to identify a time when they were getting along.

C. Utilize scaling questions to clarify the nature of their situation; ask them to identify a time when they were getting along. Rationale: The best answer is (C). Both scaling questions and asking to describe a time when they got along are in line with Solution Focused therapy. (A) is out because of the automatic thoughts. In (B), drawing a family map is used in Bowen. And in (D), telling them to resist getting better is a Strategic Intervention. (Treatment)

A 43-year-old woman, who is the mother of two teenagers initiates therapy to work on becoming more assertive. She complains at the onset of therapy that no one listens to her, whether she is at home or at work, and she is tired of feeling insignificant. During the initial session she reports wanting concrete tools that will help her with this matter. Once therapy begins, she is unable to carry out treatment tasks or practice the tools provided by the therapist. She indicates she is fearful of initiating change particularly since her culture does not approve of assertive women. Which of the following interventions should be used in this situation? A. Support the client's efforts to pursue her original goal and offer to provide family therapy to address communication problems. B. Acknowledge the difficulty in carrying out treatment tasks and reformulate treatment goals with the client. C. Validate the client's fears and explore her perception of the cultural conflict. D. Seek consultation to address cultural considerations more effectively with client.

C. Validate the client's fears and explore her perception of the cultural conflict. Rationale: Validating the client's fears and exploring her perception of the cultural conflict (C) would allow the therapist to address the client's expressed concerns and start where the client is. Supporting the client's efforts to pursue her original goal and offering family therapy(A) as well as seeking consultation (D) ignores the client's perceived impediments to achieving the goals. Reformulating the goal with the client (B) would be premature and should not be considered until after the cultural component is explored. (Treatment)

A therapist is conducting an intake interview with Athena, a 48-year-old woman. Athena has experienced episodes of depression throughout her life. When she was 6-years-old, her older brother died in a car accident. According to her, this event completely pulled her family apart and she was lost between her mother's grief and her father's depression. At the end of the session she asks, "How would I know when therapy is over?" How would a Cognitive Behavioral Therapist respond to this inquiry? A. "You will tell me what your goals for therapy are, and when you reach those goals we will terminate." B. "We will collaborate on formulating our treatment goals, and I will tell you when it is time to terminate." C. "I will determine what the treatment goals are based on what you told me, and once you feel that you have reached them we will terminate." D. "We will collaborate on formulating the treatment goals and once we've determined you are close to reaching your goals we will discuss maintaining progress and begin termination."

D. "We will collaborate on formulating the treatment goals and once we've determined you are close to reaching your goals we will discuss maintaining progress and begin termination." Rationale: One of the tenets of CBT therapy is a collaborative stance. Answer (D) reflects such collaboration and includes proper protocol for termination, which is spreading out the sessions. Answers (A), (B), and (C) do not suggest collaboration and give either client or the therapist too much control. (Treatment)

Immediately following a group psychotherapy session, a group member, Clara, returned to the office of the therapist to privately discuss an incident that had occurred during the session. The client reported that another member, Mandy, had broken a group rule by being disrespectful. Clara complained, "I know at the very beginning we talked about not forming cliques in the group. Well Mandy sure did not hear this rule. She was openly talking about Clarissa and her weight to others and I know Clarissa could hear her. Then she invited some of the group members to her house for a party, but did not invite the rest of us. She is so rude! I want you to properly deal with this matter." Which of the following would be the most appropriate therapeutic response? A. Explore the client's feelings about observing the group member's disrespect individually with Clara and inquire if other group members shared this experience when the group meets next. B. Separately process the client's reasoning for not addressing the situation immediately in the group and revisit group rules to ensure a safe environment during the next group meeting. C. Assure the client that it is acceptable to privately report such concerns and maintain the client's confidentiality. D. Acknowledge client's concerns and suggest the client consider bringing up this incident for discussion in the next group session.

D. Acknowledge client's concerns and suggest the client consider bringing up this incident for discussion in the next group session. Rationale: When a group member expresses concerns about group dynamics, it is always important to deal with the issue with the group as a whole (D). Answers (A) through (C) do not provide an opportunity for the group as a whole to deal with the issue so we would prioritize the answer that brings it back to the group. (Treatment)

A therapist working at a community clinic is meeting with a 16-year-old girl for the first time. The girl reveals that she has been depressed for most of the past year. To help her cope, she has been buying sleeping pills off her classmates and friends. During the session the therapist determines that it is necessary to initiate a 5150 because the client presents as a serious danger to herself. After informing the client of the need to initiate a 5150, she becomes upset stating that such a step is not necessary. She demands that her mother who is in the waiting room be included. The mother becomes hysterical. How should the therapist proceed? A. Acknowledge the intensity of the situation; create a safety plan for mother and daughter; determine if there is another family member available for support B. Validate the client's fear; assess further to see if the client has a plan and how many pills she has available; invite a colleague into the room to help manage the situation C. Validate the client's fear; determine if there is another family member available for support; discuss the possibility of voluntary hospitalization. D. Acknowledge the intensity of the situation; contact the Psychiatric Emergency Team to initiate a 5150; process with mother and daughter.

D. Acknowledge the intensity of the situation; contact the Psychiatric Emergency Team to initiate a 5150; process with mother and daughter. Rationale: The best answer is D. The therapist already determined a need for a 5150, so there is a clear reason to move forward with that despite the mother and daughter being upset. Since the daughter stated that hospitalization is not necessary, an option of voluntary hospitalization is no longer viable. So (C) is out. The stem states that the therapist has already determined the need for 5150; therefore further assessment is not indicated. So (B) is not the best answer. Answer (A) could be done later, but at this point we need to address the need for hospitalization. (Crisis Management)

A therapist is running a support group for teen moms. During one of the group sessions, a 17-year-old client reports that her boyfriend came home high last night, accused her of cheating on him, and started hitting her. "It was like he lost his mind. There was no reasoning with him. He is a completely different person when he is high," she stated. She shows the group members the bruises on her forearms that she had held up to protect her face. How should the therapist proceed in clinically managing the client's disclosure? A. Assess how the client's child was impacted by the incident; Create a safety plan for the mom and her child; Process the incident with the group. B. Encourage the client to go to the hospital; Create a safety plan for the mom and her child; Make a report to protective authorities. C. Determine the client's current level of safety; refer client's boyfriend to drug treatment; Encourage the client to go to the hospital. D. Assess how the client's child was impacted by the incident; Inform the client that a child abuse report must be filed; Process the report with the group.

D. Assess how the client's child was impacted by the incident; Inform the client that a child abuse report must be filed; Process the report with the group. Rationale: Since the client is a 17-year-old and she was physically abused, it would be the therapist's obligation to file a CPS report immediately. The only two answers that have that option are (B) and (D). (B) is a good option, however, if we compare different aspects of each of these two answers we can see that some of the wording is better in (D) than in (B), and we would do (D) before (B). For instance, we would assess the child's safety before sending the client to the hospital. Informing the client that a CPS report must be made and processing it with her and with the group, as opposed to simply making a report, would be a better way to manage this situation clinically since it allows both the client and the group to process that information. Answer (A) does not have an option of making a report, which must be in the answer. And answer (C) does not have that option either, but it also has us referring the client's boyfriend to drug treatment, and he is neither here in the room, nor is it a priority before a CPS report and her child's safety assessment. (Crisis Management)

A therapist is meeting with a gender non-conforming teen. The teen tells the therapist that things have been tough for them recently. Even though they have a good group of friends at school, their parents at home just do not understand what they are all about. They want them to stop dying and cutting their hair and grow it out. They keep pressuring them to dress in a more traditional manner and do not allow them to be active in any of the LGBTQ community affairs. Meanwhile, they are trying to figure out their own sexual identity and gender preferences. "Having my family's voice be so loud, drowns out my own," they share. What should a Solution Focused therapist do in formulating treatment goals? A. Define the problem; break down the goals into small steps to focus on each week; identify client strengths. B. Challenge the client's definition of the problem; compliment client competencies; visualize client's future C. Ask how the client has coped in the past and help relocate the resources; compliment client competencies. visualize client's future D. Clarify achievable goals with the client; identify client strengths; compliment client competencies.

D. Clarify achievable goals with the client; identify client strengths; compliment client competencies. Rationale: In the early stage of therapy, a Solution Focused therapist works with the client to clarify achievable goals and identifies the client strengths. Complimenting a client is a classic SF intervention (D). Solution focused therapists generally do not focus on client problems (A) or (B). Asking clients how they have coped in the past and identifying resources (C) is a middle stage intervention. (Case Conceptualization and planning)

Jose, a 32-year-old accountant, is referred to therapy by his EAP. He is approved for 8 treatment sessions. The referral from the EAP states that Jose has poor time management skills and recently had issues with his productivity. Jose shares that his boyfriend was recently transferred to another city for work and it has caused a strain on their relationship and his overall health. He shares that he likes his job but just can't concentrate like he used to. What action would a Solution Focused therapist take in the final phase of treatment? A. Process Jose's feelings about termination and compliment his progress and growth. B. Identify Jose's resources and strengths as well as identify possible setbacks and ways to stay on track. C. Identify what "productive employment" would look like for Jose and compliment his progress and growth. D. Compare Jose's scale of current functioning to the initial scale and identify possible setbacks and ways to stay on track.

D. Compare Jose's scale of current functioning to the initial scale and identify possible setbacks and ways to stay on track. Rationale: Answer (A) is a good answer, yet it is not specific to Solution Focused theory, and since that is what the question is asking, the ideal answer will include elements from the theory (if that option did not exist, this could be an acceptable option). Answer (B) is not the best answer since identifying strengths and resources are indicative of the middle stage of this theory. Answer (C) includes an intervention ("Identify what 'productive' would look like") that would be better fitting in the beginning stage of treatment. Answer (D) is the best answer since, as a Solution Focused therapist, we would utilize scaling and compare how the client is doing now as compared to how he was doing when the treatment began. The second half of the answer is also a fitting Solution Focused final stage intervention. (Treatment)

An adolescent client is mandated to receive counseling for anger management by her juvenile probation officer. Although the client is 14-years-old, her parents consented to treatment. The therapist has met with the client for three sessions and believes the client is finally beginning to trust him. In the fourth session, the client is coherent and cooperative, but the therapist believes he smells alcohol and notices the client is slurring some of her words. Which of the following actions should the therapist take in this case? A. Contact the parents since they consented to treatment and address safety concerns with them. B. Confront the client about suspicions and postpone the session. C. Inform the client you must notify the probation officer since she is a mandated client and has violated probation conditions. D. Confront the client about suspicions and proceed accordingly

D. Confront the client about suspicions and proceed accordingly Rationale: Answer D is the strongest answer based on the information provided in the vignette. The vignette notes the therapist believes the client has been drinking, but belief is different from certainty. The only answers that directly address the therapist's uncertainty are B and D. The only reason we would postpone/reschedule the session is if the client is truly under the influence and since we do not yet know that, answer B is incorrect. Following the confrontation, the therapist may choose to use the interventions noted in answers (A), (B), or (C). (Crisis Management)

A therapist is seeing a family of four, including a 15-year-old daughter and 12-year-old son. The stated goal of therapy is to decrease ongoing conflict between the siblings and improve communication. The mother, who has a history of trauma, has developed a positive relationship with the therapist and requests to concurrently meet with the therapist for individual therapy. Which of the following actions should the therapist take to manage the ethical issues involved in this case? A. Provide the mother with referrals to other therapists because it would be unethical to provide individual and family therapy concurrently. B. Continue family therapy and meet with mother individually while working to maintain clear boundaries C. Consider referring the family to another therapist because of the mother's severity of symptoms and level of rapport between the mother and the therapist. D. Consider the implications of concurrent individual therapy for the family and discuss them with the client.

D. Consider the implications of concurrent individual therapy for the family and discuss them with the client. Rationale: Some family therapists decide to work with family members individually and some decide to refer family members out for individual work. It would be important to discuss the implication of such a request and how it would affect the family before deciding how to proceed. Answer D fully addresses these considerations. Answer A is incorrect because providing concurrent therapy is not always unethical. Whether the therapist and the mother decide to work together (B) or the therapist decides to provide referrals (A) and (C), it would be important to first discuss the implication of such a request with the family (D) and consider the full clinical picture. (Law and Ethics)

A 33-year-old man who lost custodial rights with his two children following a CPS case is court ordered to receive individual therapy. The man must attend therapy as a precondition for visitation rights with his children being reinstated. In the first two sessions with the therapist, the client presents as guarded and distrustful of the therapist. He misses the next two sessions. What actions should the therapist take to address the client's missed sessions and court requirements? A. Close the client's file and provide a written report to the court documenting the client's number of sessions attended and termination due to lack of attendance. B. Call the client to reschedule appointments and inform him of the cancellation policy as well as amount owed for missed session fees. C. Close the client's file and provide him with a written report that he can share with the court regarding the number of sessions attended and date of termination. D. Contact the client to discuss the consequences of missed appointments and explore the client's feelings regarding therapy.

D. Contact the client to discuss the consequences of missed appointments and explore the client's feelings regarding therapy. Rationale: By calling to discuss the consequences of the missed appointments and exploring the client's feelings toward therapy, the therapist can work to build rapport and ensure the client understands what would happen if he did not comply with the order (D). This may or may not lead to termination. Assessing the client's motivation to continue and potentially rescheduling appointments (B) can be part of a phone call. If, after such conversation the client continues to miss, the therapist might provide a report to the client or court (A) as well as terminate therapy with the client (B). (Treatment)

After working together for several months, a therapist discovers that she attends the same church as her client. The client, a 76-year-old recent widow with ongoing health issues, asks the therapist to accompany her to weekly church services. The client explains to the therapist that it is difficult for her to get around and she does not have any support nearby. What interventions should the therapist consider? A. Deny the client's request; clarify the concept of dual relationships; document the interaction; empower the client to build a support network. B. Deny the client's request; discuss professional boundaries; address issues of dependency with client; empower client to build a support network. C. Deny the client's request; clarify the concept of dual relationships; document the interaction; refer the client to a support group for widows. D. Deny the client's request; discuss professional boundaries; process the client's reaction; empower the client to build a support network.

D. Deny the client's request; discuss professional boundaries; process the client's reaction; empower the client to build a support network. Rationale: This question is asking what interventions the therapist should consider. We are going to put the answers in order. Since there are a lot of repetitive answers we are going to compare the elements that are different. The first goal of the therapist should be to deny the request, discuss boundaries and process the client's reaction since such rejection might be painful for the client. We would also attempt to help the client find an alternative support network (D). (A) is not a good answer since documenting the interaction would come after processing the client's reaction. (B) is not a good answer since the client does not exhibit issues of dependency. (C) is missing an important element of processing the client's reaction, which would be an important step in this case. (Law and Ethics)

A 22-year-old client initiates therapy with complaints of anxiety and hopelessness. The client currently lives with her stepsister, is unemployed, and is attempting community college. She discloses that her father left her family when she was a baby and her mother died of an overdose two years ago. The client has a much older boyfriend who travels a lot for work and she thinks he is cheating. Which of the following areas should the therapist inquire after in order to complete a comprehensive assessment for this client? A. Gather family history; determine client's strengths; identify coping mechanisms. B. Explore suicide risk; determine somatic symptoms; identify familial coping patterns. C. Explore substance use; determine somatic symptoms; identify familial coping mechanisms. D. Explore suicide risk; determine client's strengths; identify coping patterns.

D. Explore suicide risk; determine client's strengths; identify coping patterns. Rationale: The client expressed hopelessness so suicide risk is essential and it narrows it down to B and D right away. D is more specific to the client and thus a stronger answer. (Clinical Evaluation)

A therapist who specializes in play therapy and is certified in art therapy meets for the first time with a 6-year-old girl. The parents are nervous about taking their daughter to therapy, but feel it's necessary due to ongoing concerns regarding their daughter's behavior at home and in school. During an initial session, while utilizing play therapy, the girl exhibits knowledge of sexual organs and how they function. The therapist believes such knowledge is unusual for her age. How should the therapist further assess the child in this case?: A. Elicit further information to complete a child abuse report; refer to a pediatrician for a physical exam. B. Schedule to meet with the girl's parents and ask if the child has been sexually abused; Meet with the girl's teacher for academic and social assessment. C. Explore the parents' concerns about the girl's behavior; refer to a pediatrician for a physical exam. D. Explore with the parents what types of behaviors the girl is exhibiting; ask the child about the source of her knowledge.

D. Explore with the parents what types of behaviors the girl is exhibiting; ask the child about the source of her knowledge. Rationale: The therapist needs to get clarification from the parents about what type of behaviors concern them and understand the origins of the client's knowledge before determining if there is reasonable suspicion of abuse, answer D. Answer A makes assumptions about the girl's behavior and origin of knowledge, neither are known based on information in the vignette. Answer B is again jumping to conclusions without first gathering basic information. Answer C ignores the girl's sexual knowledge, which is explicitly noted as unusual by the therapist. (Clinical evaluation)

A therapist receives a school-based referral for a female 14-year-old. Her teacher told the therapist that the student has been increasingly volatile in interactions with peers and school staff. The student's mother insists that she be a part of treatment as she has also noticed behavioral problems at home. The mother informs the therapist that her daughter was sexually abused at an early age and has never talked about it with anyone. The daughter denies any memory of the abuse. She states that she does not want to see herself as broken or damaged in any way. How should the therapist proceed? A. Discuss the impact of the abuse on the family and the daughter's behavior; gather more information regarding the daughter's symptoms both at home and at school. B. Clearly identify the treatment unit before developing a treatment plan; gather additional information regarding the abuse. C. Discuss the impact of the abuse on the family and Lara's behaviors; clearly identify the treatment unit before developing a treatment plan. D. File a child abuse report; gather more information regarding the daughter's symptoms both at home and at school.

D. File a child abuse report; gather more information regarding the daughter's symptoms both at home and at school. Rationale: The question is asking what actions the therapist should take in this case. The information shared in the session indicates the need for the therapist to immediately file a child abuse report since the victim is still a minor, regardless of whether or not the daughter acknowledges the abuse; the therapist has enough for reasonable suspicion. This is a good example of a question where the answers are all correct, but the priority is to report the child abuse report. We can then move on to the actions identified in answers A, B, and C. (Crisis management)

A therapist decides to increase her marketing efforts for her private practice where she specializes in treating postpartum depression. She creates a brochure about her psychotherapy practice and solicits testimonials from former clients to include in the brochure. She asks the clients to share how much her therapeutic skills have helped them overcome their depressive symptoms, regardless of where they are in treatment. The therapist's actions are: A. Acceptable because therapists are permitted to solicit testimonials. B. Acceptable if she obtains written authorization and removes any identifying information of the clients, thus protecting their right to confidentiality. C. Unethical since she solicited client's testimonials and this would place an inappropriate burden on the client. D. Illegal and unethical because she solicited testimonials and doing so might create unreasonable expectations of successful treatment for potential new clients.

D. Illegal and unethical because she solicited testimonials and doing so might create unreasonable expectations of successful treatment for potential new clients. Rationale: The Code of Ethics states that MFTS may not solicit testimonials from those clients/patients who, due to their particular circumstances, are vulnerable to undue influence. While this language technically allows therapists to solicit testimonials, it is hard to say who is or is not vulnerable to undue influence. In addition, in this particular situation, the therapist is asking for very specific testimonials, which would not allow the clients to share their own experiences honestly. The therapist is exerting influence on the testimonial. For these reasons, this scenario is unethical. It is also illegal since the therapist is pushing clients to make specific statements for advertising that could be misrepresentative of their experience and misleading to others, creating unreasonable expectations of successful treatment. Answer A is incorrect because it is an incomplete representation of what occured in this situation. Answer B does not address the pressure being exerted by the therapist onto the clients and is also wrong. Answer C is a possibility, but it is not as comprehensive an answer as D. Therefore, the correct answer is (D). (Law and Ethics)

A 32-year-old male with an extensive criminal history has been in therapy for several months. His probation officer strongly encouraged him to receive therapy because of ongoing anger management and impulse control issues. The client informed the therapist at the onset of treatment that he was concerned about sharing information that could be used against him with his probation officer. In a session, he reveals the fact that he robbed a store 3 days ago. Legally, the therapist should: A. Determine if the client physically threatened anyone and consider making a police report. B. Inform the client that his probation officer must be informed since it is a violation of his probation. C. Maintain confidentiality and process the consequences of the client's criminal activity. D. Maintain confidentiality and process the incident with the client.

D. Maintain confidentiality and process the incident with the client. Rationale: Making a police report, even if someone was harmed (A) or informing the probation officer (B) would break the client's confidentiality that we are legally required to maintain. A would only be true if the stem indicated the client harmed someone and that victim was part of a protected class. Answer C is incorrect because we would want to more generally process the incident with the client rather than focus on the specific consequences. Therapists are not legally required to break confidentiality and report past acts of violence, thus answer option (D) is correct. (Law and Ethics)

Maria, a Mexican-American 40-year-old divorced mother of four, is referred to therapy by her minister. Maria shares that she has been very overwhelmed. She is growing increasingly worried because her oldest child, a 19-year-old son named Enrique, has been sleeping with a knife under his pillow, is acting out of control, and even threatened her. Maria reports a recent promotion at work, but adds that she is having a hard time concentrating and worries she may get in trouble. She says, "I don't want my family to know how bad it is; they would be so upset if they knew I came to therapy. I keep praying that he will get better." What human diversity issues should the therapist consider when developing a comprehensive treatment plan? A. Maria's level of family support and her motivation for treatment. B. Maria's recent promotion and the severity of Maria's anxiety. C. Maria's spiritual beliefs and the safety of the other children in the home. D. Maria's cultural views of therapy and her spiritual beliefs.

D. Maria's cultural views of therapy and her spiritual beliefs. Rationale: (D) is the only answer that has only human diversity issues. Levels of social support, motivation for treatment, severity of symptoms, her promotion and the safety of the children are all present in the vignette and could be part of the treatment plan, but the question is specifically asking about human diversity issues that will be part of the treatment plan. (Case Conceptualization and Planning)

A therapist meets with an older couple, Stefan and Mila. The couple reports they are experiencing increased arguments and distance between them. "I have to poke and prod him all day in order to get anything out of him," says Mila. "It is like living with a wall," she adds while her husband is looking down at his feet. During one session, the husband finally tells the wife that the reason he is so shut down is because a few weeks ago, when he told her that he was getting depressed, she told him to talk to the therapist instead. "It was clear that telling you about my depression was just a burden for you. So I figured I would not bother you anymore, "he says. What formulation would an EFT therapist offer to this couple? A. This distance you have been experiencing is a pretty powerful problem you are dealing with. How have you managed to keep the distance from getting even bigger between you? B. Stefan, it sounds like you believe that Mila does not want to hear about anything you are going through since she did not want to hear about your depression, is that right? C. Have you ever had an issue with communication or feeling distant with one another in the past? How were you able to resolve it? What would you need to do to get that to happen again? D. Mila, it sounds like you miss your husband and to regain a sense of connection are poking and prodding Stefan to try reaching him. Stefan, you want to know that your pain matters to Mila, and when you thought you were a burden, you decided it would be safer to stop being vulnerable.

D. Mila, it sounds like you miss your husband and to regain a sense of connection are poking and prodding Stefan to try reaching him. Stefan, you want to know that your pain matters to Mila, and when you thought you were a burden, you decided it would be safer to stop being vulnerable. Rationale: The answer that reflects the attachment perspective of EFT is (D). Answer (A) is reflective of a Narrative theory approach. Answer (B) describes an example of overgeneralization, which is a cognitive distortion. This is something a Cognitive therapist would say. Answer (C) is an example of Solution-Focused questioning. (Treatment)

A therapist is treating Arturo, a 29-year-old construction worker. Arturo has a history of anger issues manifested by fights and physical altercations throughout his twenties. During the latest session, Arturo reports that a new boss laid him off from a job he's had for four years. The client denies thoughts of hurting the boss, but is enraged when he talks about him. The following day, Arturo's wife leaves a message informing the therapist that her husband didn't sleep at all last night, was up drinking, and plans to "surprise the boss" after work. How should the therapist clinically manage the situation? A. Contact the wife to verify the threat and document both the content of the message and the conversation with her in the client's medical record. B. Document the content of the message and contact the client's former employer to report the danger. C. Contact the client to assess risk since the information regarding the threat cannot be taken seriously if it comes from a third party. D. Notify local law enforcement and the client's former employer of the threat only revealing the information that is relevant to the situation at hand.

D. Notify local law enforcement and the client's former employer of the threat only revealing the information that is relevant to the situation at hand. Rationale: There is enough information to justify breaking confidentiality, and time might be of essence, thus (A) is not the best answer. Family members with information about danger to others can trigger a "duty to protect" situation, thus (C) is out. The best answer is (D) because there is enough information to justify breaking confidentiality. You MUST notify police so that makes (D) a better answer than (B). In addition, the law requires that we still honor the client's confidentiality and only reveal the information that communicates the severity of the risk. (Crisis Management)

A therapist receives a phone call from a 58-year-old married woman who is distressed by her husband's recent inability to sustain an erection. She reports that her husband has suffered from depression and was recently prescribed Paxil. She says the physician has explained how SSRIs can cause this problem but she tells the therapist, "It's not because of the medicine, it's because he no longer loves me." What interventions would the therapist consider in creating a treatment plan in this case? A. Offer individual therapy to the client; add a comment in client's psychotherapy notes reminding the therapist to assess client's attachment history; work to address the client's distorted beliefs and assumptions B. Offer individual therapy to the client; refer the client to NAMI to help her learn about the effects of SSRIs and depression on libido; prioritize addressing the client's beliefs and fears about the way her husband feels about her. C. Offer conjoint sessions to the client and her husband; suggest that the husband seek a second opinion; refer the client to NAMI to help her learn about the effects of SSRIs and depression on libido D. Offer conjoint sessions to the client and her husband; work collaboratively with the husband's prescribing health care provider if seeing them together; add a comment in client's psychotherapy notes reminding the therapist to assess client's attachment history

D. Offer conjoint sessions to the client and her husband; work collaboratively with the husband's prescribing health care provider if seeing them together; add a comment in client's psychotherapy notes reminding the therapist to assess client's attachment history Rationale: The woman is struggling with changes to her marital relationship; therefore, conjoint therapy (D) would allow both the husband and wife to explore their feelings. In addition, according to the standard of care it is best practice to collaborate with other healthcare providers involved in the case. The client's comment at the end is possibly pointing to attachment wounding, thus gathering the client's attachment history would be an important part of creating a treatment plan. Individual therapy (A) would only address one partner in a shared issue, plus there is no evidence of the client's distorted beliefs and assumptions in the stem. The woman has already received education from the doctor regarding SSRIs, so sending her to the National Alliance for Mental Illness (an organization that provides resources to individuals with mental health disorders and their families) would not be effective (B). Suggesting the husband seek a second opinion (C) is unnecessary and premature. (Case Conceptualization and Planning)

A therapist works with a university student who reports compulsive exercise, as well as binging and purging several times per week. The behaviors have been occurring for four months. In Addition, the client reports nightmares, worrying about everything, and continually feeling "on edge." The client says she broke up with her boyfriend over a month ago after a violent fight during which he tried to choke her. Which of the following provisional diagnoses should the therapist consider in this case? A. Nightmare disorder; Anorexia Nervosa; Adjustment disorder with anxiety B. Adjustment disorder; Bulimia Nervosa; Unspecified Anxiety Disorder C. Acute Stress Disorder; Unspecified Eating Disorder; Generalized Anxiety Disorder D. Posttraumatic Stress Disorder; Bulimia Nervosa; Unspecified Anxiety Disorder

D. Posttraumatic Stress Disorder; Bulimia Nervosa; Unspecified Anxiety Disorder Rationale: According to the DSM, the symptoms described in the stem are indicative of PTSD, Bulimia Nervosa and Unspecified Anxiety Disorder (D). The symptoms listed are better accounted for by PTSD than Nightmare Disorder or Adjustment Disorder and do not meet the criteria for Anorexia Nervosa (A) and (B). In answer C, the timeline is too long for Acute Stress Disorder, and symptoms are better accounted for by Bulimia Nervosa and Unspecified Anxiety Disorder than Unspecified Eating Disorder or Generalized Anxiety Disorder. (Diagnosis)

A therapist has been working with a client for over two years with a focus on the client's social anxiety. The therapist believes the client has successfully completed therapy and begins the process of termination. The client's initial reaction is one of surprise. The client states a lack of readiness and appears to feel rejected by the therapist. What actions should the therapist take to address the client's reaction? A. Normalize the client's feelings toward termination and contract for additional sessions. B. Process client's feelings of rejection and collaborate with client to develop new treatment goals. C. Normalize client's reaction to termination and review client's strengths and weaknesses. D. Process client's feelings in response to termination and focus on goals accomplished.

D. Process client's feelings in response to termination and focus on goals accomplished. Rationale: It is common for clients to lose sight of the reason for treatment (symptom reduction and improvement of functioning) and it is therefore important to keep the client aware of the goals of treatment and stay focused on them (D). It would also be important to process their feelings to alleviate feelings of rejection. If therapy is successful and goals have been achieved, it would not be appropriate to contract for additional sessions (A) or develop new treatment goals without a clear need (B). Reviewing a client's strengths and weaknesses is not wrong, but it is not as helpful to the termination process as directly reflecting on goals achieved (C). (Treatment)

Nathan, age 5, is brought to therapy by his mother Linda because he has been acting out at kindergarten. The school told Linda that Nathan won't stay in his seat, frequently interrupts the class, and has been aggressive with his peers. Linda shares that the family recently moved across the country to live with Linda's parents because Nathan's father was physically abusive toward both her and Nathan. Linda starts to cry and says, "I know why he's acting this way." After informing Linda that a child abuse report must be filed, how should the therapist proceed in this case? A. Process her feelings about the report and refer her to a domestic violence support group. B. Explore the nature of the abuse and obtain a release to speak to the school. C. Explore the nature of the abuse and observe how Linda interacts with her son. D. Process her feelings about the report and instill hope that therapy can help her son process the trauma he has experienced.

D. Process her feelings about the report and instill hope that therapy can help her son process the trauma he has experienced. Rationale: The best answer is (D). You would want to start by processing the client's feelings and then instill hope that you can help her son. All the other components in the other answers could be done at some point, but instilling hope would be important in this case. It's important to note that the answer is not claiming the therapist can make everything better, but is just saying that the son will have a place where he can process what has happened. (Treatment)

A five-year-old child presents for therapy after she started wetting the bed again. During the initial assessment, the mother explains that this behavior started soon after she gave birth to her third child. Which of the following interventions should the therapist first take? A. Begin a course of play therapy to address the regressive behaviors being exhibited by the child. B. Assess for potential sexual abuse due to enuresis being a common sign of abuse. C. Refer the child for a medical evaluation to rule out any underlying biological causes. D. Provide psychoeducation on normal child development and possibility of regressive behaviors being linked to recent changes in the family.

D. Provide psychoeducation on normal child development and possibility of regressive behaviors being linked to recent changes in the family. Rationale: Children will often revert to an earlier developmental stage when attempting to cope with stress. It is important for the mother to understand normal child development and to discuss the recent changes(D). Following the provision of psychoeducation, the therapist might receive additional information that would indicate the need to begin play therapy (A), to assess for sexual abuse (B), or to refer the child for a medical evaluation (C). However, based on the information provided, taking any of these steps prior to psychoeducation would be overreactions. (Treatment)

A frail 91-year-old female nursing home resident has begun to refuse some meals. A medical evaluation determined the need for a gastric feeding tube. Although her son is in favor of having the feeding tube placed, the resident refuses. The son demands that the resident's therapist working in the hospital should get the doctor to put in the feeding tube. What initial step should the therapist take in this case? A. Refer the son for treatment to cope with his mother's refusal to eat. B. Explain to the son the process of becoming his mother's conservator. C. Explain to the son that his mother has a right to self-determination. D. Refer the resident for a competency evaluation.

D. Refer the resident for a competency evaluation. Rationale: The strongest answer is D. Client's generally have a right to self-determination, unless it is determined that they lack competency to make decisions for themselves. Based on what is provided in the vignette, we do not know if the woman is capable of making health decisions for herself or not. Therefore, our starting point would be to refer her for a competency evaluation. This would need to happen before we can do what is described in answer C or B. Answer A is not a priority, since we want to focus on answers that focus on the resident who is the actual client. (Crisis Management)

A couple is seeing a therapist for marital problems. The couple has a history of infidelity and resulting lack of trust. During a session, the wife compliments her husband. The husband responds to the wife's compliment by stating, "It's nothing" and avoids eye contact. Which of the following interventions would a Systems therapist use to address the observed interaction between the husband and wife? A. Explore the wife's reason for the compliment and how the lack of trust inhibits the husband's ability to accept it. B. Assist the husband in accepting the compliment and explore how it affects him emotionally. C. Focus on the husband's inability to accept praise and explore the role trust plays in this dynamic. D. Share the observation of their interaction and investigate the impact of the wife's compliment on each of them.

D. Share the observation of their interaction and investigate the impact of the wife's compliment on each of them. Rationale: The therapist can help the couple develop a greater awareness of their interactions by sharing his/her observation and facilitating an exploration of this dynamic (D). In addition, the interventions included in answers (A), (B), and (C) are not as strong because they only address one member of the treatment unit rather than the entire treatment unit--the couple. Furthermore, A is also wrong because it assumes the husband is lacking trust. (Treatment)

Silvia, age 16, is referred by her probation officer for court-mandated therapy. She has been arrested three times for assault, stealing, and drug possession. She was recently released from a juvenile correction camp where she spent six months. She is currently residing in a group home. Silvia states, "Don't expect me to share stuff with you. The only people I trust are my homies; my gang is my family." Silvia has a history of marijuana, crystal meth, and alcohol abuse. She adds, "Can you help me sleep? I can't shut my brain off at night. I just think of all the people who have wronged me and what I want to do to them. There's a girl trying to get with my man and if she does, I'm going to cut her." She is currently enrolled in a GED program but reports that she's never been good at school and just can't focus in class. She jumps in her seat when the office phone rings. What additional information should be evaluated to help the therapist decide the course of treatment? A. Silvia's arrest history and her family substance use history. B. Silvia's trauma history and past substance use. C. Sylvia's arrest history and current substance use. D. Silvia's trauma history and current substance use.

D. Silvia's trauma history and current substance use. Rationale: In order to decide on the course of treatment, we need to identify this client's diagnosis and any crisis issues that we might be dealing with. Answer (A) includes Silvia's arrest history and we already know how many times and what she was arrested for, so there is no need to get more information about it. Plus, we are much more interested in her personal substance use history, as opposed to her family's, since she is the client. The first part in answer (B) is correct - it is important to know the client's history of trauma in order to determine her diagnosis and appropriate treatment plan. We see that the client jumps when the phone rings, has a history of violence, and has been involved in a gang, all of which point to a possible trauma history. The second part of Answer (B) is not as relevant as the second part of answer (D), since we would want to find out how much and what she is currently using; we already know her history with meth, alcohol, and marijuana. Answer (C) has the current substance use, but the first half, as discussed, is already known. Answer (D) is the best answer as it prompts us to find out more about her history of trauma and her current substance use, both of which will help in determining the course of treatment. (Clinical Evaluation)

A 34-year-old woman seeks treatment for increasing symptoms of anxiety. She is the single mother of a 4-year-old girl, after having divorced from her daughter's father 3 months ago. She reports that their separation was mutual and they continue to be friends, but she states that her feelings of anxiety are increasing. She is increasingly concerned about her ability to be a single parent and has recently been considering sending her daughter to live with the father. She reports feeling disconnected from her daughter and unsure of how to "get things back to normal." Which of the following interventions should the therapist consider when addressing the client's anxiety? A. Encourage the client to discuss the benefits of psychotropic medications with a medical doctor. B. Offer a joint session with the client's ex-husband to identify co-parenting techniques that would address the client's concerns. C. Assign the client to read psychoeducational books on divorce and co-parenting. D. Teach the client calming skills and ways to better discriminate between a relaxed versus tense state of being.

D. Teach the client calming skills and ways to better discriminate between a relaxed versus tense state of being. Rationale: This question is asking for interventions that directly address the client's anxiety, so we want to find an answer that specifically focuses on this issue--the more focused specifically on anxiety, the better. Answer (A) is incorrect because it's not clear psychotropic medicine is appropriate and the way it is worded is not quite right, as it only encourages the client to discuss benefits, without providing a balanced approach. Answer (B) may or may not help the client's anxiety, so while it may be something the therapist could do, it's not a very strong option. Answer (C) is similar to (B), it could help, but does not specifically address anxiety. Answer (D) is the strongest option because it most directly addresses the client's anxiety, which is what the question is asking about. (Treatment)

A couple brings in their 13-year-old daughter to see a therapist in private practice. During the first session the mother tells the therapist that they have been wanting to bring the daughter in to see someone for a couple of years now, but she has been refusing to come to therapy until now. The parents complain their daughter is moody, irritable, and she reports no one at school likes her. Although these things have been going on for some time, it is the daughter's recent demands that made the parents decide the situation was intolerable and brought the family to therapy. About two weeks ago she told her parents she wants to quit some of her extracurricular activities, as well as change schools. How would a Cognitive Behavioral Therapist versus a Solution Focused Therapist initially assess this client? A. The CBT therapist would collaborate with the client to identify the cognitive distortions; the solution focused therapist would collaborate with the client to identify reasons she believes no one likes her B. The CBT therapist would identify the problems confronting the client and collaborate to develop treatment plan; the solution focused therapist would ask the client how they will know things are better C. The CBT therapist would collaborate with client to identify the symptoms; the solution focused therapist would ask the client how they would know things are better D. The CBT therapist would collaborate with the client to define the problem; the solution focused therapist would ask the client how she would know things were better

D. The CBT therapist would collaborate with the client to define the problem; the solution focused therapist would ask the client how she would know things were better Rationale: Answers (A) is out because identifying cognitive distortions is not part of the initial assessments for CBT and solution focused does not focus on the problem. Answer B is incorrect because the therapist would not identify the problem and developing the treatment plan is not a part of assessment. That leaves answers C and D. (D) is a better answer because the first part of the answer for CBT gives a fuller picture of the issue rather than just identifying the symptoms. In CBT, it is called a functional analysis when the therapist understands when and where someone has symptoms so that the therapist can really understand and "define the problem." (Clinical evaluation)

A therapist admits to an intimate relationship with one of his former clients. The therapist is the subject of regulatory board investigation. He tells the board investigator that the therapeutic relationship ended not quite two years ago, he truly loves this woman and they are engaged to get married. When the client is interviewed, she reports that she initiated the sexual contact with the therapist and reciprocates his feelings. The therapist terminated therapy to avoid any conflict once they realized they had emotions for each other. Which of the following descriptions most accurately represents the therapist's actions? A. It is considered a boundary violation, but not something the board will pursue due to the couple's reciprocal feelings and lack of harm to client B. The actions of the therapist are legal due to the amount of time that has passed, yet it continues to be unethical. C. The therapist's actions are both legal and ethical since the client initiated the relationship, the therapist ended the therapeutic relationship, and no harm or exploitation is indicated. D. The therapist's actions are considered illegal and unethical due to termination of therapy and the amount of time that has lapsed.

D. The therapist's actions are considered illegal and unethical due to termination of therapy and the amount of time that has lapsed. Rationale: The actions described in this stem are both illegal and unethical (D). The therapist's behavior is considered a violation of boundaries, but the second part is incorrect because the board would pursue action against the therapist regardless of the couple's ongoing relationship. Answer (B) is inaccurate as the actions not just unethical, but illegal as well. Answer (C) is inaccurate because it would not matter if the therapist or client initiated the relationship. (Law and Ethics)

A therapist is working with a woman from Mexico who speaks limited English and shares, with fear in her voice, that she is undocumented. The client was referred by her friend after the friend noticed bruises and contusions on the client's body. The client shares that her husband is "extremely abusive," but has threatened to have her deported if she tells anyone. The woman also notes that she comes from a family with a history of domestic violence and says this is just a "normal" part of relationships. What actions should the therapist take in this case? A. Explore the client's fear of being deported and connect the client to legal resources available for undocumented victims of crime. B. Explore the cultural context of DV and refer the client to a shelter to help end the cycle of abuse. C. Validate client's expressed feelings of normalcy regarding DV and collaboratively develop a safety plan. D. Validate client's fears of being deported and develop a comprehensive safety plan including referrals to legal counsel.

D. Validate client's fears of being deported and develop a comprehensive safety plan including referrals to legal counsel. Rationale: The best answer in this case is answer (D). The client has indicated that she is undocumented and her husband threatened to have her deported if she told anyone he was abusive. Plus,there is a tone of fear noted in the vignette. In addition, developing a safety plan and linking her to legal resources so she understands her legal rights would be important. Answer A is not as comprehensive as (D), so it can be ruled out. Answer (B) is a possibility, but again, a safety plan is a better option than a shelter referral. Answer (C) is incorrect because we would not want to validate domestic violence being normal or acceptable. (Crisis Management)

A self-referred client tells a therapist during the initial intake that he is dissatisfied with his current job and is interested in finding a new one. He shares that he has worked at his current company for several years and wants to change careers, but fears he is limited in his skills and is too old to start anew. What should the therapist further assess? A. Vocational history; Need for vocational counseling; History of depression B. Vocational history; Educational attainment; Career Interests C. Vocational and social history; Severity of anxiety; Career Interests D. Vocational and social history; Therapeutic expectations; Need for vocational counseling

D. Vocational and social history; Therapeutic expectations; Need for vocational counseling Rationale: The best answer here is D. Vocational history is certainly going to be something we are going to assess due to the presenting issue. It is going to be important to make sure that this person has appropriate expectations for therapy and knows the services that we are and are not able to provide. And due to the nature of the presenting issue and the fact that we are not able to offer vocational counseling as part of regular therapy, we would also need to assess the need for a vocational counseling referral. Answer A can be eliminated since there is no indication of depression in the stem. Answer B is out since career interests and educational attainment are going to be done as part of vocational counseling. Answer C is incorrect since anxiety is not indicated in the stem and career interests should be covered in vocational counseling. (Clinical Evaluation)


Conjuntos de estudio relacionados

Research: Finding and Evaluating Sources

View Set

Fundamentals of Business Final Exam 2023

View Set

Chapter 6: Analyzing the Audience, COMM 1010 CHP. 6, Chapter 6, public speaking chapter 6

View Set